Sunteți pe pagina 1din 88

Recrea tii stiin tifice - cea nt ai brazd a

La 15 ianuarie 1883 apare la Ia si primul num ar al revistei "Recrea tii stiin tifice", revist a ce va dura timp de sase ani, cu cte un num ar pe lun a. Obiectivele urm arite, racordate la cerin tele din acele timpuri ale nv a t amntului, sunt "de o n telepciune ce face cinste fondatorilor revistei" [6] si apar expuse cu deosebit a claritate n "C atr a cetitori" - cuvtul de nceput din 15 ianuarie 1883:

Apari tia revistei "Recrea tii stiin tifice" este strns legat a de condi tiile istorice din acea epoc a. Unirea Principatelor si Proclamarea Independen tei Romniei, ct si reformele 1

structurale din timpul domniei lui Al. I. Cuza si apoi a regelui Carol I au creat cadrul politic si legislativ al form arii statului romn modern si afirm arii acestuia. Inv a t amntul romnesc, ntocmai ca si societatea romneasc a n general, a trecut prin mari prefaceri si fr amnt ari: nfiin tarea celor dou a universit a ti din Ia si si Bucure sti, reforma nv a t amntului din 1964, multele regulamente menite s a organizeze re teaua de scoli si s a stabileasc a programele acestora etc. Pe de alt a parte, licen tia tii romni ai universit a tilor apusene (din Fran ta, Germania sau Italia) odat a reveni ti n tar a realizau imediat faptul c a, n climatul existent, greu ar fi putut ntreprinde cercet ari originale proprii. Acestora le revenea obliga tia cu mult mai important a pentru acel moment de a contribui la edificarea nv a t amntului romnesc, de a tine lec tii si a elabora manuale si cursuri n limba romn a, de a preg ati genera tia viitoare ce urma s a fac a pasul c atre crea tia stiin tific a original a. Fondatorii reviste sunt: N. Culianu, C. Climescu si I. Melik - profesori la Facultatea de stiin te din Ia si, G. I. Lucescu si V. Paladi - profesori de matematic a la Liceul Na tional coala Militar din Ia si, G. I. Ro siu si I. D. Rallet - profesori de matematic a la S a din Ia si, coala Militar G. Zarifopol - profesor de fizic a si chimie la S a din Ia si, I. V. Praja - profesor coala Normal de matematic a la S a "Vasile Lupu" din Ia si si I. M. Dospinescu - profesor de matematic a la Gimnaziul " Stefan cel Mare" din Ia si. ar tiin Este prima revist a din t a cu prol s tic , materialele publicate acoperind diversele ramuri ale stiin tei: matematic a, zic a, chimie, mineralogie, geograe, astronomie, cosmograe etc. Se adreseaz a cu prec adere elevilor din scolile secundare, studen tilor si profesorilor. Majoritatea fondatorilor erau licen tia ti n matematic a sau profesau aceast a disciplin a; tiin se explic a astfel faptul c a "Recrea tiile s tifice" au un con tinut predominant matematic ; multe numere de revist a au un con tinut exclusiv matematic. Un num ar are n medie 25 pagini; doar n anii II, V si VI numerele 7 si 8 (de vacan t a) au fost tip arite mpreun a, ca o singur a revist a de 40 pagini. Structura de baz a a unui num ar este: articole, probleme rezolvate si probleme propuse. Date ind condi tiile n care a ap arut, materialele publicate n paginile revistei, articole si probleme, erau preluate sau erau prelucr ari din tratatele si revistele de circula tie din acea vreme. Articolele de matematic a vizeaz a diversele ei ramuri: aritmetic a, algebr a, geometrie (sintetic a, analitic a, descriptiv a si diferen tial a), mecanic a teoretic a si astronomie, matematic a actuarial a, istoria matematicii, problemele nv a t amntului matematic. Problemele propuse au trezit un viu interes printre elevii din Ia si si din toat a tara; pleca ti la studii n str ain atate, unii dintre fo stii colaboratori ai revistei au continuat s a trimit a solu tii. Pentru stimularea elevilor, redac tia public a solu tiile corecte primite si men tioneaz a numele tuturor celor care au dat-o. Sunt tip arite liste de rezolvitori n ordinea num arului de probleme rezolvate. n "C atr a cetitori" din nr. 1/1885, redac tia revistei apreciaz a c a rezultatele ob tinute n primii doi ani sunt pozitive: "[...] rezultatele la care am ajuns snt n destul de mul t amitoare. O mi scare n aceast a direc tie, ntre elevii eminen ti din scoalele noastre, putem zice c a s-a determinat. Un num ar nsemnat de tineri ne trimet regulat solu tii, dintre care unele 2

destul de ingenioase". Nu ne propunem s a facem o analiz a a con tinutului revistei (v. [6],[7],[10]), vom selecta doar cteva aspecte ce consider am c a sunt semnificative si interesante. C. Climescu , sufletul revistei "Recrea tii stiin tifice", a publicat multe articole si din domenii diverse. n ciclul "Cteva curbe celebre si importante", nceput n vol.II si ncheiat n vol.III, expune principalele curbe plane clasice: cisoida lui Diocles, concoide, cicloide, spirale etc. I. Melik public a n nr. 3/1883 articolul "Despre scrierea numerelor cu litere chirilice". G. I. Ro siu traduce (dup a o edi tie italian a) prima carte a "Elementelor" lui Euclid si public a n vol.II si III ale revistei. Preciz am c a traducerea complet a n limba romn aa "Elementelor" a fost f acut a mult mai trziu de Victor Marian si publicat a n Biblioteca Gazetei Matematice n trei volume, 1939-1941. I. D. Rallet contribuie cu articole variate: maxime si minime geometrice, propriet a ti ale patrulaterului circumscriptibil, formulele fundamentale ale trigonometriei sferice, determinan ti, echilibrul unui punct material etc. G. I. Lucescu public a printre altele un studiu amplu si documentat despre m asurarea timpului si calendar, iar I. V. Praja abordeaz a chestiuni de aritmetic a (probleme de amestec s. a.), geometrie (transversale, poli si polare s.a.), analiz a etc. Revista "Recrea tii stiin tice" a reu sit s a atrag a colaborarea unor eminen ti profesori din acele timpuri: Miltiade Tzony - Facultatea de stiin te din Ia si, a publicat "Un curs de probleme", ce este prima culegere de probleme de mecanic a teoretic a din tar a (98 probleme); P. Tanco - profesor din N as aud, cu chestiuni de lozoa matematicii si de calendar; Constantin Gogu - Universitatea din Bucure sti, cu cteva scrisori despre calendar; Iacob Solomon - inginer, cu chestiuni de istoria matematicii din antichitate; profesorii din Ia si V. Bu tureanu (mineralogie) si August Scriban (geograe) s.a. Nu pu tini sunt aceia care, n drum c atre o str alucit a carier a, au fost n tinere tea lor activi rezolvitori ai "Recrea tiilor stiin tice": Ermil Pangrati - profesor de geometrie descriptiv a si rector al Universit a tii din Bucure sti, Anastasie Obreja - creatorul scolii de chimie organic a din Ia si, Vasile Cristescu - unul din cei patru "stlpi" ai Gazetei matematice, Dimitrie Pompeiu - ilustrul matematician romn, Petre Culianu, Gr. G. Stratilescu si mul ti al tii. Rezolvitorii erau mai ales elevi ai liceelor si scolilor militare sau studen ti si proveneau din toate col turile t arii (de atunci!): Dorohoi, Bac au, Brlad, Foc sani, Bucure sti, Craiova. Apar si rezolvitori cu profesii mai dep artate de matematic a: un preot din Bucure sti, un profesor de limba francez a din Bac au, o persoan a ce semneaz a cu "Vrfu cu dor" etc. Cu o munc a sus tinut a si mari sacrificii materiale, redactorii au scos la timp si au asigurat revistei un nivel de calitate nalt. Ei sunt pe deplin con stien ti de rezultatele ob tinute si de faptul c a prin munca lor au apropiat momentul apari tiei lucr arilor originale. Acest lucru rezult a f ar a echivoc din cuvntul redac tiei "C atr a cetitori" la nceputul anului al VI-lea:

Greut a tile materiale fac ca odat a cu publicarea num arului din decembrie 1888, adic a exact dup a sase ani, revista s a nceteze s a apar a. Se cuvine s a men tion am c a sus tin atorul principal al "Recrea tiilor stiin tifice" a fost C. Climescu. Pe coperta revistei din al VI-lea i Administra an este scris: "Redac tia s tia la Dl. C. Climescu, Profesor la Facultatea de te, Strada Butu 22". Aceea Stiin si adres a apare si n casetele ce urmeaz a titlul n fiecare num ar din ultimul an de apari tie (cum se poate vedea si n reproducerea de mai sus). 4

Fo sti rezolvitori ai "Recrea tiilor stiin tifice" si vor aduce aminte cu recuno stin t a de aceasta. Peste timp, mari matematicieni romni vor avea cuvinte de apreciere pentru curajul, sacrificiile si fapta celor care "au tras cea nt ai brazd a". Peste nici sapte ani de la dispari tia lor, "Recrea tiile stiin tifice" si afl a o continuare n "Gazeta matematic a", ai c arei fondatori si colaboratori au stiut si au reu sit s a nving a imense greut a ti si obstacole si s a fac a din visul cercet arii originale o realitate; aceasta si-a serbat un veac de existen t a nentrerupt a si este mereu tn ar a. Prof. dr. Temistocle BRSAN

Recrea tii stiin tifice prezen t a n con stiin ta posterit a tii


Rezolvirea problemelor este unul din cele mai bune stimulente pentru a atrage pe cineva c atre studiul matematicilor. Experien ta noastr a personal a ne probeaz a lucrul tiin acesta. Mai mul ti dintre noi datoresc acest gust revistei Recrea tii S tifice ce a ap arut n timp de 6 ani la Ia si si pe care noi ncerc am a o continua. Redac tia ["Gazetei matematice"] [2, p.1] Pe cnd la Bucure sti se petreceau aceste prefaceri de societ a ti stiin tifice, profesorii de stiin te din Ia si se hot ar asc s a scoat a o revist a stiin tific a, si anume Recrea tii stiin tifice care a ap arut cu mari greut a ti sase ani, de la 1883-1888. Revista consacra cea mai mare parte matematicilor si propunea probleme pentru folosul liceenilor. Aceast a revist a a contribuit mult la r aspndirea gustului pentru studiul matematicilor la noi n tar a. Ion Ionescu [3, p.11] Cea dinti ncercare de a ie si din acest impas, de a rupe iner tia, de a determina un curent de preocupare stiin tic a si de a crea astfel un nceput de atmosfer a prielnic a dezvolt arii stiin tei matematice, a fost f acut a la Ia si prin publicarea Recrea tiilor stiin tice. teica [4, p.69] Gheorghe Ti Omagiu pios primilor pionieri ai studiilor matematice la noi; omagiu pios acelei vechi tiin reviste matematice Recrea tii S tice si vrednicilor ei fondatori si colaboratori; omagiu Gazetei Matematice si acelor care au ntemeiat-o si sus tinut-o prin colaborarea lor pn a azi. Gr. G. Stratilescu [5, p.364] "Recrea tiile stiin tifice" au ap arut n 1883 viznd obiective de o n telepciune ce face cinste fondatorilor revistei. [...] Discret si oarecum nea steptat cum a ap arut, apune, la 15 decembrie 1888, prima revist a stiin tific a romneasc a menit a s a deschid a drum cercet arii originale. Ilie Popa [6, pp.492,493] 5

Pe lng a nv a t amntul matematic secundar si superior, o contribu tie important a la dezvoltarea stiin tei matematice la noi n tar a si a interesului fa ta de aceast a stiin t a au adus-o, n perioada aceasta de preg atire, cele dou a reviste care au ap arut la Ia si si Bucure sti: "Recrea tii stiin tifice", care a durat din 1883 pn a n 1888, si "Gazeta matematic a", nfiin tat a la 15 septembrie 1895. George S t. Andonie [8, p.236] n tara noastr a, publica tiile periodice destinate propag arii gustului si competen tei pentru propunerea si rezolvarea de probleme matematice aniverseaz a un secol de existen t a. n 1883 a ap arut la Ia si, prin dragostea si devotamentul unui grup de intelectuali de seam a, profesori universitari sau secundari, ingineri, fizicieni, medici, revista "Recrea tii S tiin tifice", c areia Universitatea din Ia si si Societatea de S tiin te Matematice din R. S. Romnia i-au aniversat centenarul apari tiei la sfr situl anului 1983. Nicolae Teodorescu [9, p.7] Revista si-a ntrerupt activitatea pe nea steptate. Ea a reu sit s a trezeasc a un viu interes pentru matematici n rndul elevilor si studen tiolor. Mare parte din coresponden tii forma ti de "Recrea tii stiin tifice" au devenit profesori si ingineri cu o serioas a preg atire n matematicile elementare. Nicolae Mih aileanu [11, p.177] Bibliografie 1. Recrea tii stiin tice (1883 - 1888) - colec tia revistei. 2. *** - Introducere, Gazeta matematic a, an.I, nr.1, septembrie 1895, (Mihail Roco este autorul acestei "Introduceri" - sarcin a ncredin tat a de redac tia G.M.). i redactarea Gazetei Matematice, articol 3. I. Ionescu - Constituirea, administrarea s ap arut n volumul Gazeta Matematic a, 1895-1935. Istoric - nv at aminte (volum jubiliar), Biblioteca Gazetei Matematice, vol. XI, Bucure sti, 1935. teica - Rolul Gazetei Matematice n dezvoltarea s tiin 4. Gh. Ti tei matematice n Romnia, ibidem. 5. S arb atorirea celor 40 ani ai "Gazetei matematice". Cuvntarea D-lui profesor Gr. G. Stratilescu, G.M. XLI (1936), 361-374. tiin 6. I. Popa - Recrea tii s tifice - precursoare a Gazetei Matematice, G.M.F., seria A, 9/1955, 492-493. ii 7. I. Popa - Dezvoltarea matematicii, ap arut n Contribu tii la istoria dezvolt arii Universit at din Ia si, vol. II, Bucure sti, 1960. t. Andonie - Istoria matematicii n Romnia, vol. I, Ed. S tiin 8. G. S tic a, Bucure sti, 1965. 9. N. Teodorescu s.a. - Probleme din Gazeta Matematic a, Ed. Tehnic a, Bucure sti, 1984 (citat din Prefa t a, semnat a de acad. N. Teodorescu, pre sedintele S.S.M.R.). 10. Gh. Banta s - O pagin a din istoria matematicii romne sti: centenarul revistei "Recrea tii tiin s tice", Probleme de istoria si lozoa stiin tei, vol. X, Acad. R.S.R., liala Ia si, 1984. 11. N. Mih aileanu - Revistele de matematici elementare din Romnia (pn a la 1848), Ed. Gil, Zal au, 1995. 6

Cteva curbe celebre si importante 1. Cisoida lui Diocles1


Se d a un cerc, pe care se ia un punct A; fie AB y diametrul ce trece prin acest punct si T T 0 tangenta n B ; prin A se duce o secant a care taie cercul n H Q si tangenta n G; pe secant a se ia, cu ncepere de la punctul A, o lungime AM egal a cu HG - por tiM unea de secant a dintre cerc si tangent a - ; locul geometric al punctelor M este Cisoida lui Diocles. A P S a nsemn am prin R raza cercului; originea de coordonate s a fie A; direc tia diametrului AB s a fie luat a ca ax a de x si perpendiculara n A pe acest diametru s a fie axa de y . Fie M un punct al Cisoidei, ale c arui coordonate snt x = AP si y = M P . Prin ipotez a avem AM = HG, de unde rezult a AP = DB . n triunghiul dreptunghiu AHB , avem apoi triunghiurile asemenea AM P si AHD dau HD AD HD 2R x = sau = . MP AP y x Eliminnd HD ntre rela tiunile (1) si (2), avem (2R x) y 2 = x3 sau x3 + xy 2Ry 2 = 0. HD2 = AD DB = x (2R x) ;
T H G

L x K D B

(1)

(2)

(3)

Aceasta este ecva tia Cisoidei. Curba este simetric a n privirea axei de x, c aci la fiecare valoare dat a lui x, corespunde pentru y dou a valori egale si de semne contrare. Ea se compune din dou a ramuri indefinite, egale ntre ele, situate de o parte si de alta a axei de x. Originea A este un punct de napoiere de specia nt aia. n adev ar, dac a ne raport am la teoria punctelor multiple, stim c a coordonatele unui asemenea punct, satisfac ecva tiei curbei si nt ailor derivate par tiale; avem coordonatele punctului A snt x = 0 si y = 0, care substituite n aceste rela tiuni dau f (0, 0) = 0,
1

f (x, y ) = x3 + xy 2 2Ry 2 ,

0 fx = 3x2 + y 2 ,

0 fy = 2xy 4Ry ;

0 = 0, f0

0 f0 = 0,

tiin Articol preluat din "Recrea tii S tice", an II (1884), nr. 1, 19-23. Ciclul "Cteva curbe celebre si importante" cuprinde 9 lec tii prezente n numerele din anii II si III n care sunt expuse principalele curbe plane: Cisoida lui Diocles (care deschide ciclul), concoida lui Nicomede, melcul lui Pascal , strofoida, ovalele lui Cassini, cicloide, epicicloide, conice si spirale. S-au p astrat termenii de matematic a din textul original, dar s-au f acut modific ari n privin ta lexicului si ortografiei (astfeliu - astfel, valor - valori, dau - dau, adec a - adic a, sa - s-a etc).

ceea ce nsemneaz a c a originea este un punct duplu; apoi dac a form am ecva tia tangentelor n acest punct g asim y 2 = 0; adic a tangentele n acest punct se confund a cu axa de x. A sadar, originea este un punct duplu de napoiere si de nt aia specie, c aci curba este de o parte si de alta a tangentei. Cisoida admite o asimptot a paralel a cu axa de y , c aci stim c a aceste asimptote se cap at a egalnd cu zero coeficien tii celei mai nalte puteri a lui y , ceea ce d a 2R x = 0, de unde x = 2R, adic a tangenta la cercul T T 0 este asimptot a Cisoidei. Asimptote neparalele cu axele nu snt. Cercul dat este numit cercul director al Cisoidei. T Newton a dat Cisoidei urm atoarea descrip tie mecanic a. K H Fie un punct fix A si o dreapt a fix a T T 0; N din A se duce perpendiculara AD pe dreapta fix a; apoi se imagineaz a un unghi drept care I se mi sc a astfel c a una din laturile lui trece prin M punctul A, iar extremitatea celeilalte laturi, - A O R D L luat a egal a cu AD -, se razim a pe dreapta fix a; dac a G este vrful unghiului drept si H G extremitatea laturii a doua, punctul M din mijlocul laturii GH descrie Cisoida. T S a lu am mijlocul O al dreptei AD si din punctul D ca centru cu DO ca raz a s a descriem un cerc; s a unim AH , apoi OM , M ind mijlocul laturii GH . Vom demonstra mai nti c a dreptele AH si OM snt paralele ntre ele. n adev ar, triunghiurile dreptunghe AGH si ADH snt egale c aci AH = AH , apoi GH = AD conform enunciului. De aici rezult a AG = DH si fiindc a unghiurile ARG si HRD snt egale ca opuse la vrf, apoi rezult a c a si triunghiurile dreptunghe AGR si HDR snt egale ntre ele. Din egalitatea acestor dou a din urm a triunghiuri rezult a GR = RD si fiindc a OD = = GM , apoi mai rezult a OR = RM si AO = HM si prin urmare dreptele AH si OM snt paralele. Fiindc a punctul O este mijlocul dreptei AO, din paralelismul acestor drepte, urmeaz a c a OM taie pe HD ntr-un punct care-i la mijlocul dreptei HD. S a ar at am acum c a triunghiurile HIM si DIM snt egale; n adev ar, avem mai nti HM = DO = DN ; apoi ungh.N ID = ungh.HIM ; pe urm a succesiv ungh.DN I = = ungh.M OR = ungh.HAD = ungh.AHG = ungh.HM I , a sadar IM = IN si fiindc a OI = IK , apoi rezult a IM = IN si prin urmare OM = N K . A sadar, n mi scarea unghiului drept AGH , punctul M descrie o Cisoid a al c arei cerc director este acel descris din D ca centru cu DO ca raz a. Cisoida a fost imaginat a de Diocles (500 a. Ch.) pentru a rezolvi problema a dou a medii propor tionale1 . Iat a cum se rezolve ste aceast a problem a.
1

Sublinierele din aceasta si n textul original. fraza nu apar

Ecva tia (3) fiind rezolvit a n privirea lui y d a: x2 . (4) y=p x (2R x) Lu am semnul + naintea radicalului fiindc a consider am ramura de deasupra axei de x. S a ns amn am prin z ordonata punctului de pe cerc al c ariu abscis a este x; avem 2 z = x (2R x) . (5) Comparnd ecva tiile (4) si (5) avem raporturile 2R x z x = = . (6) z x y Fie a si b liniile ntre care se cere a se afla dou a medii propor tionale. b a S a nmul tim terminii raporturilor (6) prin , ceea ce d y (2R x) b bz bx y y y = = . (7) bz bx b y y S a lu am pe Cisoid a un punct astfel ca s a avem (2R x) b = a, (8) y ceea ce revine a considera punctul comun Cisoidei, - reprezentat a prin ecva tia (3) -, si dreptei - reprezentat a prin ecva tia (8) -; atunci raporturile (7) se pot scrie bz bx a y y = = . bz bx b y y bx a bz = si = , vom avea = = . Aceste trei raporturi ne dau dou a S a punem y y b bz bx ecva tii, din care vom scoate pe si . Astfel cantit a tile si vor fi cunoscute si y y aceste vor fi cele dou a medii propor tionale ntre a si b. Dac a Cisoida este construit a, pentru a g asi punctul de pe ea cu ajutorul c aruia putem rezolvi problema, trebuie s a construim dreapta (8), care este o dreapt a ce trece prin punctul B , pentru aceea lu am o lungime BK = a, r adic am perpendiculara KL = b, dreapta BL este dreapta (8). Punctul M n care aceast a dreapt a taie Cisoida este punctul c autat, si avem AP = x, M P = y, QP = z ; QP AP prin urmare cele dou a medii propor tionale ntre a si b vor fi b si b . M P M P (Va urma)

C. CLIMESCU
9

Scrierea numerelor cu litere chirilice2


Toate popoarele, afar a de vechii chinezi si de un trib pu tin cunoscut de care vorbe ste Aristot3 au adoptat sistema de numera tie zecimal a, n care numerele snt mp ar tite n perioade de cte zece unit a ti. Baza, n aceast a sistem a de numera tie, este zece, adic a zece unit a ti de un ordin oarecare trebuiesc ca s a formeze o unitate de ordinul imediat mai nalt. Scrierea oric arui numer se face ast azi numai cu zece semne, numite cifre, dintre care nou a reprezint a pe cele dinti nou a numere ntregi, si care snt : 1 unu, 2 doi, 3 trei, 4 patru 5 cinci, 6 sese, 7 septe, 8 opt, 9 nou a.

Semnul al zecelea este 0 sau zero, prin care se arat a lipsa de unit a ti de un ordin oarecare. S-a admis c a orice cifr a pus a la stnga unei alte cifre reprezint a unit a ti de zece ori mai mari dect aceasta din urm a, adic a unit a ti de ordinul imediat superior. Astfel nct, dnd acestor semne, diferite locuri se poate exprima, ntr-un chip nu se poate mai u sor, orice numer, fie orict de mare numerul unit a tilor din care el se compune. Semnele acestea le-am mprumutat, pe la mijlocul secolului al X, de la arabi care si dn sii pare c a le-au luat de la indieni, adev ara tii ntemeietori ai stiin tei ctimilor. n vechime, era obeciul de a se reprezenta numerele cu litere. Grecii nsemnau numerele cu literele din alfabetul lor; diferite ordine de unit a ti: unimi, zeci, sute, . . . , se deosebeau unele de altele prin accente puse deasupra literelor. Romanii, pentru scrierea numerelor, ntrebuin tau septe semne principale: I unu, V cinci, X zece, L cincizeci, C o sut a, D cinci sute, M o mie.

Cu ajutorul acestor litere, si prin chipul combin arei lor f acut a dup a ni ste reguli bine statornicite, se pot scrie numere din o mie si orice numer de sute, zeci si unimi. n c ar tile romne sti vechi, n cronici precum si n c ar tile religioase, pe nscrip tii, hrisoave, se g asesc numere scrise cu litere chirilice. n reprezentarea chirilic a a numerelor, unimilor, zecile si sutele se nseamn a, dup a cum se vede n tabela de mai la vale, fiecare cu cte un semn deosebit, o liter a a alfabetului chirilic avnd deasupra lor un semn, poate spre a ar ata c a litera este luat a drept numer. Astfel, pentru a scrie un numer oarecare, mai mic dect o mie, n nota tia chirilic a, trebuiesc 27 semne. Acelea si litere, avnd semnul n stnga lor si pu tin mai jos, servesc a reprezenta unit a tile din clasa miilor, adic a unimile de mii, zecile de mii si sutele de mii. Alte rnduri de semne conven tionale, puse al aturi cu acelea si litere, ar fi putut servi spre a reprezenta unit a tile din celelalte clase mai nalte, clasa milioanelor, a miliardelor,
2 3

Articol preluat din "Recrea tii S tiin tifice", an I (1883), nr. 3, 57-60. Bossut, Istoria general a a matematicilor.

10

11

. . . ; totu si, numerele exprimnd milioanele precum si ordine mai nalte urm atoare se scriu n cuvinte. Caracteristic este modul cum se potrive ste de bine scrierea numerelor n nota tia chirilic a cu numirea lor. Se stie c a, numerele cuprinse ntre zece si dou azeci prezint a o excep tie, n ceea ce prive ste numirea lor, de la regula general a ntemeiat a pe modul form arii lor. Pe cnd la numerele de la dou azeci nainte, compuse din zeci si unimi, se enun t a mai nti zecile si apoi unimile, dac a snt , ca de exemplu n: patruzeci si patru, septezeci si nou a etc., la numerele cuprinse ntre zece si dou azeci, se enun t a mai nti unimile si apoi zecile: treisprezece, optsprezece etc. n reprezentarea chirilic a a numerelor, se urmeaz a ntocmai dup a cum se enun t a: unit a tile se scriu nainte sau dup a zeci, dup a cum este si numirea numerului. Este lesne de v azut c a numerele exprimate cu litere se pot supune calculului ca si numerele scrise cu cifre arabe; singura deosebire st a ntr-aceea c a calculul va fi, n cazul nti, cu atta mai lung si mai greu, cu ct numerul va fi mai mare.

I. M. MELIK

12

Acad. Radu Miron la a 75-a aniversare


Academician profesor doctor docent Radu Miron.....ce se mai poate oare spune n cteva rnduri!? S i totu si...profesorul Radu Miron este preferatul multor genera tii de studen ti ai Faculta tii de Matematica si. Prezen ta din Ia carismatica n amteatru, reu se ste sa si cu o dic tie captiveze n mod natural auditoriul. Cu mult calm remarcabila , cele mai ntunecate capitole ale matematicii se limpezesc, iar fereastra opaca ta si pentru orbul din spatele oglinzii capa ta o transparen de cristal chiar disimulat n student. S i totu si...profesorul R. Miron arunca peste noi, ca un prestidigitator, o plasa imensa tine peste 250 de lucra stiin tifice, note care este opera sa, opera care con ri bibliografice, mongrafii etc. Tra ind peste 50 de ani n atmosfera Seminarului Matematic Al. Myller, un adeva tie stiin tica scoala rat laborator de crea n cmpul abstract al matematicii, o academica tinuta tii de de nalta , profesorul R. Miron este un continuator al acelei genera aur a matematicienilor romni. Men tiona m aici numele lui Alexandru Myller, fondatorul Seminarului Matematic, Octav Mayer, Gheorghe Vra nceanu, Grigore Moisil, Mendel Haimovici, Adolf Haimovici, Dimitrie Mangeron, Constantin Climescu, Ilie Popa, Gheorghe Gheorghiev s. a. Remarcat de profesori nca tie este numit asistent n din primii ani de studen anul II, iar dupa tii este ncadrat ca cerceta absolvirea faculta tor la Institutul de Matematica si a Academiei. In 1957 si sus tine teza de doctorat cu ti, Filiala Ia tlul Problema geometriza rii sistemelor mecanice neolonome , sub conducerea academicianului Mendel Haimovici, lucrare publicata si n ntregime n revista Studii Cerceta ri Matematice. Parcurgnd ntreaga ierarhie universitara este numit n 1969 profesor la Catedra de Geometrie a Faculta tii de Matematica din Universitatea Al. I. Cuza, iar n 1973 director al Institutului de Matematica tii de Matematica . Decan al Faculta ntre anii 1972-1976, conduca sef al Catedrei de Geometrie, membru tor de doctorat din 1972, al Consiliului Profesoral si al Senatului, profesorul R. Miron a desfa surat o bogata activitate didactica si educativa . Activitatea de cercetare stiin tifica a profesorului R. Miron este bine cunoscuta n lumea ntreaga si recunoscuta tia sa importanta si originala . Este stabilita contribu la dezvoltarea Geometriei Diferen tiale moderne si a aplica tiilor ei n Fizica Teoretica . Profesorul R. Miron a creat si dezvoltat n matematica noi ramuri ca: geometria congura tiilor Myller, teoria invarianta a spa t iilor Finsler, spa tii Lagrange, spa tii Lagrange generalizate, teoria subspa tiilor Lagrange, geometria spa tiilor Lagrange de ordin superior, spa tii Hamilton, spa tii Hamilton generalizate, teoria geometrica a spa tiilor brate, teoria lagrangeana a relativit a t ii s i electromagnetismului, ecua tii Einstein s i Maxwell. A rezolvat multe probleme deschise ca: prelungirea structurilor riemanniene, nsleriene, lagrangeene, spa tii Finsler de ordin superior etc. Cerceta rile ini t iate s i dezvoltate de profesorul R.Miron au avut un mare impact asupra speciali stilor n geometrie si nu numai. Profesorul Makoto Matsumoto de la Universitatea din Kyoto, n cartea sa Fundamentele Geometriei Finsler si Spa tii 13

Finsler speciale se refera si ecua tii fundamentale ale la conceptul de reper Miron reperului Miron. Profesorul Masao Hashiguchi atribuie numele de "spa tiu Miron unui caz remarcabil de spa tii Hamilton, introduse pentru prima data de profesorul R. Miron, iar G. S. Asanov, de la Universitatea din Moscova, aplica modelele Lagrange ale profesorului R. Miron n cosmologie si ob tine cea mai buna tie teoretica devia a periheliului planetelor Marte, Venus si Mercur. Bazndu-se pe teoria profesorului R. Miron, G. Beil (S.U.A.) a ob tinut o buna si P. L. Antonelli (Canada) teorie gauge a aplicat-o n biologie. Profesorul R. Miron a creat n Romnia o scoala de matematica de nalt nivel, care a cooperat pe parcursul multor ani cu oameni de stiin ta din Japonia, Rusia, S.U.A., Germania, Italia, Anglia, Canada, Ungaria, Egipt etc. Un numa r mare de doctoranzi din tara s i str a in a tate (Japonia, Italia, Ungaria, Vietnam) au ob tinut titlul de doctor n matematica sub conducerea d-sale. Avnd un renume deosebit n lumea Matematicii, profesorul R. Miron a fost invitat ca visiting professor la prestigioase universita ti ca: Universitatea din Tsukuba (Japonia, 1988, 1990, 1992 ), Bary (Italia, 1987 ), Freiburg s i Mnchen (Germania, 1975, 1990 ), Edmonton (Canada, 1992 ). Profesorul R. Miron a publicat o parte din lucra rile sale n colaborare cu geometri japonezi: M. Matsumoto, M. Hashiguchi, Y. Ichijio, S. Kikuchi, S. Watanabe, S. Ikeda sau cu membrii ai Seminarului Na tional de Geometrie Finsler si Lagrange, ini tiat de dnsul n 1980 la Universitatea din Bra sov. Este primul pre sedinte al Societa tii Balcanice a Geometrilor constituita la ini tiativa sa si a profesorului G. Tsagas de la Universitatea Aristotel din Thessaloniki (Grecia) n 1994-1995. Instituto per la Ricerca di Base din Italia i-a oferit profesorului R. Miron titlul de Full Professor in the Division of Mathematics of the I.R.B.. Profesorului R. Miron i s-a acordat Premiul Ministerului Educa tiei (1963), premiul Gh.Ti teica al Academiei (1968) s i a fost ales membru al Academiei Romne (1991). Profesorul R. Miron a scris n colaborare cu profesorul M. Anastasiei o carte de pionerat Geometria spa tiilor Lagrange: teorie s tii , publicata i aplica n 1994 de Kluwer Academic (S.U.A.) n prestigioasa serie Fundamental Theories of Physics. Mai amintim doua titluri importante de monografii care concentreaza ideile matematice ale profesorului R.Miron: Geometria spa tiilor Lagrange de ordin superior. Aplica tii n Mecanica si Geometria spa tiilor i Fizica s (Kluwer Academic, 1997) Finsler de ordin superior (Hadronic Press, U.S.A., 1998). Impreuna cu profesorul P. L. Antonelli de la Universitatea din Alberta, profesorul R. Miron este editorul ca tii Geometrie Finsler s tii n Fizica i Lagrange.Aplica i r s Biologie , publicata deasemeni de Kluwer Academic n 1996. Chiar daca pe 3 octombrie 2002, profesorul Radu Miron a mplinit frumoasa vrsta de 75 de ani, sunt convins ca "surprizele matematice" vor continua sa ne uimeasca . Prof. dr. Alexandru NEAGU

14

Numere prime din progresii aritmetice


Petru Minu t1
Un numa ste numa ti divizori r natural p, p > 1, se nume nu are al r prim daca nafara si p. de 1 Lema . Un numa r natural n, n > 1, are un divizor prim. Demonstra tie. Fie M mul timea tuturor numerelor naturale care sunt divizori ai lui n diferi ti de 1. M 6= , deoarece n M . n M exista un numa r care este cel mai mic, p. Ara ta m, prin reducere la absurd, ca p este prim. Presupunem ca p este compus: p = ab, 1 < a < p. Din a | p si p | n rezulta ca a | n. Am ga sit un divizor al lui n mai mic ca p ceea ce contrazice alegerea lui p. Teorema 1. n mul timea numerelor naturale exista o infinitate de numere prime. Demonstra tie. Exista ti numere prime. De exemplu 2, care nu poate avea al divizori n afara si 2. Folosim metoda reducerii la absurd. Presupunem ca de 1 exista o mul time finita de numere prime n N, P = {p1 , p2 , . . . , pk }. Considera m numa rul ajuta un numa r prim p, p | N . Din tor N = p1 p2 . . . pk + 1. Deoarece N > 1, exista p P rezulta ta ca p | p1 p2 . . . pk . Daca doua numere sunt multipli de p, atunci diferen lor este multiplu de p. Rezulta si contrazicem defini tia ca p | 1, ceea ce implica p=1 numa rului prim. Observa tie. Teorema 1 o ga tata si demonstrata sim enun pentru prima oara n opera lui Euclid "Elemente" (sec. III . Ch.) si este cunoscuta sub denumirea de teorema lui Euclid. Se cunosc numeroase demonstra tii ale acestei teoreme. Singurul numa siruri: r prim par este 2. Aranja m numerele impare n doua 3, 7, 11, 15, . . . , 4k 1, . . . 1, 5, 9, 13, . . . , 4k + 1, . . . (1) (2)

Constata siruri (progresii aritmetice), mergnd pna m ca n aceste la termeni de rang tot mai mare, ga si alte progresii sim noi termeni care sunt numere prime. Daca lua m aritmetice, de exemplu: 3, 13, 23, 33, 43, 53, 63, 73, 83, . . . (3) 2, 7, 12, 17, 22, 27, 32, 37, 42, 47, . . . (4) constata si lucru. Este u sor de demonstrat ca m acela n progresia (1) exista o infinitate de numere prime. Teorema 2. Exista o infinitate de numere prime de forma p = 4k 1, k N. Demonstra tie. Proceda ta m prin reducere la absurd. Am pus deja n eviden cteva numere prime de aceasta sirul (1)). Presupunem ca forma ( exista un numa r finit de numere prime de acest fel: p1 , p2 , . . . , pn . Construim numa rul ajuta tor N = = 4p1 p2 . . . pn 1. Deoarece N > 1, exista p prim, p | N . Orice numa r prim p diferit de 2 este de forma p = 4k 1 sau p = 4k + 1. Daca ti divizorii primi ai lui N sunt to de forma p = 4k + 1, numa si 4 | N + 1, rul N este de forma N = 4h + 1, deci 4 | N 1 ceea ce implica tie! Exista 4 | 2. Contradic divizori primi ai lui N de forma 4k 1. Fie
1

Prof. dr., Univ. "D. Cantemir", Tg. Mure s

15

p un asemenea divizor. Rezulta si cum ca p {p1 , p2 , . . . , pn }, deci p | 4p1 p2 . . . pn p | N rezulta tie! Presupunerea ca p = 1. Contradic exista un numa r finit de numere prime de forma p = 4k 1 nu poate fi adeva rata . Teorema 2 se generalizeaza dupa cum urmeaza : Teorema 3. Pentru orice numa r natural n, n 6= 0, exista o infinitate de numere prime p de forma p = nk 1, k N. Demonstra tie. Pentru n = 1, {nk 1 | k N} = N {1} si arma tia teoremei este adeva rata (teorema lui Euclid). Pentru n = 2, {nk 1 | k N} = = {1, 1, 3, 5, 7, . . . } si arma tia teoremei este adeva rata (exista o innitate de numere prime impare). Pentru demonstra tia teoremei n cazul n > 2 vom folosi lema urma toare: Lema r natural n , n > 1 , avem: . Pentru orice numa Y (5) 1r<n r 1 (mod n) .
(r,n)=1

proprietatea ca r2 1 (mod n) avem r (n r) 1 (mod n). Rezulta ca membrul k nti al congruen tei (5) este congruent cu (1) , unde 2k este numa rul acelor r cu proprietatea r2 1 (mod n) (r si n r au ambii aceasta proprietate). Lema este demonstrata . Revenim la demonstra tia teoremei n cazul n > 2. Vom ara ta prin reducere la absurd, ca exista o infiniate de numere prime p de forma p = nak 1, k N, unde a este produsul numerelor naturale mai mici ca n si prime cu n luat cu semnul + sau dupa cum produsul acestor numere este congruent cu +1 sau 1 modulo n. Presupunem ca exista un numa r finit de numere p de forma p = nak 1: p1 , p2 , . . . , ps . Considera m numa rul ajuta tor N = nap1 p2 . . . ps 1. N > 1 deoarece chiar n cazul s = 0, N = na 1 > n 1 1. Exista p prim, p | N ; p este de forma p = nau + r, (r, N a) = 1. Din p = N au + r nu + r (mod n) rezulta si nu + r dau acela si ca p rest la mpa tirea cu n. Rezulta si nu putem avea 1 < r < n 1 deoarece ar r ca r<n rezulta ca si contrazicem faptul ca r | n sau r | a, deci r | p p este prim. Prin urmare, p este de forma p = nau 1. Daca ti divizorii lui N ar fi de forma p = nu + 1, N to ar fi si el de aceasta forma . Prin urmare, exista un divizor prim p al lui N de forma p = nau 1. Rezulta c a p { p , p , . . . , p si cum p | N am avea } , p | nap1 p2 . . . ps 1 2 s p | 1, deci p = 1 si contrazicem defini tia numa rului prim. Teorema 2 se ob tine din Teoremei 3 lund n = 4. Din Teorema 3 rezulta ca exista o infinitate de numere prime p de forma p = 6k 1, k N sau p = 8k 1, k N s.a.m.d. Pentru a ara tine o infinitate de numere prime vom folosi ta ca progresia (2) con urma toarea 2 Lema . Oricare ar fi numa rul natural n, n > 1, numa rul (n!) + 1 are divizori primi s stia sunt de forma p = 4k + 1. i ace 16

Demonstra tie. Pentru n = 2 congruen ta este evidenta si r fixat, . Pentru n > 2 1 r < n, (r, n) = 1, stim ca tie unica ta rx 1 (mod n). exista solu pentru congruen Deci, exista un singur r0 , 1 r0 < n, (r0 , n) = 1 astfel nct rr0 1 (mod n). n 0 ti r pentru membrul nti al congruen tei (5) nlocuim Q produsele rr cu 1 pentru to Q 0 care r 6= r. Rezulta r. Observa m ca pentru r cu ca (r,n)=1 r (r,n)=1
r2 1(mod n)

Demonstra tie. Pentru n > 1, numa rul (n!) + 1 este impar, mai mare ca 1. 2 Exista p | (n!) + 1, p 6= 2. Deci p este de forma p = 4k + 1 sau p = 4k + 3. Daca p 2 2(2k+1) p1 +1, adica +1 este de forma p = 4k +3, din p | (n!) +1 rezulta p | (n!) p | (n!) p si apoi p | (n!)p + n!. Conform cu mica teorema a lui Fermat p | (n!) n!. Rezulta p | 2n!, deci p n, p | n! ceea ce implica tie! p | 1, contradic Teorema 4. Exista o infinitate de numere prime p de forma p = 4k + 1, k N. Demonstra tie. Folosim din nou metoda lui Euclid. Presupunem ca exista un si considera numa m r finit de numere prime de forma 4k + 1: p1 = 5 < p2 < < ps 2 numa rul ajuta tor N = [(p1 p2 . . . ps )!] + 1. Acesta admite, conform lemei, un divizor prim p de forma p = 4k + 1 si ajungem din nou la contradic tia p | 1. Pentru generalizarea Teoremei 4 avem nevoie de cteva chestiuni prega titoare. Fie 2h 2h k i k un numa tia x 1 = 0 are ra + r natural, k 1. Ecua da cinile xh = e k = cos k Q 2h 2h i sin , h = 0, 1, . . . k 1. Considera m polinomul Fn (x) = (h,n)=1 x e k i , k unde produsul se face dupa numerele h {0, 1, . . . , n 1} care sunt prime cu n. Gradul lui Fn (x) este (n) ( (n) = numa si rul numerelor naturale mai mici ca n prime cu n, este cunoscut a sub numele de func t ia indicatoare a lui Euler ). Observ a m Q ca xk 1 = n | k Fn (x) (produsul se face dupa divizorii pozitivi ai lui k ). Fie k x 1 = Fk (x) Gk (x), unde Gk (x) este cel mai mic multiplu comun al polinoamelor xn 1, n | k , n < k, avnd coeficientul termenului de grad cel mai nalt egal cu ti ntregi, atunci si Fk (x) este un 1. Deoarece Gk (x) este un polinom cu coeficien polinom cu coeficien ti ntregi. Observa m ca pentru orice numa r ntreg x, x 6= 1, avem Fk (x) Gk (x) 6= 0. Lema 1. Fie n un divizor propriu al lui k ( n 6= 1, n 6= k). Pentru orice numa r k 1 x ntreg x, x 6= 1, avem: xn 1, n | k. x 1 n Demonstra tie. Nota m k = nd, x 1 = y . Vom avea:
d

(y + 1) 1 xk 1 1 d2 y + + d d (mod y ) . = y d1 + Cd = xn 1 y xk 1 Daca , din | y rezulta si, cum d | k , rezulta = xn 1, n |d | k. x 1 Lema 2. Fie x Z, x 6= 1. Orice divizor prim, comun lui Fk (x) s i Gk (x) este un divizor al lui k . Demonstra tie. Fie p prim, p | Fk (x), p | Gk (x). Din p | Gk (x ) rezulta ca exista Q n N , n | k, n < k , astfel nct p | Fn (x) (deoarece Gk (x) = n | k, k<x Fn (x) si daca tin unul un numa r prim divide un produs atunci el divide cel pu dintre factori). xk 1 xk 1 n n 1 . Conform Lemei , n si p | Fk (x) rezulta Din p | x 1 si p | x p| n x 1 x 1 1, p | k . Teorema 5. Pentru orice numa r natural k, k 1, exista o infinitate de numere prime de forma p = nk + 1, n N. Demonstra tie. Pentru k = 1 enun tul teoremei este adeva rat (teorema lui Euclid ). Pentru k > 1, ara t a m mai nti c a exist a numere prime de forma p = nk + 1. 17

Pentru x = ky , y Z vom avea: Fk (x) Gk (x) = xk 1 1 (mod k). Deoarece ecua tiile Fk (x) = 1 au un numa r finit de ra da cini, putem alege y astfel nct Fk (x) 6= 1. Exista numere prime p care sunt divizori ai lui Fk (x). Deoarece p - k (p | k p | x p |1), rezulta si deci p - xn 1, (conform Lemei 2) ca p - Gk (x) n oricare ar fi numa si rul natural n, n | k , n < k . Deci x 6 1 (mod n), n | k, n < k xk 1 (mod n). Fie n = (k, p 1). Exista si t astfel nct doua numere ntregi s s p1 t x 1 (mod p). Nu putem avea n < k, n = sk + t (p 1). Rezulta ca xn = xk deci n = k . Conform cu mica teorema ca a lui Fermat xp1 1 (mod p). Rezulta p 1 este multiplu de k . ntr-adeva r, daca este cea mai mica putere ntreaga , strict pozitiva a lui x astfel nct x 1 (mod p), atunci xa 1 (mod p) | a. Daca b a a a = b, atunci x = x 1 (mod p). Daca x 1 (mod p), a = b + r, 0 r < , nu putem avea > 0 deoarece xa xr 1 (mod p) si contrazicem alegerea lui . Deci, p 1 = nk , adica p = nk + 1. Fie p1 un numa r prim de forma p1 = n1 k + 1. Lua m k1 = p1 k . Conform primei pa r t i a demonstra t iei exist a un num a r prim p de forma p2 = np1 k + 1, adica pentru 2 orice numa r prim p de forma p = nk + 1 exist a un num a r prim p de aceea s i forma , 1 1 2 p2 > p1 . Rezulta c a exist a o infinitate de numere prime p de forma p = nk + 1 , n N. Enun tul cel mai general, care cuprinde drept cazuri particulare toate teoremele prezentate, l constitue teorema urma toare, cunoscuta n literatura matematica sub denumirea de teorema lui Dirichlet. Teorema 6. Oricare ar fi numerele l Z s i k N , (l, k ) = 1, progresia aritmetica l, l + k, l + 2k, . . . l + nk, . . . con tine o infinitate de numere prime. Condi tia (l, k ) = 1 este necesara ti termenii progresiei . Daca (l, k ) = d > 1, to sunt multipli de d. Demonstra tia Teoremei 6, n cazul general, nu poate fi fa cuta prin metode ale matematicii elementare. Problema numa rului de numere prime dintr-o progresie aritmetica a fost pusa pentru prima oara n 1775 de Leonard Euler n cazul particular l = 1. n cartea sa "Thorie des nombres" A. M. Legendre a dat o demonstra tie Teoremei 6 bazata pe o ipoteza tie a teoremei a , care ulterior s-a dovedit a fi falsa . Prima demonstra fost data n 1837 de Lejeune P. G. Dirichlet care a creat un aparat analitic special (seriile Dirichlet ). Demonstra tia lui Dirichlet este considerata stere al actul de na teoriei analitice a numerelor. Bibliografie 1. I. Creanga t, Gh. Opai t, C. Reischer - Introducere n teoria , C. Cazacu, P. Minu numerelor, Editura didactica si pedagogica sti, 1965. , Bucure 2. Hua Loo Keng - Introduction to Number Theory, Springer Verlag, Berlin, Heidelberg, 1982. 3. P. Minu t - Teoria numerelor. Capitole introductive, Editura "Crengu ta Glda u", Ia si, 1997. 4. C. P. Popovici - Teoria numerelor, Ed. didactica si pedagogica sti, 1973. , Bucure 5. W. Sierpinski - Ce s stiin tifica tim s i ce nu s tim despre numerele prime, Editura , Bucure sti, 1966. 18

Recrea tie matematica si nu numai


Horea BANEA1
Este cunoscuta urma toarea problema - joc: Sa se descompuna poligonul din figura ala tinute prin realipire sa turata prin doua linii drepte astfel nct din poligoanele ob se realizeze un pa tia este indicata n figura . trat. Solu

Sugerat de aceasta, propunem urma toarea problema : Dintr-o bucata de carton, de forma poligonului de mai sus, printr-o ta ietura dupa o dreapta tinute se realizeaza i realipirea buca ilor ob s t diferite figuri de forma unor poligoane convexe. Sa tiile distincte. Sa se ga seasca toate situa se calculeze lungimile laturilor poligoanelor ob tinute. n lega tura cu enumerarea propusa facem urma toarele preciza ri: Situa tii distincte sunt cele n care T = ta si/sau P = poligonul ob tinut ietura difera ntre ele. Cazurile n care cu aceea si T ob tinndu-se componente simetrice, se poate realiza acela si P n diferite moduri, vor fi considerate doar variante echivalente ale aceleia si situa tii cu excep tia cazurilor n care alipirea aceleia si componente se face la alt segment al componentei de baza (=cea mai mare) care vor fi considerate ca situa tii distincte. Cnd T este variabila tinndu-se acela si tip de P dar cu dimensiuni variabile , ob depinznd de un parametru, se considera tie, dar cazurile particulare ca o singura situa ale parametrului care conduc la P cu anumite particularita ti se enumera distincte de cazul general. Nu se enumera tinute prin ntoarce, fiind socotite variante echivalente, figurile ob rea pe verso a ntregii figuri ob tinute ntr-un caz. Pentru a u sura urma rirea efectua rii ta ieturilor indica m gruparea lor n raport cu anumite puncte remarcabile prin care au fost duse: M : T116 ; N : T1722 ; P : T23 ; Q : T25,26 . Enumerarea lungimilor laturilor se face ncepnd cu cea superioara , n sens matematic. Justicarea calculelor, bazndu-se doar pe teorema lui Pitagora si pe asema narea triunghiurilor, se lasa pentru cititori. Trapez dreptunghic variabil: p 5, y, 4y 2 8y + 29, 2 y ; 0 < y < 1. Triunghi dreptunghic: 5, 29, 2.

Conf. dr., Univ. Transilvania, Bra sov

19

Patrulater inscriptibil:

19 4 3 29 2 29 , , , . 5 5 5 5

Pentagon: 3,

4 3 29 4 2 29 , , , . 5 5 5 5

Hexagon: p p 2x + 2 3, 4 x2 , , 2 x, x, 4 x2 ; x x 1, 84, x4 + x3 2, 75x2 4x 1 = 0. Obs. Nu se considera si P5 . Hexagon: p p 2x + 2 3, 4 x2 , , 2 x, 4 x2 , x; x x ca la P6 . Obs. Aceea si ca la P6 . 2 8 Trapez dreptunghic: 3, , 13, . 3 3

Trapez dreptunghic variabil: p 3 + x, 4x2 + 4x + 5, 2 x, 2; 0 < x < 2.

Trapez dreptunghic ortogonal: 4,

13, 1, 2.

x+2 Pentagon: 3, 2 1 x, , 2 1 x, 2; x x 0, 89 , 4x3 + x2 3x 1 = 0. Obs. Nu se considera si P5 . Trapez dreptunghic circumscriptibil: 5+ 5 5 5 51 , 3, , 2; x = . 2 2 2 Trapez dreptunghic: 7/2, 2 2, 3/2, 2. Obs. Realizat prin alipirea altor triunghiuri fa ta de P5 . Trapez dreptunghic: 3, 5, 2, 2.

20

Trapez dreptunghic variabil: p 3 x, 4x2 4x + 5, 2 + x, 2; 0 < x < 1/2. 5 5 Dreptunghi: , 2, , 2. 2 2 Obs. Are variante echivalente.

Trapez dreptunghic variabil: p x, 4x2 20x + 29, 5 x, 2; 0 < x < 5/2. Trapez dreptunghic circumscriptibil: 5 5 5+ 5 x= , 3, , 2. 2 2 Trapez dreptunghic ortogonal: 1, 13, 4, 2.

Triunghi dreptunghic:

29, 5, 2.

Trapez dreptunghic variabil: p 4y 2 8y + 29, y, 5, 2 y ; 0 < y < 1. Paralelogram: 5 5 10, , 10, . 3 3

Trapez isoscel:

2 8 10, , 10, . 3 3

Triunghi dreptunghic: 5, 2 5, 5.

Trapez: 4, 2 5, 1, 5.

21

Pentagon: 3,

1 1 , 2 5, , 5. 2 2

Pentagon: 4 3 9+2 3 3 3 6+2 3 , , , 2 3, . 3 3 3 3 Pentagon: 3 3 6+2 3 6 3 4 3 3, , , , . 3 3 3 3 Pentagon: 3, 2 2, 1, 2, 2. Obs. Cele doua si pot schimba triunghiuri locurile. Sunt si variante echivalente. 5 5 Dreptunghi: , 2, , 2. 2 2 5 5 , 5, , 5. 2 2 7 3 , 5, , 5. 2 2

Paralelogram:

Trapez isoscel:

5 5 5 3 5 Patrulater inscriptibil: , , , . 2 2 2 2

5 5 3 5 1 Pentagon: , , , , 2. 2 2 2 2

Hexagon inscriptibil cu axa de simetrie: 1, 2, 1, 3, 1, 2. Hexagon cu centru de simetrie: 2, 2, 1, 2, 2, 1.

22

Hexagon cu centru de simetrie: 1, 2, 2, 1, 2, 2.

Hexagon inscriptibil cu axa de simetrie: 1, 2, 1, 2, 1, 3. 5, 1, 2.

Pentagon: 3, 1,

Pentagon: 3, 2 2, 2, 3 2, 2,. Obs. Cele doua si pot schimba truinghiuri locurile. Sunt si variante echivalente. Pentagon cu axa de simetrie: 2, Obs. Are variante echivalente. 2, 2, 2, 2.

Pentagon variabil cu axa de simetrie: p p 3x, 4x2 4x + 2, 4x2 4x + 2, 3x, 2; 1/2 < x < 1. Hexagon cu centru de simetrie: 2, 1, 2, 2, 1, 2.

Obs. Are variante echivalente. Hexagon inscriptibil cu axa de simetrie: 3, 1, 2, 1, 2, 1. Obs. Are variante echivalente. Dreptunghi: 5, 1, 5, 1. Obs. Are variante echivalente. Dreptunghi: 5, 1, 5, 1. Obs. Are variante echivalente. * * * Aceasta lista lunga de poligoane (adresa m cititorilor provocarea de a mai ga si altele remarcabile) permite utilizarea ei n diferite moduri adaptate la nivelul cuno stin telor celor ca si la scopurile urma rora li se adreseaza rite de propuna tor. De exemplu, propunem urma toarele: 1) Sa tiile care dau triunghiuri sau patrulatere (eventual se ga seasca doar situa 23

particulare: trapeze s. a.) 2) n ce situa tie se ob tine figura cu perimetrul maxim sau minim? 3) Ce tipuri distincte de poligoane se pot ob tine? (triunghi dreptunghic, dreptunghi, paralelogram, trapez, trapez isoscel, trapez dreptunghic, trapez circumscriptibil, trapez ortodiagonal, patrulater inscriptibil, pentagon, pentagon cu axa de simetrie, hexagon, hexagon cu centru de simetrie, hexagon cu axa de simetrie; s-au luat n considerare doar particularita tile clasice nelund n considerare particularita ti ca: pentagon cu doua unghiuri drepte s . a.). 4) Care ta ietura da cel mai mare numa r de variante echivalente? (adica sa se ob tina acela s i poligon; de exemplu T d a 16 variante c a ci dac a not a m triunghiurile 20 decupate prin I cel de sus si II cel de jos, respectiv prin F (fa ta si V (verso), atunci ) ele pot fi a sezate pentru a forma pentagonul n cele doua tii S (sus) si J (jos) pozi astfel: S : IF IF IV IV IIF IIV IIF IIV J : IIF IIV IIF IIV IF IF IV IV si toate aceste combina tii nca o data numa rate daca ntoarcem ntreg pentagonul pe verso. 5) Daca ta tii nu se pot realiza? se considera o singura fa a cartonului care situa (De exemplu T2 cu P4 ). 6) Care situa tii duc la poligoane congruente? (De exemplu P35 , P39 , P45 ). 7) De ce nu se pot realiza poligoane convexe cu mai mult de sase laturi? 8) Rezolvarea aproximativa tiilor de gradul 3 si 4 care au apa a ecua rut. (Acestea pot constitui un pretext pentru a prezenta la o activitate suplimentara formulele lui Cardano, respectiv Ferrari). 9) Relativ la ta ieturi: care este cea mai mica ? cea mai mare? cea care mparte poligonul n doua ti echivalente (de aceea si arie)? buca * * * n ncheiere sa tiala timea de situa tii de revenim la problema ni adaptnd-o la mul mai sus si anume: Se poate ca printr-o singura tiala ta ietura n linie dreapta sa realiza m din figura ini un pa trat? Ra sa cum este indispunsul este armativ doar daca ndoim n prealabil cartonul a cat mai jos (dupa bisectoarea unghiului drept format de cele dou a t a ieturi prezentate n solu tia sa).

MATEDar aceasta si ta tia o alta problema cu ndoire iere poate ini RECREATIE MATICA.

24

Asupra unor perechi de siruri liniar recurente


1 D. M. BATINE TU-GIURGIU

n aceasta tia proprieta tile unor perechi de siruri, nota matematica vom eviden fiecare satisfa ta cnd o anumita recuren liniara omogena de ordinul al doilea cu coecien ti constan ti. Spunem ca sir (xn )n0 de numere reale, satisface o recuren ta un liniara (reala ), omogena si exista de ordinul al doilea, daca exista a, b R, b 6= 0 k N astfel nct: xn+2 + axn+1 + bxn = 0, n k. (1) n func tie de valorile coeficien tilor a, b R si de condi tiile ini tiale xk = u R, xk+1 = v R se ob tin diferite siruri, dintre care unele cunoscute si de elevii de liceu. De exemplu, daca tine recuren ta: a = b = 1, k = 0 se ob (2) xn+2 xn+1 xn = 0, n N, pe care o vom numi recuren ta Fibonacci-Lucas. Daca ta (2) considera tine n recuren m x0 = F0 = 0, x1 = F1 = 1, xn = Fn se ob s ta: irul lui Fibonacci care satisface recuren (3) Fn+2 = Fn+1 + Fn , n N. tine Daca ta (2) considera n recuren m x0 = L0 = 2, x1 = L1 = 1, xn = Ln se ob s sir care satisface recuren ta: irul lui Lucas, Ln+2 = Ln+1 + Ln , n N. (4) Daca tine recuren ta liniara n (1) lua m a = 2, b = 1, k = 1 se ob : xn+2 2xn+1 + xn = 0, n N xn+2 xn+1 = = x2 x1 = r R, n N , (5) numita ta progresiilor aritmetice de ra tie r R. recuren n fine, daca ta (1) lua tinem recuren ta n recuren m a = 2, b = 1, k = 0 ob liniara ti constan ti de tip Pell : cu coeficien xn+2 2xn+1 xn = 0, n N. (6) tine Daca ta (6) considera n recuren m x0 = P0 = 0, x1 = P1 = 1, xn = Pn se ob s ta: irul lui Pell, care satisface recuren Pn+2 = Pn + 2Pn+1 , n N. (7) tinem De asemenea, daca n (6) lua m x0 = Q0 = 1, x1 = Q1 = 1, xn = Qn ob s sir care satisface recuren ta: irul lui Pell asociat (Qn )n0 , (8) Qn+2 = 2Qn+1 + Qn , n N. Mai departe, vom enun ta si demonstra unele propozi tii care scot n eviden ta , anumite proprieta ti pe care le verifica sir care verifica unele perechi formate dintr-un recuren ta (2) si un sir care verifica ta (5). recuren Propozi tia 1. Daca ta (2) n care x0 = c R+ , (xn )n0 satisface recuren x1 = d R tie r R, atunci: de ra + , iar (un )n1 este o progresie aritmetica n X uk xk = un xn+2 r (xn+3 x4 ) x2 u1 , n N . (9)
k=1
1

Profesor, Colegiul Na tional Matei Basarab, Bucure sti

25

ntr-adeva tia (10) este echivalenta r, rela cu n X uk xk + un+1 xn+1 = un+1 xn+3 r (xn+4 x4 ) x2 u1
k=1

Demonstra tie. Vom demonstra afirma tia prin metoda induc tiei matematice. Pentru n = 1 rela tia (9) devine u1 x1 = u1 x3 r (x4 x4 ) x2 u1 x2 u1 = = u1 (x3 x1 ) = u1 x2 ceea ce arata tul este adeva ca pentru n = 1, enun rat. Pentru n = 2 rela tia (9) devine u1 x1 + u2 x2 = u2 x4 r (x5 x4 ) x2 u1 u2 (x4 x2 ) = u1 (x1 + x2 ) + r (x5 x4 ) u2 x3 = (u1 + r) x3 = u2 x3 de unde se deduce ca tul este adeva si pentru n = 2. enun rat Presupunem ca tul este adeva tia (9) este verifi enun rat pentru n 2, (adica rela cata si sa si pentru n + 1. Avem de ara ) demonstra m ca ea este verificata tat ca : n +1 X uk xk = un+1 xn+3 r (xn+4 x4 ) x2 u1 . (10)
k=1

rela tie care (n condi tiile verifica tiei (9)) este echivalenta rii condi cu:

un xn+2 r (xn+3 x4 ) x2 u1 + un+1 xn+1 = un+1 xn+3 r (xn+4 x4 ) x2 u1 un+1 (xn+3 xn+1 ) = un xn+2 + r (xn+4 xn+3 ) xn+2 un+1 = un xn+2 + rxn+2 = (un + r) xn+2 = un+1 xn+2 de unde (daca tinem seama ca tia (10) este ade xn R rela + , n N ) deducem ca va tiei matematice rezulta tul este adeva rata . Conform principiului induc ca enun rat pentru orice n N si astfel propozi tia este demonstrata . Observa tie. Daca x0 = 0 = F0 , x1 = 1 = F1 iar (un )n1 este o progresie aritmetica tie r din rela tia enun tului deducem ca de ra n X uk Fk = un Fn+2 r (Fn+3 F4 ) u1 ,
k=1

adica tinut Problema C:2310 propusa am ob de Florin Rotaru n G.M.-9/2000, p.360. Daca aici lu a m r = 0 s i u = 1 , n N deducem ca sirul lui Fibonacci n verifica rela t ia n X Fk = Fn+2 1, n N .
k=1

tie r. atunci s irul (un )n1 este o progresie aritmetica de ra Demonstra tie. Vom face si aici demonstra tia prin metoda induc tiei matematice. Pentru n = 2 rela tia enun tului devine: u1 (x1 + x2 ) + rx3 = u2 x3 u1 x3 + rx3 = u2 x3 (u1 + r) x3 = u2 x3 de unde daca tinem seama ca xn > 0, n N deducem ca u2 = u1 + r. 26

Propozi tia 2. Daca ir de numere reale strict pozitive care (xn )n0 este un s satisface recuren ta (2) iar s irul (un )n1 are proprietatea ca exista r R astfel nct n X uk xk = un xn+2 r (xn+3 x4 ) x2 u1 , n N , (11)
k=1

u1 x1 +u2 x2 = u2 x4 r (x5 x4 )x2 u1 u1 x1 +r (x4 + x3 x4 )+u1 x2 = u2 (x4 x2 )

Presupunem ca si sa un = un1 + r, n 2 demonstra m ca un+1 = un + r. ntr-adeva tia (11) se scrie: r, pentru n + 1 rela n +1 X uk xk = un+1 xn+3 r (xn+4 x4 ) x2 u1
k=1

k=1

n care daca tinem seama de rela tia (11) ob tinem: un+1 (xn+3 xn+1 ) = un xn+2 + r (xn+4 xn+3 ) un+1 xn+2 = un xn+2 + rxn+2 de unde prin simplificare cu xn+2 > 0, n N deducem ca un+1 = un + r. Conform principiului induc tiei matematice rezulta ca un+1 = un + r, n N ceea ce arata tie r. ca (un )n1 este o progresie aritmetica de ra tie r > 0, u1 > 0 Propozi tia 3. Daca de ra (un )n1 este o progresie aritmetica iar (xn )n0 este un s i ir de numere reale astfel nct x0 0, x1 > 0, x2 = x1 + x0 s daca n X uk xk = un xn+2 r (xn+3 x4 ) x2 u1 , n N , (12)
k=1

n X

uk xk + un+1 xn+1 = un+1 xn+3 r (xn+4 x4 ) x2 u1

un xn+2 r (xn+3 x4 ) x2 u1 + un+1 xn+1 = un+1 xn+3 r (xn+4 x4 ) x2 u1

atunci xn+2 = xn+1 + xn , n N. Demonstra tie. Proceda si acum prin induc tie matematica tum . Conform enun lui pentru n = 0, avem x2 = x1 + x0 . Pentru n = 1 rela tia (12) devine u1 x1 = u1 x3 r (x4 x4 ) x2 u1 u1 x1 + u1 x2 = u1 x3 de unde, daca tinem seama ca un > 0, n N , ob tinem ca tia enun tului este adeva x3 = x2 + x1 , adica arma rata si pentru n = 1. Presupunem ca si sa xk+2 = xk+1 + xk , k = 0, n demonstra m ca (13) xn+3 = xn+2 + xn+1 . Daca n (12) nlocuim n cu n 1 deducem ca
n 1 X k=1

uk xk = un1 xn+1 r (xn+2 x4 ) x2 u1 , n 2.


n 1 X k=1

(14)

Daca tinem seama de (14) ob tinem ca n (12) un xn + uk xk = un xn+2 r (xn+3 x4 ) x2 u1

un xn + un1 xn+1 r (xn+2 x4 ) x2 u1 = un xn+2 r (xn+3 x4 ) x2 u1 un (xn + xn+1 xn+2 ) + r (xn+3 xn+2 xn+1 ) = 0, dar xn + xn+1 xn+2 = 0 n baza ipotezei de induc tie si deci ra mne si Conform principiului induc tiei matematice rezulta ca xn+2 = xn+1 + xn , n N astfel propozi tia este demonstrata . Bibliografie , Editura Albatros, Bucure sti, 1979. 1. M. D. Ba tu - Siruri tine 2. Gazeta Matematica tia 1895-2001. , Colec 27 r (xn+3 xn+2 xn+1 ) = 0 xn+3 = xn+2 + xn+1 .

Asupra unei probleme de construc tie


Anca TIMOFTE, Alexandru T URCANU 1
Punctul de plecare al acestei note a fost una dintre problemele propuse spre rezolvare absolven tilor clasei a VII-a n cadrul Concursului "Recrea tii Matematice" din 27 august 2002. Enun tul acestei probleme este urma torul: Fie dat un segment [M N ]. Construi ti cu rigla s i compasul un pa trat ABCD astfel nct M [AB ], AM = M B , iar N [AC ], AN = 3N C . (Descrie ti toate construc tiile care trebuie efectuate.) (Gabriel Popa) Prezenta tia data sa cum a reie sit din m n continuare solu de autorul problemei, a baremul de corectare: A D Solu tia 1. Sa si fie P presupunem problema rezolvata mijlocul segmentului [BC ]. Atunci N P kBD, P M kAC (ca \ linii mijlocii), deci N P M P si m(M P B ) = 45 a . Daca a 2 M este lungimea laturii pa , NP = tratului, atunci M P = 2 N a 10 1 a 2 , deci M N = , de unde N P = M N . = 4 4 5 B C Construc tia. Vom lua ca unitate un segment u de P lungime egala cu cea a segmentului [M N ].

u 5 u 5 5u u u x

x=

1 u 5

u 1 Ca n figurile de mai sus, construim un segment de lungime u. Intersecta m 5 1 cercul de diametru [M N ] cu cercul de centru N si raza tinnd punctul P . u, ob 5 Construim triunghiul dreptunghic isoscel de ipotenuza si afla [M P ] m astfel vrful B al pa ta tratului. Apoi, A si C sunt simetricele lui B fa de M , respectiv P . Vrful D se ob tine ca intersec tie a paralelelor duse prin A si C la BC , respectiv AB . Demonstrarea faptului ca tile ABCD astfel determinat este pa trat cu proprieta dorite, este imediata . Evident c a problema are solu t ie, unic a pn a la o izometrie a planului. Vom da mai jos nca tii ale acestei probleme. Prima are avantajul ca doua solu nu folose ste nici un punct auxiliar; este nsa stere a construc tiilor necesara o buna cunoa
1

Elevi, S coala nr.7 "Octav B a sani ncila ", Boto

28

cu rigla si compasul. A doua are la baza tionament mai elaborat, dar utilizeaza un ra numai construc tii la nivelul manualelor. Solu tia 2. Pe figura si nota tiile din prima solu tie, aplica m teorema cosinusului n 4AM N : MN2 \ = AM 2 + AN 2 2AM AN cos(M AN ) = !2 a 2 3a 2 5a2 a 3a 2 2 + = = , 2 2 4 2 4 2 8

2 10 u, Rezulta tie : determina urma toarea construc m un segment de lungime a = 5 unde u = M N . Vrful A al pa tia arcului capabil de 45 tratului este la intersec a construit pe [M N ] drept coarda , cu cercul de centru M si raza . Afla m apoi B ca 2 fiind simetricul lui A fa ta de M etc. D Solu tia 3. Presupunem problema rezolvata A si aplica m teorema lui Menelaus n 4ABC cu transversala M N P ; ob tinem: AM BP CN M =1 MB P C NA BP T N = 3 BC = 2P C. PC Aplicnd acum Menelaus n 4M N P cu transver- B P C sala C N A, ga sim: MN P C BA M N =1 = 1 M N = N P. CB AM N P NP \. \ CDP Sa si ADM observa m ca 4DAM 4DCP (C.C.), de unde M D = DP Ultima rela tie arata c a \ \ \ ) = m(M \ \ ) = m(ADC \) = 90 , m(M DP ) = m(M DC ) + m(CDP DC ) + m(ADM a sadar 4M DP este dreptunghic isoscel. Fie {T } = M P CD; teorema fundamentala PT PC 1 a sema = = . na rii aplicata n 4P BM cu CT kBM arata ca PM PB 3 Construc tia. Afla ta m P ca simetric al lui M fa de N . Intersecta m cercul de diametru [M P ] cu mediatoarea acestui segment, determinnd vrful D al pa tratului. PT 1 Afla = , apoi fie C m punctul T [M P ] care mparte segmentul n raportul PM 3 intersec tia dreptei DT cu semicercul de diametru [DP ] aflat n semiplanul delimitat de dreapta DP ce con tine punctul N . Vrfurile A si B ale pa tratului se construiesc acum cu u surin ta . Observa tie. Problema se poate generaliza considernd ca si N sunt punctele M luate astfel nct AM = m M B si AN = n N C .

deci a =

2 10 MN. 5

29

Cteva aplica tii ale teoremei lui Casey


Marius PACHI T ARIU 1
O generalizare remarcabila a teoremei lui Ptolemeu este teorema lui Casey. Se nume ste distan ta tiala si C2 , notata d12 , lungimea tangentei tangen dintre cercurile C1 lor comune exterioare. Teorema lui Casey. Daca cercurile C1 , C2 , C3 , C4 sunt tangente (toate interior sau toate exterior) la cercul C , ordinea punctelor de tangen ta ind data de numerotarea acestor cercuri, atunci are loc rela tia: Rezultatul ra mne adeva rat daca cercurile Ci (toate sau o parte din ele) degenereaza n puncte sau daca cercul C devine dreapta . Aplica tia 1. Fie ABC un triunghi nscris n cercul C s i cercurile C1 , C2 , C3 tangente la C interior precum s i laturilor (BC ), (CA) s i respectiv (AB ) astfel nct A s ti diferite fa ta i C1 , B s i C2 , C s i C3 sa fie de pa r de BC , CA, respectiv AB . Nota m cu l1 , l2 , l3 lungimile tangentelor din A, B, C la cercurile C1 , C2 , respectiv C3 . Are loc echivalen ta: b+c c+a a+b , l2 = , l3 = . d12 = d23 = d31 l1 = 2 2 2 A Solu tie. Observa C m mai nti ca C1 , C2 , C3 sunt tangente la laturi n mijlocul acestora. Aplicnd teorema lui Casey pentru cercurile C si A, C2 , C1 , C3 ; C si B , C3 , C2 , C1 ; C si C3 C , C1 , C3 , C2 , ob tinem: B b c c a a b C2 d13 + d12 = l1 d23 , d12 + d23 = l2 d13 , d23 + d13 = l3 d12 . 2 2 2 2 2 2 Din acestea, rezulta tia "". Invers, dupa imediat implica C1 C b+c nlocuirea lui l1 cu etc., aceste rela tii se scriu: 2 b (d13 d23 ) = c (d23 d12 ) , c (d12 d13 ) = a (d13 d23 ) , a (d23 d12 ) = b (d12 d13 ) , d12 d13 d23 d12 d13 d23 0 i.e. = = = a b c a+b+c (suma numa ra torilor fiind nula ). Deducem ca d12 d13 = 0, d23 d12 = 0, d13 d23 = 0, deci d12 = d23 = d31 , q.e.d. b) = 60 . Fie Aplica tia 2. Fie ABC un triunghi nscris n cercul C s i cu m(A 0 0 0 C (O , R ) cercul tangent la C interior s i la laturile [AB ] s i [AC ]. Sa se arate ca 4 A 0 R = r, unde r este raza cercului nscris n triunghiul dat. 3 C Solu tie. Fie {X } = AB C 0 , {Y } = AC C 0 si l = AX = C b ). = AY = XY (4AXY este echilateral, ca ci m ( A ) = 60 X Relativ la C si cercurile A, C , C 0 , B aplica m teorema lui Y Casey: O' B C b (c l) + c (b l) = al,
1

d12 d34 + d23 d41 = d13 d24 .

Elev, cl. a IX-a, Colegiul Na tional, Ia si

30

2bc bc bc 2bc 4 = = r= r = r. Pe de alta parte, a+b+c p S bc sin A 3 XY l 4 R0 = O 0 X = = . Ca urmare, R0 = r. 2 sin 60 3 3 Aplica tia 3. Cercurile Ci (Oi , ri ), i = {1, 2, 3, 4} sunt tangente (n ordinea numerota i, pentru orice i {1, 2, 3, 4}, Ci este tangent la Ci1 rii) la cercul C (O, r) s s tia ca i Ci+1 ( C1 ind C4 , iar C5 ind C1 ). Atunci, n condi punctele O, O1 , O3 ct s i O, O3 , O4 sunt coliniare, avem: a) 4r1 r2 r3 r4 = (r r1 ) (r r2 ) (r r3 ) (r r4 ), daca Ci sunt tangente interior la C ; b) 4r1 r2 r3 r4 = (r + r1 ) (r + r2 ) (r + r3 ) (r + r4 ), daca Ci sunt tangente exterior la C . Solu tie. Se stabile ste u sor ca doua cercuri de raze a O1 si b tangente exterior au lungimea a tangentei comune exterioare data de d = 2 ab. Ca urmare, teorema lui Casey ne conduce la rela tia: O O2 O4 2 r1 r2 2 r3 r4 + 2 r2 r3 2 r1 r4 = d13 d24 . de unde ob tinem ca l=

Datorita tii punctelor O, O1 O3 , avem d2 coliniarita 13 = O3 2 = (2r r1 r3 ) (r1 r3 )2 , adica = 4 ( r r ) ( r r3 ); d2 1 13 2 analog d24 = 4 (r r2 ) (r r4 ). nlocuind n rela tia precedenta tinem formula de la punctul a). Punctul b) se dovede ste n mod asema ob na tor. Aplica tia 4. Fie dat un cerc C s i pe el punctele A s i B . De o parte s i de alta a dreptei AB considera i tangente coardei m cercurile C1 , C2 tangente interior la C s [AB ] n punctele X s tia punctelor X s i respectiv Y . Sa i Y pe AB se determine pozi 1 pentru care d12 = AB . 2 Solu tie. Cu teorema lui Casey aplicata si cerA lui C curilor C1 , A, C2 , B , ob tinem AX BY + AY BX = AB d12 . d12 Condi tia din enun t este echivalenta cu X 1 AX BY + AY BX = AB AB C1 2 Y 2AX BY + 2AY BX = (AX + BX ) (AY + BY ) C2 C AX BY + AY BX AX AY BX BY = 0 B (AX BX ) (BY AY ) = 0 AX = BX sau AY = BY, adica si Y trebuie sa unul dintre punctele X fie mijlocul coardei [AB ] (cela lat putnd fi oriunde pe [AB ]) pentru a fi ndeplinita tia problemei. condi

Bibliografie. 1. M. Dra sin - Despre utilitatea unui rezultat prea pu tin folosit: teorema lui Casey, gu G.M. 12/1995, 716-720. 2. N. Roman - Asupra unor probleme date la O.I.M., G.M. 3/2000, 99-102.

31

Extinderi de inele si corpuri o posibila tie de recapitulare finala lec


Dumitru GHERMAN 1
Pentru ca recapitularea sa ta tina aiba eficien , trebuie ca n organizarea ei sa se de unele principii: la recapitulare nu se parcurge din nou ntreaga materie; trebuie sa se urma reasca , pe ct este posibil, realizarea unei lega turi ntre diversele ramuri ale matematicii scolare;

recapitularea trebuie sa aduca elemente noi, probleme care pot fi rezolvate prin prelucrarea creatoare a cuno stin telor anterioare; se are n vedere stimularea lucrului individual al elevului, folosind bibliografia indicata si / sau ca de profesor utnd noi surse; n cele ce urmeaza tie de , vom prezenta un proiect didactic pentru o posibila lec recapitulare finala la clasa a XII-a. Nu ne propunem sa rezolva m toate problemele sau sa tin fondului clasic demonstra m toate teoremele ce vor apa rea; majoritatea apar si poate fi consultata bibliografia. I. Inelul ntregilor pa tratici. Fie d un numa r ntreg liber de pa trate; definim o h i n Z d = x C | x = m + n d , m, n Z . h i 1) Z d ; +, este un subinel al corpului numerelor complexe, chiar domeniu de integritate. h i m n 2) Z d este izomorf cu inelul matricelor de forma , m, n Z, n dn m raport cu opera tiile i uzualehcu matrice. h i si numai daca 3) Inelele Z d si Z d 0 sunt izomorfe daca d = d 0 (se arata n primul rnd ca doua si un izomorfism f ntre cele inele invariaza elementele lui Z atunci el este bine determinat de valoarea f d ). h i 4) Subinelele unitare ale lui Z d sunt de forma o n An = a + bn d | a, b Z , n N. h i 5) Definim aplica tia norma N : Z d Z, N m + n d = m2 d n2 . Daca nota = m n d conjugatul ntregului pa tratic x = m + n d , se arata ca N m cu x are propriet a t i asem a n a toare modulului: N ( x ) = x x , N ( xy ) = N ( x ) N ( y ) . De h i aici, x U Z d N (x) U (Z) = {1}. 6) Grupul multiplicativ al elementelor inversabile din Z [i] este U (Z [i]) = {1, i}.
1

recapitularea trebuie sa tina si cerin tele examenelor scolare. cont de structura

Profesor, Liceul Teoretic Mihail Sadoveanu, Pa scani

32

7) Fie C\Q astfel nct mul timea A = {m + n | m, n Z} este inel fa ta de opera tiile uzuale din C. Daca A are exact patru elemente inversabile, atunci A = Z [i]. 8) Daca tine o infinitate de elemente si putem d {2, 3, 5}, atunci U (Z [d ]) con ga si n U (Z [d ]) elemente pozitive orict de mici (este suficient sa ga sim un singur element, considernd apoi puterile acestuia si conjugatele lor). Problemele 1-6 sunt rezolvate n [3]; problema 7 a fost propusa de Marcel Tena la etapa finala a Olimpiadei de Matematica n 1997, iar 8 poate fi ga sita n variantele examenului de bacalaureat din ultimii ani. II Corpul numerelor pa tratice. Fie d un numa r ntreg liber de pa trate; definim n o Q d = z C | z = a + b d, a, b Q . 1) Q d ; +, este subcorp al lui C (inversul elementului nenul a + b d 1 a b d d , deoarece a2 db2 6= 0; altfel, se ajunge la este 2 Q 2 a db a d= / R\Q!). b a b d este izomorf cu mul timea matricelor de forma , a, b Q, care 2) Q db a formeaza corp n raport cu opera uzuale. tiile 3) Corpurile Q si numai daca d d0 sunt izomorfe daca si Q d = d0 ; sin gurele automorsme ale corpului Q d sunt aplica tia identica si cea de conjugare, ambele invariind elementele lui Q. 4) Daca si f (x)= g (x) un subcorp K C este astfel nct End K = {f, g } x Q, atunci exista d . un ntreg liber de pa trate d 6= 1 pentru care K = Q 5) Daca z ) = f (z ), z Q d ; de aici urmeaza f Q [x], atunci f ( ca orice polinom cu coeficien ti ra tionali, are eventualele ra d n perechi da cini din Q conjugate. Problemele 1-3 pot fi ga site n [3], problema 4 a fost propusa la etapa finala a Olimpiadei de Matematica , iar 5 poate fi rezolvata din 1988 de ca tre Marcel Tena urmnd pas cu pas demonstrarea unor rezultate analoage din manuale. III Extinderi pa Corpuri pitagorice. Fie r Q r / Q; tratice. + astfel nct definim ca mai sus corpul Q ( r ) , care este subcorp al lui R din pozitivitatea lui r. 1) Q ( r) este cel mai mic subcorp al lui R care include Q { r } . 2) Putem gndi tiu vectorial, definind nmul tirea "vec pe Q ( r) ca un Q-spa torilor" din Q ( r ) cu "scalari" din Q prin restric t ionarea nmul t irii obi snuite din tiu vectorial este 2, o baza Q ( r) (de fapt, din R). Dimensiunea acestui spa ind si s i alte baze!). {1, r} (a se ga 3) Polinomul f = X 2 r Q [X ] este ireductibil peste Q, dar da cina admite ra r n Q ( r); orice alt polinom g Q [X ] care admite r a d a cina r se divide prin f. Spunem ca f este polinomul minimal al lui r . 33

d se face utiliznd 4) Din puncte de vedere geometric, extinderea lui Q la Q compasul. De exemplu, r este abscisa unuia dintre punctele de intersec tie ale 1 1 cercului de centru O si raza (r + 1) cu dreapta y = (r 1). 2 2 , r , Q , definim Q = Q r1 , Qr2 = Qr1 r2 , . . . 5) Considernd r 1 2 r + 1 am adjunc tionat la Q, pe rnd, elementele rn , . . . ; spunem ca , Qrn = Qrn1 r1 , r2 , . . . , rn , . . . . Am construit astfel sirul de extinderi Cum Q este mul time numa rabila , putem alege r1 , r2 , . . . , rn , . . . astfel nct orice numa tinut, pornind de la Q, prin efectuarea unui numa r ob r finit de aduna ri, sca deri, nmul tiri, mpa tiri si extrageri de ra de numa ste r da cini pa trate (un astfel[ r se nume numa tina timea nuQrn - mul apar unui anumit Qrn . Nota m K = r pitagoric ) sa
n

Q Qr1 Qr2 Qrn R.

merelor pitagorice; se arata sirului r1 , r2 , . . . , rn , . . . ca aceasta nu depinde de alegerea si ca formeaz a un subcorp al lui R , nchis la opera t iile aritmetice si la extragerea ra d a cinii p a trate s i care este "cel mai mic" (n sensul incluziunii) cu aceste propri eta ti. IV Construc tii cu rigla si compasul. 1) Daca L este un subcorp al lui R iar (D) este o drepta ce trece prin doua puncte avnd coordonatele n L L, atunci ecua tia dreptei are coeficien ti din L. Analog, un cerc care are centrul de coordonate din L L si trece printr-un astfel de punct, are coeficien tii ecua tiei sale din L. 2) Fie L subcorp n R, iar M (x, y ) un punct n plan; spunem ca M este constructibil cu rigla s tinut prin intersec tii i compasul plecnd de la L daca el poate fi ob de drepte si cercuri avnd coecien tii n L. Daca un astfel de punct, atunci M este fie x, y L, fie exista u L, u > 0 astfel nct x, y L ( u). 3) Numim numa i compasul un numa r real ce este coorr constructibil cu rigla s donata a unui punct constructibil. Se arata ca orice numa r real constructibil este totodata si numa ta r pitagoric. Ca o consecin , polinomul minimal al unui numa r constructibil are gradul putere a lui 2. De aici rezulta imposibilitatea dubla rii cubului, trisec tiei unghiului s i cuadraturii cercului (pentru ama nunte, v.[1],[2],[4]). 4) Corpul ordonat K R se bucura de un anumit tip de completitudine, numita si compasul, nu se va putea spune completitudine euclidiana : cercetat cu rigla niciodata ste vreun punct. Tocmai confuzia dintre aceasta ca lipse completitudine si completitudinea Cantor - Dedekind a lui R a ntrziat solu tionarea problemelor clasice ale antichita tii. Bibliografie 1. T. Brsan - Trisec tia unghiului, Recrea tii Matematice, 2/2001, 38-41. 2. E. Moise - Geometrie elementara dintr-un punct de vedere superior, E.D.P., Bucu re sti, 1980. 3. C. Ni ta sti, 1974. , T. Spircu - Probleme de structuri algebrice, Ed. Tehnica , Bucure 4. I. Tofan, C. Volf - Algebra - Inele, Module, Teorie Galois, Ed. Matrix-Rom, Bucure sti, 2001. 34

Metode si procedee de rezolvare a problemelor de maxim sau de minim


Gheorghe CROITORU 1
Ne propunem n cele ce urmeaza sa prezenta m, prin exemple, o serie de procedee si metode prin care pot fi solu tionate problemele de maxim sau de minim. Rezolvarea pe mai multe ca si probleme va permite cititorului sa ta i a unei aceleia compara eficien acestora, precum si sa aleaga contextul cel mai potrivit pentru aplicarea uneia sau alteia dintre ele. a3 Problema 1. Sa , x R se afle minimul expresiei E (x) = x2 + + , unde x a R+ este dat. Solu tia 1. Aplicnd inegalitatea mediilor, ob tinem ca r a3 a3 a3 a3 3 3 3 3 x 3 x + + 3 3 2 x +a 2 2 = 3 2a , 2 2 E (x) = = x x x 2 3 a a egalitatea fiind atinsa . Urmeaza , i.e. x = atunci cnd x3 = ca Emin = 3 2 2 3 3 2a2 = . 2 si suma x + y = const, atunci produsul Solu tia 2. Se stie ca , daca x, y R + x y m n x y (m, n N ) este maxim pentru = ; dual, daca xm y n = const, atunci m n x y suma x + y este minima = (aceste afirma tii se extind la un numa pentru r finit m n de termeni / factori si la cazul n care exponen tii sunt din Q+ ). 3 2 a = a6 = const, urmeaza ntruct x2 ca E (x) are valoare minima atunci x 3 3 2a2 a3 a a = . Se ob tine Emin = E cnd x2 = x= . 3 3 2x 2 2 2 Solu tia 3. Fie func tia f : (0, ) R, a x 0 f (x) = E (x). Extremele acestei func tii se pot 3 2 ga si folosind prima derivata . Avem ca f 0 (x) = 0 f 0 + + 2x3 a3 a f & E % = , care se anuleaza . pentru x = min 3 x2 2 Tabelul de varia tie este prezentat ala turat. sin x 3 Problema 2. Afla ti valorile extreme ale expresiei E (x) = , x R. cos x + 2 (Paul Georgescu, Gabriel Popa, Problema 24739, G.M. 9/2002) x tinem ca Solu tia 1. Pentru x 6= (2k + 1) , k Z, notnd t = tg , ob E = 2 2 2 3t + 2t 3 3t + 2t 3 , t R. Pentru a afla mul timea valorilor lui E , fie y = , = t2 + 3 t2 + 3 tia 0, ceea t R, deci t2 (y + 3) 2t + (3y + 3) = 0, unde t R. Se impune condi
1

Profesor, Liceul Teoretic Al. I. Cuza, Ia si

35

# 2 3 2 3 ce conduce la y 2 , 2 + . Pe de alta parte, E ((2k + 1) ) = 1 3 3 ! 2 3 2 3 2 3 2 3 , 2 + , iar Emax = 2 + 2 . Urmeaza ca Emin = 2 3 3 3 3 (se dovede ste u sor faptul ca aceste valori extreme sunt efectiv atinse). Solu tia 2. Definim punctele M (cos x, sin x), A (2, 3); atunci E (x) este tocmai panta dreptei AM . Pentru x R, M parcurge cercul trigonometric si cum A este n exteriorul acestui cerc, urmeaza ca valorile extreme ale lui E (x) sunt atinse atunci cnd AM este una dintre tangentele duse din A la cerc. Fie d : y 3 = m (x + 2) ecua tia unei drepte prin A; aceasta este tangenta la C (0, 1) cnd dist (O, d) = 1. Folosind formula care da ta de la un punct la o dreapta tinem distan , ob ( ) 2 3 |2m + 3| 2 = 1 3m + 12m + 8 = 0 m 2 . 3 m2 + 1 2 3 2 3 n concluzie, Emin = 2 , iar Emax = 2 + . 3 3 Solu tia 3. Putem evident utiliza derivata nti n studiul func tiei ata sate expresiei E (x). Problema 3. Dintr-o bara tine prin strunjire o metalica de forma cilindrica se ob bara tiune astfel paralelipipedica . Sa se determine dimensiunile dreptunghiului de sec nct pierderea de material sa fie minima . Solu tia 1. Notnd cu x si y dimensiunile dreptunghiului si cu R raza cilindrului, problema revine la a ga si maximul func t iei f ( x, y ) = xy , n condi tiile x, y > 0, x2 + y 2 = 4R2 . nsa , cum x, y sunt pozitive, produsul xy este maxim odata cu 2 2 2 produsul x2 y 2 . Deoarece suma x2 + y 2 este constanta , x este maxim pentru x y = utat sunt, prin urmare, x = y = R = y 2 = 2R2 . Dimensiunile dreptunghiului ca 2. 2 2 2 Solu tia 2. Studiul func t iei f ( x, y ) = xy pentru x, y > 0 , x + y = 4 R revine la studiul func tiei g (x) = x 4R2 x2 , x (0, 2R) si acesta se face apelnd la derivata g 0 . Solu tia 3. Cnd se cauta tii f (x, y ), ntre variabile existnd extremele unei func o lega tur a de forma ( x, y ) = 0 , se aplic a n general metoda multiplicatorilor lui Lagrange. Daca tia auxiliara f, sunt de clasa C 1 , considera m func F (x, y ) = f (x, y ) + (x, y ) , tii daca si Punctul (x0 , y0 ) din domeniul lui f este punct de extrem al acestei func numai daca tie a sistemului (x0 , y0 ) este solu
0 (x, y ) = 0, Fx 0 Fy (x, y ) = 0,

"

R.

(x, y ) = 0,

neadmi tnd solu tiile pentru care 2x = 2y = 0. Ob tinem imediat ca x = y = R 2. Problema 4. Doua se A, B sunt situate respectiv la 10 km s i 15 km de un ora ru rectiliniu, iar proiec tia lungimii AB pe direc tia rului este de 20 km. Cele doua ora se trebuie alimentate cu apa de la o uzina amplasata pe marginea rului. Se cere 36

(x0 , y0 ) = 0, (x0 , y0 ) = 0 nu sunt satisfa iar egalita tile cute simultan. 2 2 2 n cazul nostru, F (x, y ) = xy + x + y 4R si avem de rezolvat sistemul y + 2x = 0, x + 2y = 0, x2 + y 2 4R2 = 0,

0 x

0 y

pozi tia uzinei pentru care lungimea conductelor ce o leaga se sa de cele doua ora fie minima . Solu tia 1. Daca ta tia rului, iar M este pozi tia A00 este simetricul lui A fa de direc uzinei, evident ca [AM ] [A00 M ], deci AM + M B = A00 M + M B . Aceasta din urma suma este minima cnd A00 , M, B sunt puncte coliniare; punctul M de amplasare a uzinei astfel ob tinut se caracterizeaza ta unghiurilor , fa prin congruen cute de M A, respectiv M B cu normala la direc tia rului (v. figura 1). Notnd x = A0 M , din 4M AA0 4M BB 0 ob tinem ca AA0 10 A0 M x = = x = 8 (km). BB 0 M B0 15 20 x
B B A A x M B A TA N P M TB d

Fig.1

Fig.2

Solu tia 2. Un fir inextensibil de lungime suficient de mare este fixat n A, trecut printr-un inel M ce poate culisa pe direc tia d si apoi printr-un mic scripete aflat n B . Capa sata tul liber are ata o greutate P , care la echilibru se va afla ct mai aproape de sol, minimiznd astfel lungimea M A + M B . n aceasta tie de echilibru, tensiunile pozi TA tioneaza telor este romb, si TB care ac n fire sunt egale n modul, paralelogramul for deci punctul ca ta unghiurilor si . utat M este determinat din nou de congruen Solu tia 3. Locul geometric al punctelor X pentru care XA + XB = const este o elipsa si B . Considernd fasciculul de elipse omofocale (de focare A si de focare A B ), punctul ca tia cu d a acelei elipse din fascicul ce este utat M este dat de intersec tangenta ta unghiurilor la d. Proprietatea optica a elipsei asigura din nou congruen si . q Solu tia 4. Cu nota tiile din Solu tia 1, AM = 100 + x2 , M B = 225 + (20 x)2 q si avem de determinat minimul func tiei f (x) = 100 + x2 + 225+(20 x)2 , x [0, 20]. Nota si metode de abordare a problemelor de ex. Exista multe alte procedee trem; men tiona ta lor, metodele programa si pe cele ale m, pentru importan rii liniare grafurilor. Pentru alte aplica tii, poate fi consultata bibliograa. Bibliografie. 1. M. Cerchez - Aplica tii ale matematicii n practica sti, 1975. , E.D.P., Bucure 2. A. Leonte, C. Niculescu - Culegere de probleme de algebra i analiza s matematica , Ed. "Scrisul Romnesc", Craiova, 1981. 3. C. Udri ste, E. Ta tiilor reale de variabile reale, i maxime ale func na sescu - Minime s Ed. Tehnica sti, 1980. , Bucure 4. Gazeta Matematica tie). (colec 37

Comentarii asupra unui exerci tiu


SU 1 Dan PLAE
Exerci tiu. Fie numerele naturale nenule a, b. Sa ta: se demonstreze echivalen Solu tie (tip culegere). 1) 19 | (5a + 4b) 19 | 11 (5a + 4b)
(19,4)=1 (19,9)=1

19 | (5a + 4b) 19 | (9a + 11b) .

19 | (55a + 44b) 19 | [19a + 4 (9a + 11b)] 19 | 4 (9a + 11b) 2) 19 | (9a + 11b) 19 | 5 (9a + 11b) 19 | (45a + 55b) 19 | [19b + 9 (5a + 4b)] 19 | 9 (5a + 4b) 19 | (5a + 4b) .

19 | (9a + 11b) .

Comentariu metodic Se poate pune, firesc, ntrebarea: prin care ra tionament s-a ajuns la concluzia ca expresia 5a + 4b trebuie nmul tita m n cele ce cu 11, iar 9a + 11b cu 5? Prezenta urmeaza ta un punct de vedere n aceasta privin . Determina m numerele naturale n, x, y, x 6= 0 astfel nct sa avem: n (5a + 4b) = 19 (xa + yb) + z (9a + 11b) . Comparnd coeficien tii lui a si b ob tinem: 19x + 9z = 5n . 19y + 11z = 4n nmul tind prima rela tie cu 4 si a doua cu 5 si sca tinem znd membru cu membru, ob 76x 95y = 19z , de unde, prin npa tirea la 19, rezulta r : Da si y acele valori pentru care expresia x + y + z este minima si z 6= 0. (Minim lui x malitatea expresiei precedente nu este necesara ci recomandata pentru simplificarea calculelor!). Astfel, considernd x = 1, y = 0 ob tinem z = 4. Rezulta si, deci, n = 11 egalitatea: 11 (5a + 4b) = 19a + 4 (9a + 11b) . n mod analog, pentru demonstrarea implica tiei reciproce, determina m numerele naturale n, x, y, z 6= 0 astfel nct sa avem: n (9a + 11b) = 19 (xa + yb) + z (5a + 4b) . Efectund calculele, se ob tine z = 11x + 9y . Rezulta , lund x = 0, y = 1, ca z=9 si n = 5. Deci, avem: 5 (9a + 11b) = 19b + 9 (5a + 4b) . Observa tie. Ra tionamentele sunt valabile si pentru numere ntregi! n ncheiere, propunem 1) 2) 3)
1

4x 5y = z.

cititorilor demonstrarea urma te: toarelor echivalen 11 | (2a + 5b) 11 | (3a + 2b) , 23 | (2a + 3b) 23 | (9a + 2b) , 19 | (11a + 2b) 19 | (18a + 5b) .

Profesor, S coala Normala si Vasile Lupu, Ia

38

Cteva probleme privind triplete pitagoreice


1 Mircea CR SMAREANU

Subiectul triplete pitagoreice are o istorie bogata , fiindu-i dedicate zeci de articole (a se vedea n acest sens capitolul IV din [2], unde la pagina 189 sunt citate si cteva tabele cu astfel de triplete). Defini tie. Tripletul de numere naturale nenule (x, y, z ) cu max (x, y ) < z se nume ste pitagoreic daca x2 + y 2 = z 2 . Se stie ca forma generala a unui triplet pitagoreic ([1], [2, ex.5.8, p.125-127]) este: x = 2 2 , y = 2, z = 2 + 2 , (1) cu , numere naturale nenule si prime ntre ele, adica (, ) = 1. n cele ce urmeaza prezenta m cteva generaliza ri ale unor rezultate referitoare la triplete pitagoreice, rezultate aate n bibliografia romna . 1. 60 | xyz . Demonstra tie. Avem xyz = 2 2 2 2 + 2 = 2 4 4 . (i) Divizibilitatea cu 3 (5). Daca (5) am terminat. Daca sau este multiplu de 3 nu, conform teoremei lui Fermat, avem 2 2 1 (mod 3) 4 4 1 (mod 5) de unde rezulta si 5. divizibilitatea cu 3 (ii) Divizibilitatea cu 4. Deoarece (, ) = 1 cel mult unul dintre si poate fi par. 2l + 1 xyz = 4k (2l + 1) 4k 2 4l2 4l 1 1 ) = 2k , = (ii2 4k + 4l2 + 4l + 1 2 2 ( ii ) = 2 k + 1 , = 2 l + 1 xyz = 16 (2 k + 1) (2 l + 1) k + k l l 2 2k 2 + 2k + 2l2 + 2l + 1 . n concluzie avem si divizibilitatea cu 4. Observa tie. Divizibilitatea cu 4 constituie Problema C:827, G.M.-10/1988, autor Augustin Stan, iar divizibilitatea cu 5 Problema E:6303, G.M.-8/1978, fa ra autor. n [2] la pagina 171 este citat P. Lenthric ca fiind autor al acestui rezultat, n jurul anului 1830! 2. z s i orice putere a sa este suma a doua pa trate diferite. Demonstra tie. Pentru z avem concluzia datorita tiei (1) cu (, ) = 1. Pen rela tefan Kleitsch tru puterile lui z aplica m Problema E:5888 , G.M.-5/1977, autor S (pentru rezolvare a se vedea G.M.-10/1977, p.405-406): Daca un numa r natural este suma a k pa trate diferite atunci orice putere a sa este suma a k pa trate diferite. 3. Se cer lungimile catetelor unui triunghi dreptunghic a sa nct produsul lor sa fie de p ori perimetrul, cu p un numa r prim dat.
1

Lector dr., Facultatea de matematica si , Univ. Al. I. Cuza, Ia

39

Demonstra tie. Din xy = p (x + y + z ) rezulta 2 2 2 = p 22 + 2 adica tiile (, ) = ( ) = p. Cum p este numa r prim rezulta ca avem solu (1, p) , (p, 1) deci (, ) = (p + 1, 1) , (p + 1, p). n concluzie avem: 2 (i) x = (p + 1) 12 = p (p + 2), y = 2 (p + 1), (ii) x = (p + 1)2 p2 = 2p + 1, y = 2p (p + 1). Observa tie. Pentru p = 2 se ob tine Problema OG:111, G.M.-1/1991, autor Valer Pop. 4. (G.M.-5/1979, Problema O:35, Bucur B. Ionescu) Exista triplete pitagoreice cu x, y, z numere prime? Solu tie. Din y = 2 rezulta singura posibilitate = = 1 dar atunci x = 2 2 = 0, imposibil. Deci ra spunsul este negativ. 5. (G.M.12/1979, Problema E:6736 , I. Joldi s) x + y + z | xy . Demonstra tie. x + y + z = 2 ( + ), iar xy = 2 2 2 .

6. (Problema 7.8, [3, p.190 + p.199]) x2 xy + y 2 este suma a doua pa trate. 2 2 2 2 2 2 2 2 2 2 2 Demonstra tie. x xy +y = 2 +4 = ( + ) + +2 ( )2 . 7. Daca i q sunt numere naturale nenule s i prime ntre ele, sa p s se rezolve ecua tia diofantica p2 x2 + q 2 y 2 = 2p2 q 2 z 2 . Solu tie. Considernd x = qu si y = pv ob tinem u2 + v 2 = 2z 2 de unde rezulta 2 2 uv u+v u+v uv 2 2 2 2 2 z = + si deci = 2 , = , z = + . n 2 2 2 2 concluzie: x = q 2 + 2 2 , y = p 2 2 2 , z = 2 + 2 cu (, ) = 1. Observa tie. Pentru p = 2, q = 3 se ob tine Problema 5.9 din [3, p.119].

Bibliografie 1. V. Claudian - Analiza diofantica , G.M.-1/1970, 1-9. 2. L. E. Dickson - History of the theory of numbers, vol. II - Diophantine Analysis, Chelsea, N. Y., 1952. 3. P. Radovici-Ma rculescu - Probleme de teoria elementara , a numerelor, Ed. Tehnica Seria Culegeri de probleme de matematica si zica sti, 1986. , Bucure

40

Cteva aplica tii ale inegalita tii Cauchy-Buniakowski


1 Ioana CRACIUN s i Gheorghe CRACIUN

2 2 2 0 (ai bj aj bi )2 = a2 i, j = 1, n. i bj 2 (ai bj ) (aj bi ) + aj bi , 2 2 2 2 S b b S ( ab ) + b S a Suma si ob t inem: 0 a 2 a , i = 1, n. Suma m m dupa j i i i i acum dupa S 2 (ab) S (a) S (b), i: 0 S a2 S b2 2S 2 (ab) + S a2 S b2 , adica q.e.d. Aplica tii 1. Fie pa i M , N doua tratul ABCD s puncte pe cercul nscris n acest pa trat. Sa se arate ca 1 AABCD (AM AN + BM BN + CM CN + DM DN ) . 3 Fie P, R, S, T mijloacele laturilor AB, BC, CD,DA. Din teorema medianei avem: M A2 + M B 2 AB 2 M B 2 + M C 2 BC 2 , M R2 = , MP2 = 2 4 2 4 M C 2 + M D2 M D2 + M A2 DA2 CD2 , MT 2 = . M S2 = 2 4 2 4 Adunnd ob tinem M P 2 + M R2 + M S 2 + M T 2 = M A2 + M B 2 + M C 2 + M D2 AB 2 .

Daca i b1 , b2 , . . . bn sunt numere reale, n N , n 2, atunci are loc a1 , a2 , . . . an s inegalitatea Cauchy-Buniakowski: 2 2 2 2 2 (a1 b1 + a2 b2 + + an bn ) a2 b1 + b2 1 + a2 + + an 2 + + bn , a2 an a1 cu egalitate daca sau ai = bi = 0, i = 1, n. = = = i numai daca s b1 b2 bn n n P P a2 ak bk . Avem: Demonstra tie. Nota m S a2 = k , S (ab) =
k=1 k=1

Triunghiurile P M S si T M R sunt dreptunghice, deci M P 2 + M S 2 = P S 2 = AB 2 si M T 2 + M R2 = T R2 = AB 2 . Adunnd ob tinem M P 2 +M S 2 +M T 2 +M R2 = 2AB 2 , deci M A2 + M B 2 + M C 2 + M D2 = 3AB 2 (egalitate cunoscuta ). Analog, N A2 + N B 2 + N C 2 + N D2 = 3AB 2 . Folosind inegalitatea Cauchy-Buniakowski, avem: M A2 + M B 2 + M C 2 + M D2 N A2 + N B 2 + N C 2 + N D2 9AB 2 (AM AN + BM BN + CM CN + DM DN )2 3AABCD AM AN + BM BN + CM CN + DM DN. 2. Fie a1 , a2 , . . . an R. Sa tiind ca a1 x1 + a2 x2 + + se ae x1 , x2 , . . . xn s n n P P 2 2 +an xn = a2 a2 si x2 1 + x2 + + xn = i i.
i=1 i=1
1

(AM AN + BM BN + CM CN + DM DN ) sau

Profesori, Plopeni (Prahova)

41

Conform inegalita tii C-B, avem:

cu egalitate daca xi = kai , i = 1, n. Tinnd cont de condi tiile din enun t, rezulta n n n P 2 2 P 2 P 2 ca ai ai ai , care are loc cu semnul de egalitate. Atunci si
i=1 i=1 i=1

2 2 2 2 x1 + x2 (a1 x1 + a2 x2 + + an xn )2 a2 1 + a2 + + an 2 + + xn ,

tinem problema C:2331 Observa tie. Pentru n = 3 si a1 = 2, a2 = 2, a3 = 3 ob (Alfred Eckstein, G.M. - 11/2000). 3. Fie A1 , A2 , . . . An un poligon convex s i M un punct n interiorul sa u. Se proiecteaza M pe laturile A1 A2 , A2 A3 , . . . , An A1 n punctele M1 , M2 , . . . , Mn . Sa se arate ca : 1 A1 A2 2 + A2 A3 2 + + An A1 2 . A1 M1 2 + A2 M2 2 + + An Mn 2 4 Fie i {1, 2, 3, . . . , n} fixat. n triunghiurile M Ai Mi si M Ai+1 Mi aplica m teorema lui Pitagora: Mi M 2 = Ai M 2 Ai Mi 2 = Ai+1 M 2 Ai+1 Mi2 (considera m An+1 = A1 ). Scriind a doua egalitate pentru i = 1, n si sumnd, ga sim: n n X X Ai Mi 2 = Ai+1 Mi 2 .
i=1 i=1

rela tia precedenta si rezulta este o egalitate ca xi = kai , i = 1, n. nlocuind n prima n n P P 2 2 2 condi tie din enun t, ob tinem: ai = k a2 si xi = ai , i = 1, n. i , deci k = 1
i=1 i=1

1 2 2 a1 + a2 2 + + an . Aplicnd inegalitatea C-B, 2 2 2 2 2 a1 + a2 avem: (a1 x1 + a2 x2 + + an xn )2 x2 1 + x2 + + xn 2 + + an , cu egalitate daca si numai daca ta ai = kxi , i = 1, n. n consecin , rezulta ca 2 1 2 2 2 2 2 2 a + a2 a2 x2 2 + + an 1 + x2 + + xn 1 + a2 + + an 2 1 1 2 2 2 2 2 a1 + a2 2 + + an x1 + x2 + + xn , 4 adica tia dorita tia ai = kxi , i = 1, n; rela . Pentru a avea egalitate punem condi ga sim k = 2 . Deci M , . . . M sunt pe mediatoarele laturilor poligonului, care va fi 1 n inscriptibil, iar M va fi centrul cercului circumscris lui. ( ) a+b+c 4. Sa timii M = abc | p N . se determine suma elementelor mul 3 (a2 + b2 + c2 ) Inegalitatea (a + b + c)2 3 a2 + b2 + c2 se deduce imediat cu ajutorul inegaa+b+c lita tii C-B. Deci p 1, cu egalitate pentru a = b = c. Rezulta 3 (a2 + b2 + c2 ) ca si 111 + 222 + + 999 = 111 (1 + 2 + + 9) = M = {111, 222, 333, . . . , 999} = 111 45 = 4995. a1 x1 + a2 x2 + + an xn = 42

Daca tie se scrie: x2 nota m Ai Mi = xi , iar Ai Ai+1 = ai , aceasta ultima rela 1+ 2 2 2 2 2 +x2 + + xn = (a1 x1 ) + (a2 x2 ) + + (an xn ) sau, efectund calculele, 2 2 2 2 2 2 2 2 x2 1 +x2 + +xn = x1 +x2 + +xn +a1 +a2 + +an 2 (a1 x1 + a2 x2 + + an xn )

Concursul "Recrea tii Matematice"


Edi tia a II-a, Ia si, 27 August 2002 Clasa a VII-a
1. Fie x, y, z (0, ) cu x + y + z = 1. Sa se determine cele mai mici valori pe care le pot lua expresiile xy 2 yz 2 zx 2 xy yz zx E= + + . + + ; F = z x y z x y sani si Lucian Tu tescu, Craiova , Foc Cornel Noana si x = n2 + n. 2. Fie n N a) Sa si sa se arate ca x R\Q se afle [x]. b) Sa se determine primele doua zecimale de dupa virgula ale lui x pentru n = 20022002 . Cornel Noana sani , Foc 3. Fie dat un segment [M N ]. Construi ti numai cu rigla si compasul un pa trat ABCD astfel nct M [AB ], AM = M B , iar N [AC ], AN = 3N C . (Descrie ti toate construc tiile care trebuie efectuate.) Gabriel Popa, Ia si

Clasa a VIII-a
1. Daca si ctul a doua tionale sunt, fiecare, numere suma, produsul numere ira ra tionale, calcula ti suma cuburilor celor doua numere. Claudiu- Stefan Popa, Ia si (Recrea tii Matematice 2/2002) 2. Sa tia x2 (y + 1) + y 2 (x + 1) + 1 = 0. se rezolve n Z Z ecua Gabriel Popa, Ia si 3. Fie ABCA0 B 0 C 0 un trunchi de piramida oarecare. Nota m cu G, G0 centrele de greutate ale bazelor, iar {D} = BC 0 CB 0 , {E } = AC 0 CA0 , {F } = AB 0 BA0 .Sa se arate ca si GG0 sunt concurente. dreptele BE , CF Dan Brnzei, Ia si

Clasa a IX-a
1. Sa tia xn 3 [x] + 2 = 0, n N . se rezolve n R ecua Cornel Noana s ani , Foc 3 2. Sa . se arate ca pentru orice (0, 2 ), exista n Z astfel nct sin n 2 Gheorghe Iurea, Ia si ta 3. Fie Db , Dc , Fa , Ea puncte de tangen ale cercurilor exnscrise triunghiului ascu titunghic ABC cu dreptele suport ale laturilor, astfel nct B (Dc C ), C (BDb ), A B C B (AFa ), C (AEa ). Sa se arate ca : Dc Fa kDb Ea sin2 = sin2 + sin2 . 2 2 2 Temistocle Brsan, Ia si

Clasa a X-a
1. Fie P R [X ], P (X ) = X n + an1 X n1 + + a1 X + a0 . a) Sa si mai mari dect se arate ca daca toate ra da cinile polinomului sunt reale n 2, atunci (1) P (1) + an1 + 2n 1. 43

b) Sa se arate ca daca toate ra da cinile polinomului sunt reale, pozitive, mai mici 2n + 2n2 an2 + 2n4 an4 + . . . dect 2, atunci n1 < 1. 2 an1 + 2n3 an3 + 2n5 an5 + . . . Carmen Nejneru si Vlad Martinu si, Ia si 2 2. Sa se arate c a cos n arctg 2 Q , n N . Gheorghe Iurea, Ia si 0 0 s i B 3. Fie A, B doua puncte fixate, iar M un punct variabil n plan. Fie A imaginile lui M prin rota tiile n jurul punctului A, respectiv B , de unghi , respectiv 2 si direc tie, ara ti ca . Daca streaza aceea ta M parcurge o dreapta . vectorul A0 B 0 pa 2 Reciproca este adeva rata ? Gabriel Popa, Ia si

Clasa a XI-a
1. Fie P, Q R [X ] doua polinoame, fiecare avnd cte o ra da cina reala . Daca 1 1 4 4 P ti ca + x + Q (x) = Q + x + P (x) , x R, ara ta P = Q. 2002 2002 Lucian Tu tescu, Craiova 2. Fie A Mn (R) astfel nct exista si R, || 1, m N, m > n 3 pentru care Am+1 Am A + In = On . Sa se arate ca |det A| = 1. Lucian-Georges La si (Recrea tii Matematice 2/2002) dunca , Ia 3. Determina t i func t iile continue f : (0 , 1) (0 , ) pentru care f (x) f (y ) = xy , x, y (0, 1). =f 2xy x y + 1 Lucian Laza r, Baca u

44

Concurs de admitere 2002, Ia si


Facultatea de Informatica , Universitatea "Al. I. Cuza"
Analiza matematica 1. Fie (an )nN , (bn )nN doua siruri de numere reale. i) Daca si (bn )nN converge ca (an )nN converge ca tre a tre b, ce se poate spune despre convergen ta sirului a0 , b0 , a1 , b1 , . . . , an , bn , . . . ? Sa se justifice ra spunsul dat. (1)n 2n+1 + n+2 n, n N, sa ta si bn = (1) ii) Daca se studieze convergen an = n+1 sirului a0 , b0 , a1 , b1 , . . . , an , bn , . . . . 2. Fie func tia f : [1, 1] R, f (x) = ln x2 + 1 . Sa se arate ca : i) |f (x2 ) f (x1 )| < |x2 x1 |, x1 , x2 [1, 1], x1 6= x2 ; ii) exista un singur x0 (1, 1), astfel nct f (x0 ) = x0 .

Algebra 1. Fie (G, ) si (, ) doua grupuri. Sa se demonstreze ca daca f : G este izomorfism, atunci si f 1 : G este izomorfism. 2. Fie dat q Q . Sa se arate ca : i) func tia f : Z Q, f (k ) = q k , este morfism de la grupul (Z, +) la grupul (Q , ); ii) daca / {1, 1}, atunci exista q un subgrup al lui (Q , ) izomorf prin f cu (Z, +). Sa se precizeze acest subgrup. 3. Fie f Z3 [X ], f = b 1 X X 2 X n1 , n N . Ara ti ca ta f se divide 2, daca si numai daca prin X b n este multiplu al lui 3. 4. Sa si respectiv C, polinomul se descompuna n factori ireductibili peste Q, R g = X 4 + X 3 X 2 2X 2, stiind ca g se divide prin X , unde este o ra da cina de ordinul trei a unita tii.

Algebra - colegiu 1. Fie A M2 (R), t (A) suma elementelor de pe diagonala principala a matricii A si det (A) determinantul matricii A. Sa se arate ca : i) A2 t (A) A +det (A) I2 = O2 , unde I2 si respectiv si O2 sunt matricea unitate matricea nula din M2 (R); ii) daca O2 , atunci t (A) = 0. A2 = a11 a12 a13 2. i) Fie A = a21 a22 a23 M3 (R). Sa ti termenii care se scrie explicit to a31 a32 a33 apar n expresia determinantului matricii A si care sunt de forma (1)i+j +1 a1i a23 a3j . ii) fiind un parametru real, sa si sa se discute se rezolve sistemul: 2x 3y z = 1 . 4x + 6y + 2z = 2 ti ca 3. Fie H = x + y 2 | x, y Q, x2 2y 2 = 1 . Ara ta H este parte stabila a lui R n raport cu nmul tirea si ca toate elementele lui H sunt simetrizabile n raport cu opera tia indusa . 4. Sa se determine polinoamele f, g Z [X ], de gradul 1, astfel nct 2 X + 2X + 2 f + X 2 + 3X + 3 g = 1. 45

Fac. de Electronica si Telecomunica tii, Univ. Tehnica "Gh. Asachi"


Matematica 1. Ce rela tie exista si B = (n + 1) n , n 8? ntre numerele reale A = n n+1 a) A > B ; b) A = B ; c) A < B ; d) A B ; e) A B . k2 k 2. Mul timea solu tiilor inecua tiei C16 > C16 este a) {4, 5, . . . , 9}; b) ; c) {17, 18, 19}; d) {10, 11, . . . , 16}; e) {1, 2, . . . , 9}. 3. Fie inecua tia logx x + 30 1. Solu tiile acestei inecua tii sunt a) x (, 5]; b) x [6, ); c) x (1, 6]; d) x (1, ); e) x . Z 3 dx 4. Fie I (a) = ,aR si L = lim I (a). Atunci: a2 1 |x a| + 1 a) L = 2 ; b) L = 2 ln 2; c) L = 4 ln 2; d) L = 8 ln 2; e) L = ln 2. x + y + 2z = 1 x + (2 1) y + 3z = 1 5. Sistemul cu , R este compatibil nede x + y + ( + 3) z = 2 1 terminat pentru a) R, = 1; b) = 0, = 5 sau R, = 1; c) = 0, = 2; d) = 0, = 1 sau R, = 5; e) R, R. sin2 x cos2 x 6. Sa tiile ecua tiei = 2. se afle solu cos x (1 + tg x) sin x (1 + ctg x) a) x = k ; b) x = 2k ; c) x ; d) x = k ; e) x = k, k Z. 2 3 6 (1 + i)2002 7. Numa rul complex n este real pentru n N de forma (1 i) a) n N; b) n = 4k; c) n = 4k + 1; d) n = 4k + 2; e) n = 4k + 3, k N. 2 8. Se considera polinoamele f, g R [X ], f = X 2n X n + X 4 + 1, g = (X 1) . Sa tirii polinomului f la polinomul g . se determine restul r al mpa r a) r = (n + 4) X n 2; b) r = nX ; c) r = (n + 2) X + n 1; d) r = (n + 4) X + n 2; e) r = 2nX . x2 x+1 9. Se considera func t ia f care satisface rela t ia: 2 f + f = x, x+1 x2 pentru orice x R\ {1, 2}. Valoarea derivatei f (n) (2) este n! n! n! n! n! ; d) (1)n n ; e) n . a) (1)n n+1 ; b) n+1 ; c) (1)n n 3 3 2 3 3 3 xn + 1 10. Func tia f : D R, D R, f (x) = 3 , n N, are cel pu tin o asimptota x +1 verticala si nu admite asimptote orizontale sau oblice pentru a) n = 2k + 1, k N; b) n N; c) n < 4; d) n = 4; e) n = 2k, k 3.

Fac. de Automatica si Calculatoare, Univ. Tehnica "Gh. Asachi"


Matematica - ingineri 1. real a pentru care ra tiei 8ax2 da cinile x1 , x2 ale ecua 2 Valorile parametrului 2 2 a + 2a + 1 x + a + 1 = 0 satisfac x1 + x2 4x1 x2 sunt: a) a > 0; b) a = 0; c) a = 1; d) a = 1; e) a 6= 2. 2 2. Fie polinoamele f = X 2n X n + X 4 + 1, n > 4 si g = (X 1) . Sa se determine restul mpa r t irii lui f la g . 46

a) (n + 4) X n 2; b) nX + n 2; c) (n + 2) X n 2; d) (n + 4) X + n 2; e) (n 4) X + n 2. 1 1 1 3. Daca tiei x2 + x + 1 = 0 si A = 1 a a2 atunci: a este o ra da cina a ecua 1 a2 a 3 0 0 2 0 0 1 0 0 a) A2 = 0 0 3; b) A2 = 0 0 2; c) A2 = 0 0 1; 0 3 0 0 2 0 0 1 0 2 2 0 3 3 0 d) A2 = 0 2 2; e) A2 = 0 3 3. 2 0 2 3 0 3 1 4. Legea de compozi tie x y = (x + y xy + 1) este o lege de grup comutativ 2 pe mul timea: a) R; b) R\ {0}; c) R\ {0, 1}; d) R\ {1}; e) R\ { 1}. 1 1 1 . 5. Sa + + + se afle lim n 2n + 1 2n + 2 2n + n 3 1 3 a) ln 2; b) ln ; c) ; d) e; e) 1 + ln . 2 2 2 2 x + m x 6. Se da tia f : R\ {2} R, definita e , n care m func prin f (x) = x+2 este parametru real. Sa se precizeze valorile lui m pentru care f are dou a puncte de extrem. a) m [2, 6]; b) m (, 2/3]; c) m (2/3, 6); d) m (, 2) (6, ); e) m (, 2/3) (6, ). Z 1 2 x +a 7. Sa dx < 2. se determine parametrul a, astfel nct sa avem 1 < 2 0 x + 3 ! 2 3 2 3 3 a) 3 < a < 3 + ; b) 3 < a < 3; c) 3 < a < 3 1 + ; 7 . d) 3 < a < ; e) 3 < a < 6 2 8. Se da si de laturi a, b, c. Fie M, N, P proiec tiile centrului un triunghi de arie S cercului nscris pe laturi. Se cere aria 4M NP . abc (a + b + c) 4S 3 S2 3 3S 2 3 a+b+c 2 ; e) S ; b) ; c) ; d) . a) 2 3abc 16S 3abc abc (a + b + c) (a + b + c) 9. Sa tia sin (2x + 1) = cos (2x 1). se rezolve ecua a) + k ; b) + k ; c) + k ; d) (1)k + k ; e) (1)k + k , k Z. 8 2 8 8 2 8 2 8 10. Unghiul diedru dintre doua te adiacente ale unui octaedru regulat are fe ma sura, exprimata n radiani, egala cu: 2 5 3 3 ; c) ; d) 2 arccos ; e) 2 arcsin . a) ; b) 2 3 8 3 3 Matematica - colegiu 1. Solu tiile sistemului xy + x + y = 11 sunt: x2 y + xy 2 = 30 47

a) (x, y ) {(2, 3) , (3, 2)}; b) (x, y ) {(1, 5) , (5, 1); c) (x, y ) {(2, 3) , (1, 5)}; d) (x, y ) {(3, 2) , (5, 1)}; e) (x, y ) {(2, 3) , (3, 2) , (1, 5) , (5, 1)}. 2. Sa tia log2 (9 2x ) > 3 x. se rezolve inecua a) x < 8; b) 0 < x < < x < 4; d) x > 3; e) nu exista tii. solu 3; c) 2 1 0 1 3. Daca matricea A = 0 1 0 satisface A3 = aA2 + bA, atunci: 1 0 1 a) (a, b) = (3, 2); b) (a, b) = (3, 3); c) (a, b) = (2, 2); d) (a, b) = (3, 2); e) (a, b) = (2, 3). 4. Pe R se define ste legea de compozi tie prin rela tia: x y = xy + ax + 2by + 1, x, y R. Sa se determine a, b R astfel nct legea s a a si asociativ a fie comutativ . 1 1+ 5 1+ 5 1 ,b = sau a) a = 1, b = ; b) a = 0, b = 0 sau a = 1, b = ; c) a = 2 2 2 4 1 5 1 5 ,b = ; d) a = 4, b = 2; e) nu exista tie. a= solu 2 4 r n2 + 1 n + 1 ln . 5. Sa se afle lim n n+2 n 1 a) ; b) 1; c) e; d) e; e) . 2 x2 + mx + 2 6. Se considera tia f (x) = 2 , unde m R este un parametru. Sa func x + 2x + m se determine m, astfel nct domeniul ei de defini tie sa si sa fie R admita exact doua puncte de extrem. a) m (1, 2)(2, ); b) m (2, ); c) m (3, ); d) m (1, 2); e) m (, 1). R xdx , x (a, ) (a 6= 0). 7. Sa se calculeze 3/2 ( x + a ) x 2a 1 a + C ; b) 2 + C ; c) a) 2 x+a x+a + C; x+a x+a x+a x + 2a 2x+a+1 + C ; e) 2 + C. d) 3 x+a x+a 8. ntr-un patrulater convex se cunosc diagonalele d1 , d2 si un unghi dintre ele. Se cere aria patrulaterului ale ca rui vrfuri sunt mijloacele laturilor celui dat. 1 1 2 1 d1 + d2 a) d1 d2 cos ; b) d1 d2 sin ; c) 2 d1 d2 sin ; 4 4 4 1 1 d) d1 d2 (sin + cos ); e) (d1 + d2 )2 sin . 4 16 9. Se dau numerele x = cos 3, y = tg 3, z = ctg 3. Atunci a) x < y < z ; b) y < x < z ; c) z < y < x; d) x < z < y ; e) z < x < y . 10. Locul geometric al centrelor sferelor ce trec prin doua puncte distincte date este a) o sfera ; b) o dreapta ; c) doua drepte perpendiculare; d) un plan; e) doua plane perpendiculare.

48

Capacitate - teste prega titoare


Testul 1 (prof. Gheorghe TIMOHE ) I. 1. Valorile ra tionale ale numerelor x si y pentru care x 2 2 3 + +y 2 + 2 3 = 4 2 3 sunt ......................... 2. Descompusa n produs de doi factori expresia E (x) = x2 9y 2 8x + 16 este egala cu ......................... 98 97 96 95 94 93 92 91 99 + + + + + + + + , 3. Fie E = 98 97 96 95 94 93 92 91 90 unde [x] - partea ntreaga a E = ......................... a num rului x R; atunci p p 4. Solu tia n N a inecua tiei 3 + 2 2 x + 3 2 2 x < 2 este ......................... 5. Daca si y satisfac rela tia 2x + 3y = 16, atunci x = numerele prime x ........................., y = ......................... 6. ntr-un vas n forma de cub se toarna 6 l apa , ceea ce reprezinta 75% din capacitatea vasului. Diagonala cubului este egala cu ......................... 7. Dimensiunile unui paralelipiped tionale cu 6, 8 dreptunghic sunt direct propor si 10, iar diagonala are lungimea de 10 2 cm. Volumul paralelipipedului este ............ 8. Fie ABCD un trapez cu bazele AB = 13 cm si CD = 15 cm, M [AD] astfel nct M D = 2M A si M N kAB , N [BC ]. BN a) Valoarea raportului este ......................... b) M N = ......................... BC 9. Fie AC si BC doua tangente la un cerc C (O, R) n A si B , m(ACB ) = 60 , AO = 8 cm. a) m(AOB ) = ......................... b) lungimea coardei AB = ......................... II. 1. a) Reprezenta ti ntr-un sistem de coordonate carteziene toate perechile (x, y ) care verifica tiile |x 1| = 3 si |x y | 3. rela b) Fie E (x) = (3 |x| 1) / |x|. Determina ti x R pentru care E (x) N. 2. Demonstra ti ca telor de la vrfurile unui triunghi la o dreapta suma distan exterioara telor de la mijloacele laturilor triunghiului la lui este egala cu suma distan aceea si dreapta . 3. Pe planul pa tratului ABCD de latura a se ridica perpendiculara OA = a 2. Fie M si N proiec tiile lui A pe (OBC ) si (OCD), iar P si L proiec tiile lui M si A pe (AOD), respectiv M H . Sa si AL; b) unghiul diedru al se determine: a) M P planelor (AM H ) si (ABC ); c) pozi tia punctului Q pe OC astfel nct perimetrul 4OBD sa fie minim. Testul 2 (prof. Gheorghe TIMOHE )
I. 1. Propozi tia "n N, n (n + 3) + 3 este numa r prim" este ......................... 1 1 2 1 + ++ . Dintre S si este mai mare ......................... 2. Fie S = 100 101 299 3 3. Solu tiile ecua tiei ||3x 1| 12 | = 4 sunt ......................... ab, a (b, (a + b) /2) = ......................... 4. a) Daca 0 < b < a, atunci b) Daca A = 21999 , 22000 Z, atunci card A = ......................... |x + 2| + |y 3| = 7 5. Rezolvnd sistemul , ob tinem (x, y ) ......................... |x + 2| + 3 = y 3 6. Fie ABCD - trapez de baze AB = 7 cm, CD = 2 cm si laturile neparalele 49

BC = 4 cm si AD = 3 cm. Ma si BC este egala sura unghiului dintre AD cu ......................... grade. 7. Fie ABCDA0 B 0 C 0 D0 - cub. Atunci m( (A0 C 0 , AD0 )) = ......................... 8. Aria 4ABC este 268 cm2 . Aria triunghiului format de mijloacele laturilor sale este ......................... 9. Fie 4ABC cu m(A) = 60 , m(B ) = 80 , (AA0 , (BB 0 , (CC 0 - bisectoarele unghiurilor 4ABC (A0 , B 0 , C 0 apar tin cercului circumscris 4ABC ). a) m(B 0 C 0 A0 ) = ......................... b) m(A0 B 0 C 0 ) = ......................... II. 1. Fie f : R R, f (x) = 3x + 1, g : R R, g (x) = x + 1. a) Calcula tei determinata graficele tiilor si Ox. de ti aria suprafe func b) A = (x, y ) | x, y R , f ( x + y ) f 2 xy . Reprezenta ti grafic elementele + lui A. c) B = n N | |g (n)| 8 2 . Determina ti suma elementelor din B . 2. Fie numerele a = 2+1 3 2 . . . 100 + 99 101 100 si 21 3 + 2 . . . 100 99 101 + 100 . b= a) Ara ti ca ti numerele (1 a) (1 + b) si b a. ta a + b > 2. b) Compara 3. Tetraedrul ABCD se sec tioneaza si cu un plan paralel cu muchiile [AB ] [CD], AB = a, CD = b. Planul intersecteaza muchiile [BD] , [BC ] , [AD] , [AC ] n N, M, P si respectiv Q. a) Demonstra ti ca patrulaterul M N P Q este paralelogram. BM b) Daca = x, x > 0, exprima ti aria paralelogramului M N P Q n func tie de MC a, b, x si ma si CD. sura unghiului dintre AB c) Determina ti pozi tia punctului M [BC ] pentru care aria paralelogramului M N P Q este maxima .

Testul 3 (prof. Lidia BOSNCIANU )


1. Cel mai mic numa si care poate fi scris ca r natural nenul divizibil cu 88 produsul a trei numere naturale consecutive este ......................... 2. Daca si cd = 1, valoarea minima a, b, c, d R, a expresiei a2 + b2 + c2 + d2 + +2ab 2ac 2ad 2bc 2bd + 14 este ......................... 3. Numerele 5 x, 2x + 3 si 5x 2, x R, reprezinta lungimile laturilor unui triunghi. Atunci x ......................... 2 2 2 4. Ecua tiile (m 1) (x + 2)+9 = (m 1) 3 (m 1) (x + 2), (m 1) (x + 1) = = 3 (3x + m + 5), m R sunt echivalente daca m ......................... 5. Fie f : R R, f (x) = 2 |x| f (1) 3, x R. Atunci f (x) = .................... 6. Fie ABCD un dreptunghi, M A (ABC ), N mijlocul lui (BC ), M B = BD = = 6 cm si M D = 4 3 cm, atunci d (M, DN ) = ......................... 1 7. Fie 4ABC oarecare, M (BC ) astfel nct M C = BC , N AC astfel 3 1 AP nct AN = AC , iar AM BN = {P }. Atunci = ......................... 4 PM 8. ABCDA0 B 0 C 0 D0 este un paralelipiped dreptunghic cu perimetrul bazei 28 cm. Daca si formeaza diagonala paralelipipedului are lungimea de 20 cm cu o muchie 0 0 0 0 laterala un unghi cu m a sura de 30 , volumul [ ABCDA B C D ] este ......................... 9. Fie un con circular drept cu sec tiunea axiala un triunghi isoscel cu baza 8 cm 50

si perimetrul 18 cm. a) Ma tinut prin desfa surarea sura unghiului corespunza tor sectorului de cerc ob conului este ......................... b) Volumul conului este ......................... II. 1. Sa se ga seasca ultimele doua cifre ale numa rului A = 22n + 22n+1 + 22n+2 + 22n+3 42n + 42n + 42n+1 + 42n+2 + 42n+3 2n 8n

22n+1 + 22n+3 , n N . 4n + 4n+1 2. Fie x1 , x2 , . . . , xn R se demonstreze inegalitatea: + cu x1 x2 . . . xn = 1. Sa (x1 + x2 + 1) (x2 + 2 x2 + 1) . . . (xn + n xn + 1) > 3 4 n (n + 1) (n + 2) . 3. V ABCD este o piramida patrulatera regulata avnd toate muchiile de lungime a. Fie Q mijlocul lui (CV ) si (C ) cercul nscris n triunghiul V BC . a) Sa se arate ca (BDQ) (V BC ). b) Daca si N (BD), sa M este un punct oarecare pe (C ) se determine lungimea minima posibila pentru (M N ). 631 . Atunci a = ......................... 1. Fie a = 21 22 23 2100 162 a c 2. S tiind ca 6 3 + bb + 2 = 493, atunci abc = ......................... a a 13 a 3. Daca = , a, b N , atunci cea mai mare valoare pentru este .......... b b +7 b q ab + 36 / ab 36 N = ......................... 4. Mul timea A = ab N |

Testul 4 (prof. Lidia BOSNCIANU )

5. Fie f : R R, f (x 1) = 3x + 1 f (2), x R. Atunci f (x) = ............... 6. Fie M mijlocul laturii (AB ) a dreptunghiului ABCD, M N AC , N (AC ) \) = ......................... si 8M N = AC . Atunci m(BAC b )m(C b ) = 40 . Fie AD BC \, 7. Fie 4ABC cu m(B si (AE bisectoarea lui BAC \ D, E (BC ). Daca si (AE mpart BAC n trei unghiuri cu ma (AD surile direct propor tionale cu 1, 2, 3, atunci ma surile unghiurilor 4ABC sunt ......................... 8. ntr-un cilindru cu diametrul de 4 cm si na timea de 25 cm se a seaza ste l ni bile cu raza de 20 mm. a) Care este numa rul maxim de bile ce ncap n cilindru? b) Cte procente din volumul cilindrului ocupa bilele? 9. ntr-o piramida tiunea diagonala patrulatera regulata , sec este un triunghi dreptunghic isoscel. Atunci At /Al = ......................... II. 1. Fie a, b Z. Ara ti ca ta a b2 + a2 + a = 0 a = b = 0. x2 x3 . . . xn + x1 x3 . . . xn + + x1 x2 . . . xn1 2. Rezolva ti n N ecua tia y = . x1 x2 . . . xn (x1 + x2 + + xn ) 3. Fie 4ABC dreptunghic isoscel, (AB ) (AC ) si AB = a. Se duce CS (ABC ), CS = a. a) Calcula ti aria totala si volumul piramidei SABC n func tie de a. \ b) Afla ti m((SAB ) , (ABC )). c) Daca si Q este mijlocul muchiei (SA), sa CP SB se demonstreze ca patrulaterul ABP Q este inscriptibil. 51

Solu tiile problemelor propuse n nr. 1/2002


Clasele primare
P.24. Afla ti numerele a, b, c, d s si timp urma tiind ca verifica n acela toarele egalita i: a + 3 = b, b + 3 = c, c + 3 = d, a + 3 = 10. t ( Clasa I ) nv. Maria Racu, Ia si Solu tie. Din ultima rela tie afla m pe a, a = 10 3 = 7, apoi b = 7 + 3 = 10, c = 10 + 3 = 13, d = 13 + 3 = 16. P.25. Un elev din clasa I, fixnd un numa irul numerelor naturale, constata r din s ca ta lui nu este mai mica suma numerelor din fa dect 55, iar suma aceasta adunata cu numa este pe 66. Despre ce numa rul fixat nu depa s r este vorba? ( Clasa I ) Lumini ta Popa, eleva si , Ia Solu tie. Suma numerelor din fa ta numa rului ca utat poate fi 55, 56, . . . . A doua suma poate fi 66, 65, 64, . . . . Daca a doua suma nu este 66, atunci cea mai mare valoare posibila a numa rului ca utat este 65 55 = 10. Suma primelor noua numere nenule este 45, ceea ce nu corespunde datelor problemei. Deducem ca a doua suma este 66. Daca prima suma nu este 55, atunci cea mai mare valoare posibila a numa si iara si ajungem la o contradic tie. Numa rului ca utat este 66 56 = 10 rul ca utat este 66 55 = 11.

P.26. Pe trei borcane de compot, unul de cire se, altul de vi sine s i al treilea cu amestec de cire se s sine, toate etichetele au fost puse gre sit. Sco tnd un singur i vi fruct dintr-un singur borcan, determina ti con tinutul fieca ruia. ( Clasa a II-a) *** Solu tie. Se scoate un fruct din borcanul cu eticheta CV. Daca fructul este cirea sa sine sau numai , atunci n borcanul cu eticheta V nu putem avea numai vi cire se. Rezulta se si vi sine. n acest caz avem coresponden ta eticheta ca avem cire con tinut CV C, C V, V CV. Daca sina fructul extras este vi , atunci avem coresponden ta CV V, C CV, V C.

P.28. Cte pagini are o carte daca pentru paginarea ei s-a folosit cifra 9 de 117 ori? ( Clasa a III-a) Crizantema Mironeanu, eleva si , Ia Solu tie. De la pagina 1 la pagina 100 se folose ste cifra 9 de 20 ori. nseamna c a de la pagina 1 la pagina 600 se folose ste cifra 9 de 120 de ori. Pentru a folosi de 117 ori cifra 9 trebuie sa elimina m paginile: 600, 599, 598. Cartea are 597 pagini. P.29. Ioana s tine i Alina au cules mpreuna 165 de nuci. Ioana a cules mai pu nuci dect Alina; ea face un calcul s c a triplul diferen t ei dintre num a i observa rul nucilor culese de ele reprezinta tocmai num a rul nucilor culese de Alina. Cte nuci a 52

P.27. Sa tii aritmetice s i numai se scrie numa rul 31 folosind cele patru opera cifra 3 (se cer cel pu tin doua tii). solu ( Clasa a II-a) Andrea Balla, eleva sov , Bra Solu tie. 1) [(3 + 3) 3 3] (3 3 : 3) + 3 : 3 = 15 2 + 1 = 31. 2) [(3 + 3 : 3) 3 (3 3 : 3)] 3 + 3 : 3 = 10 3 + 1 = 31.

P.32. Un pa si mparte averea astfel: la primul copil 10 milioane plus o rinte cincime din rest, la al doilea copil 20 de milioane plus o cincime din noul rest, la al treilea 30 de milioane plus o cincime din noul rest s sa mai departe. Sa i a se afle suma mpa tita ti au mo steniri i numa tiind ca r de pa rinte, precum s rul copiilor, s to egale. ( Clasa a IV-a) Mihai Grtan, Ia si Solu tie. Din faptul ca primii doi copii au primit sume egale rezulta ca (R1 R2 ) : 4 ste din suma de din R1 , ceea 5 = 10, adica R1 R2 = 50. Al doilea copil prime 5 4 ce nseamna se ste pe R2 cu 20 milioane. Avem R1 R2 = 50 si ca din R1 depa 5 tinem R1 = 5 30 = 150 si 4 R1 : 5 R2 = 20 de unde rezulta R1 : 5 = 30. Ob S = 10 + 150 = 160 (milioane). Deci suma mpa tita r este de 160 milioane. Primul copil a primit 10 + 150 : 5 = 40 (milioane). Numa rul copiilor este 160 : 40 = 4.

P.31. O veveri ta si face provizii pentru i descopera un alun nca rcat cu fructe s iarna transportnd la scorbura sa alternativ: o data doua alune, o data trei alune. Dupa ce transporta 47 de alune, face o pauza pentru a se odihni. Sa se calculeze ce distan ta ta n total, daca ta a parcurs veveri de la alun la scorbura ei este o distan de x hm x dam x m, unde x are ca valoare cel mai mic numa r natural posibil. ( Clasa a IV-a) nv. Mihai Agrici, Ia si Solu tie. Numa ta de la scorbura rul x nu poate fi 0. nseamna ca x este 1. Distan la alun este de 1hm 1dam 1m= 111m. Pentru prima grupa de 5 alune parcurge traseul Alun-Scorbura -Alun-Scorbura , deci 3 111m. Pentru fiecare grupa de 5 alune, din restul de 42, parcurge traseul Scorbura -Alun-Scorbura -Alun-Scorbura , deci 4 111m. Deoarece sunt 8 grupe, veveri ta va parcurge 8 4 111m. Pentru restul de 2 alune va parcurge traseul Scorbura -Alun-Scorbura , deci 2 111m. n total veveri ta parcurge (3 + 32 + 2) 111m = 37 111m = 4107m.

P.30. Ara ti ca ta dintre oricare patru numere naturale diferite, mai mici dect 1 000 000, se pot alege doua ta a ca ror diferen sa se mparta exact la 3. ( Clasa a IV-a) Roxana Bolocan, eleva si , Ia Solu tie. La mpa tirea cu 3 resturile posibile sunt 0, 1, 2. nseamna tin r ca cel pu doua si rest la mpa tirea cu 3. Fie a = 3c + r, numere din cele patru vor da acela r b = 3d + r, a > b. Avem a b = 3c 3d = 3 (c d).

cules fiecare fata ? ( Clasa a III-a) nv. Maria Racu, Ia si Solu tie. Examinnd textul se constata ti iar Ioana doua ca Alina are trei pa r pa ti din cele cinci pa ti egale. Alina a cules 165 : 5 3 = 33 3 = 99 (nuci) iar Ioana r r a cules 165 99 = 66 (nuci).

Clasa a V-a
V.26. Sa i nenule se determine cifrele distincte s a, b, c, d, e, f, g pentru care rezultatul nmul tirii ala turate este cel mai mare posibil: Ioan Saca leanu, Hrla u Solu tie. Avem ad = e < 10 si ac = g < 10. Deoarece 53 a c e 0 0 b d f f

g g

cifrele a, b, c, . . . , g sunt distincte, rezulta ca ad, ac {1 2, 1 3, . . . , 1 9, 2 1, 2 3, 2 4, 3 1, 3 2}. Cum a 6= 1 (ca si c 6= 1, d 6= 1 (din motive ci altfel am avea d = e) similare), urmeaza ca ad, ac {2 3, 2 4, 3 2}. Ca urmare, pentru ca produsul din enun t, sa fie maxim, lua m g = 8, e = 6, a = 2, d = 3, c = 4. Au ra mas de aflat cifrele b si f . Observa si b 6= 5 m ca b, f {1, 5, 7, 9}. Dar b 6= 1 (ca ci altfel f = 3 = d) (altfel f = 5 = b). Deci b = 7 si f = 1 sau b = 9 si f = 7. n consecin ta produsul cel mai mare este 209 403 = 84227 si cifrele ca utate sunt: a = 2, b = 9, c = 4, d = 3, e = 6, f = 7, g = 8. V.27. Trei apicultori au tras mpreuna tit 700 kg miere de albine. Cnd au mpa r mierea, primul apicultor a luat juma tate, al doilea juma tate din rest, al treilea juma tate din noul rest, apoi opera tiunea se repeta pna se mparte toata mierea. Sa se afle cta miere a luat fiecare. Ca si ta lin-Cristian Budeanu, Ia Solu tie. Se observa c a primul apicultor ia de dou a ori mai mult a miere dect al doilea, iar al doilea de doua ori mai multa dect al treilea. Daca x noteaza cantitatea de miere luata de al treilea apicultor, atunci al doilea ia 2 x , iar primul 4 x si ob tinem rela tia x + 2x + 4x = 700. Rezulta c a x = 100 ; deci al treilea ia 100 kg de miere, al doilea 200 kg, iar primul 400 kg. V.28. Ara ti ca i N2 = 32002 22002 sunt numere divizibile ta N1 = 32001 + 22001 s cu 5. Dorina Carapanu, Ia si 4n 4n Solu tie. Ultima cifra a unui num a r de forma 2 este 6 , iar a unuia de forma 3 este 1. Prin urmare, 22000 = 24500 se termina n 6, iar 22001 = 22000 2 are ultima 2001 se termina si cifra tinem ca n 3. Ca urmare, N1 se termina n 5 2. La fel ob 3 este, deci, divizibil cu 5. n privin ta numa si rului N2 , observa m ca 32002 se termina n 9, 22002 n 4, iar nsu . N n 9 4 = 5. Deci, N . . 5.
2 2

V.29. Sa se afle numerele abc pentru care abc = ac b2 . Roman ta Ghi ta si Ioan Ghi ta , Blaj Solu tie. Valorile b = 0 si b = 1 nu sunt posibile deoarece condi tia din enun t se scrie a0c = 0 si respectiv a1c = ac si aceste egalita ti sunt false. Nici b = 2 nu-i o valoare posibila , ca ci 100a + 20 + c = 40a + 4c 60a + 20 = 3c, ceea ce este evident fals (3c poate fi cel mult 27). Pentru b = 3 avem: 100a + 30 + c = 90a + 9c 5a + 15 = 4c. Ultima rela tie . . implica si a = 1. Ob tinem ca tie a c . 5, adica c = 5, precum abc = 135 este o solu problemei. 2 Ara b 4. ntr-adev = b2 ta m ca nu putem r, abc = ac b avea 2 a b+ c 2100a +10 2 2 (10a + c) 10b = 10ab 100a + b c c 10b = 10 b 10 a + b 1 c 2 2 2 (1). Daca b 4 , atunci b 10 > 0 , b 1 > 0 s i din (1) rezult a c a 10 b 10 b 10 (am minorat a cu 1 si c cu 0). Constata m ca aceasta inegalitate nu-i verificata de valorile b = 4, 5, . . . , 9. Numa tie. rul abc = 135 este singura solu V.30. Daca xi , i = 1, 500 , sunt numere naturale nedivizibile cu 5, atunci 54

8 12 2000 numa rul N = 4x4 1 + 8x2 + 12x3 + + 2000x500 este divizibil cu 5. Tamara Culac, Ia si Solu tia I (a autorului). Numa rul xi , i = 1, 500, este de una din formele: M5 + 1, M5 + 2, M5 + 3, M5 + 4. Ara ta m ca x4 r avem: i = M5 + 1. ntr-adeva 4 (M5 + 1) = (M5 + 1) (M5 + 1) (M5 + 1) (M5 + 1) = M5 +1, (M5 + 2)4 = M5 +24 = = M5 + 1, (M5 + 3)4 = M5 + 34 = M5 + 1, (M5 + 4) = M5 + 44 = M5 + 1. 12 2000 Evident, avem si faptul ca sunt de forma M5 + 1. Atunci, numerele x8 i , xi , . . . , xi N = 4 (M5 + 1) + 8 (M5 + 1) + + 2000 (M5 + 1) = M5 + 4 (1 + 2 + + 500) = 501 500 = M5 + 2 500 501 = M5 . = M5 + 4 2 Solu tia II (data tescu Anca S tefania, Craiova). Avem de eleva Tu 8 4 N = 4x1 4 + 4 + 8x2 8 + 8 + + 2000 x2000 500 2000 + 2000 sau

8 2000 (1) N = 4 x4 1 1 + 8 x2 1 + + 2000 x500 1 + 4 (1 + 2 + + 500) . 501 500 Cum 4 (1 + 2 + + 500) = 4 = 2 501 500, rezulta ca acest termen 2 al numa rului N se divide cu 10. Pe de alta parte, pentru r x ce N 2 orice numa 4 { 1 , 4 , 6 , 9 } , U x nu-i divizibil cu 5 , avem: U ( x ) { 1 , 2 , 3 , 4 , 6 , 7 , 8 , 9 } , U x 8 2000 4 {1, 6}, U x8 1 , U x 1 , . . . , U x 1 { 1 , 6 } etc. Ca urmare, U x 1 2 500 8 2000 1 , U 8 x 1 , . . . , U 2000 x 1 {0, 5}; iar U 4 x4 { 0 } . 1 2 500 n consecin ta ti termenii din scrierea (1) a lui N sunt divizibili cu 10 si, deci, , to . N. . 10.

Clasa a VI-a
VI.26. Fie A = 4a + 6b c, B = 4a 3b c, C = 3a 11b 28c, unde a, b, c Z. Daca ti ca (A, B ) = 23, ara ta (A, B, C ) = 23. Cristiana Constanda, eleva si , Ia . . Solu tie. Nu trebuie sa . 23. Avem ca . 23, ara ta m, de fapt, dect ca C. A B. . . . . . . deci 9b . 23 si cum (9, 23) = 1, atunci b . 23. Din A = 4a + 6b c . 23, urmeaza acum . ca . 23, deci c = 4a 23k, k Z. n aceste condi tii, C = 3a 11b 28c = 4a c . = 3a 11b 28 (4a 23k ) = 115a 11b + 28 23k, fiecare termen fiind multiplu de 23. VI.27. Sa se rezolve n Z sistemul: 3x + 2y 8; x y 1; 3x y = 1. Mihai Cra scani ciun, Pa 7 Solu tie. Avem: y = 3x 1 8 2y 1 = 7 2y 3y 7 y , 3 y = 3x 1 3 (1 + y ) 1 = 2 + 3y 2y 2 y 1; deoarece y Z, rezulta ca y+1 y {1, 0, 1, 2}. nsa si singurele solu tii convenabile sunt (0, 1) si (1, 2). x= 3 VI.28. Sa tia se rezolve n N ecua 1 2 + 2 3 + + n (n + 1) (n + 1) (n + 2) 2n = 2 + 4 + + 2n. Dumitru - Dominic Bucescu, Ia si 55

1 [k (k + 1) (k + 2) (k 1) k (k + 1)] 3 n care da tinem prin sumare 1 2+2 3+ + n (n + 1) = m valori lui k de la 1 la n, ob 1 = n (n + 1) (n + 2). Sca tiei date suma 1 + 2 + + n, znd n ambii membri ai ecua 3 ob tinem echivalent: Solu tie. Plecnd de la identitatea k (k + 1) = n (n + 1) n (n + 1) (2n + 1) 2n (2n + 1) n (n + 1) (n + 2) 2n (2n + 1) = = , 3 2 2 6 6 n+1 altfel spus = 1, ceea ce antreneaza n = 5. 6 b VI.29. n triunghiul ascu titunghic ABC , bisectoarea interioara a unghiului B intersecteaza timea AD n E , D [BC ]. Fie F (DC astfel nct AE = EF . na l Ara ti ca ta BE AF . Tamara Culac, Ia si Solu tie. Fie E 0 [AB ] astfel nct EE 0 AB . Cum b , este egal depa E se afla pe bisectoarea lui B rtat de laturile unghiului: ED = EE 0 . Atunci 4AEE 0 4F ED \ , de unde rezulta \0 DEF (C.I.), deci AEE ca E 0 , E, F 0 sunt coliniare, adica F E AB . Urmeaza ca E este ortocentrul 4ABF , a sadar BE AF .

VI.30. Pe ipotenuza (BC ) a triunghiului dreptunghic ABC se considera i M astfel nct BN = punctele N s AB , CM = AC . Daca tiile punctelor i Q sunt proiec P s M s ti ca i N pe dreptele AN , respectiv AM , demonstra segmentele (M P ), (N Q) s i (P Q) se pot constitui n laturile unui triunghi. Ca si ta lin Calistru, Ia Solu tie. Fie {R} = M P N Q ortocentrul 4AM N ; atunci ARBC . Avem: b ) \ ) = m(BAR [ ) m(M \ m(BAM AR) = 90 m(B b) = \ \ 90 m(AM C ) = m(AM C ) m(B

b ) m(B b ) = m(M b ) = m(M \ \ \ AR). = m(M AC ) m(B AR) + 90 m(C 1 b) = 45 . Atunci \ ) = m(N \ \ Analog se arata si m(CAN AR), deci m(M AN ) = m(A ca 2 4P AM si 4QAN sunt triunghiuri isoscele, de unde M P = AP si N Q = AQ. Urmeaza ca (M P ), (N Q), (P Q) se pot constitui n laturile unui triunghi, anume 4AP Q. p a + 2 2 Q. Gheorghe Iurea, Ia si p 2 Solu tie. Fie x = a + 2 2 Q. Atunci a + 2 = 2 + 2x 2 + x si cum 1 9 a, x Q, urmeaza si 1 = 2x. De aici, x = si a = . Reciproc, daca ca a = 2 + x2 2 4 VII.26. Determina ti a Q s tiind ca 56

Clasa a VII-a

Cum dorim ca sistemul sa tii ntregi, rezulta aiba solu a = 2k, k Z. n acest caz, este imediat ca tie a sistemului dat. x1 = x2 = = xn = k constituie solu VII.28. Fie zece numere naturale nenule care au suma egala cu 55. Sa se arate ca printre ele exista trei care pot fi lungimile laturilor unui triunghi. Adrian Zanoschi, Ia si Solu tie. Sa ordona m cresca tor numerele: 1 a1 a2 a10 . Evident ca ai+1 < ai + ai+2 , i = 1, 8; ar mai trebui sa ara ta m ca exista un indice i {1, 2, . . . , 8} pentru care ai+1 > ai+2 ai . Pentru aceasta, sa presupunem contrariul: ai+1 ai+2 ai , i = 1, 8, i.e. ai+2 ai + ai+1 , i = 1, 8. Avem succesiv: a3 a1 + a2 1 + 1 = 2, a4 a2 + a3 1 + 2 = 3 si n continuare a5 5, a6 8, a7 13, a8 21, a9 34, a10 55. Atunci a1 + a2 + + a10 1+1+ +55 = 143, adica 55 143, absurd. VII.29. Fie ABCD un pa i P mijloacele segmentelor trat, O centrul sa u, iar M s (OA), respectiv (CD). Sa se arate ca triunghiul BM P este dreptunghic isoscel. Constantin Cocea si Dumitru Neagu, Ia si Solu tia I (data si). Fie de elevul Mihul Andrei, Ia M 0 , M 00 proiec tiile punctului M pe BC si respectiv CD. Avem: 1 1 0 0 00 si M M = (AD + OP ) (ca linii miM M = (AB + OO ) 2 2 si ADP O) si deci M M 0 = M M 00 . jlocii n trapezele ABO0 O Evident, M 0 B = M 00 P . Deducem ca 4 BM 0 M 4P M 00 M , 0 \ \ deci M B M P si BM M P M M 00 . Ultima rela tie conduce 0 0 M M 00 ind drept, urmeaza \ \ la BM P M M M 00 . Unghiul M\ \ ca P este unghi drept si, deci, 4BM P este dreptunghic isoscel. BM Solu tia II. Fie a latura pa tratului. Avem ca P A2 = 2 5a . Aplicnd teorema medianei n 4P AO AD2 + DP 2 = 4 10a2 10a2 , BM 2 = , deci si n 4BOA, ob tinem P M 2 = 16 16 5a2 si atunci lungimile P M = BM . Pe de alta parte, BP 2 = 4 P M , M B , P B sunt numere pitagoreice, de unde concluzia. VII.30. Fie ABCD un pa i punctele M (AD), N (BC ), trat de latura 1 s 1 4 s , demonstra ti ca {P } = BM AN . Daca i 1 < AM + BN SDCN P M = 2 3 57

p 9 2 2+1 1 a = avem ca 2 = Q. a+ 2 2= 4 2 2 VII.27. Determina ti a R astfel nct sistemul a2 a2 a2 x2 = x2 , . . . , x2 = xn , x2 = x1 1 +(a + 1) x1 + n1 +(a + 1) xn1 + n +(a + 1) xn + 4 4 4 sa tii ntregi. admita numai solu Ca si ta lin Calistru, Ia Solu tie. Adunnd membru cu membru ecua tiile, efectund reducerile si grupnd, n X a a 2 ob tinem ca xi + = 0, de unde n mod necesar x1 = x2 = = xn = . 2 2 i=1

Emil Vasile, Ploie sti Solu tie. Fie x = AM , y = BN , z = P P 0 (unde P P 0 AB ). 1 Din ipoteza , SABM +SABN SABP = , prin urmare x+y z = 2 z BP 0 P 0A xy x+y z + = 1, deci z = , 1. Dar + = x y AB AB x+y 4 unde am tinut seama de inegalitatea ntre media armonica si x+y , adica cea aritmetica . Rezulta ca x+y = 1+z 1+ 4 3 xy x + y . Pe de alta = 1 implica parte, x + y 4 x+y 4 1 4 xy = (x + y ) (x + y 1) = ; am folosit faptul evident ca x + y > 1. 3 3 9 VIII.26. Demonstra ti ca tia t2 + 1 x2 + 4t2 x + 4t2 5 = 0 are numai doua ecua solu tii n Z Z. Mihai Cra scani ciun, Pa 2 Solu tie. Ecua tia se scrie echivalent t2 (x + 2) = t x2 . Daca x = 2, atunci 2 t x2 = 0, deci t = 4. Daca x Z\ {2}, atunci t2 (x + 2) t, iar t x2 t. Egalitatea este atinsa si numai daca daca t = x = 0. n concluzie, S = {(0, 0) , (2, 4)}. tia x4 2x3 + 3x2 2x + a = 0 are o VIII.27. Determina ti a R s tiind ca ecua singura solu t ie real a . Gabriel Popa, Ia si Solu tie. Daca P ( x ) este expresia din membrul stng al ecua t iei, putem scrie: P (x) = x4 2x3 + 3x2 2x + a = x2 (1 x)2 2x (1 x) + a si de aici se observa tia are o unica tie ca P (x) = P (1 x), x R. Cum ecua solu 1 tinem n mod necesar reala x0 = . nlocuind, ob x0 , trebuie ca x0 = 1 x0 , adica 2 2 7 7 1 7 2 = 0, care admite x x+ ca . Daca , ecua tia devine x a= a= 16 16 2 4 1 singura solu tie reala (dubla ) x = . 2 VIII.28. Fie a, b numere naturale prime ntre ele. Afla ti valorile lui n pentru care Sn = an + an1 b + an2 b2 + + abn1 + bn este divizibil cu a + b. Mihaela Predescu, Pite sti Solu tie. Dac a n impar, atunci S are un num a r par de termeni s i avem c a S n= n (an + bn )+ an1 b + abn1 + = (a + b) an1 an2 b + + bn1 +ab (a + b) . an3 an4 + b) A, deci Sn . . a + b. Daca b + + bn3 + = (a n este par, n1 n2 n1 n +b atunci Sn = a a +a b + + b si cum n 1 impar, paranteza se . . . n. divide cu a + b. Rezulta ta ca aceasta divizibilitate ca Sn . a + b b . a + b; vom ara este imposibila n condi t iile date. Avem c a ( b, a + b ) = 1 , deoarece daca si d|b d | a + b, atunci d | a si d | b, adica d | ( a, b ) , deci d | 1 . Urmeaz a c a n descompunerile 58

AM BN

4 . 9

Clasa a VIII-a

lor, numerele b si a + b nu au nici un factor prim comun, afirma tie valabila si pentru numerele bn si a + b. n concluzie, pentru n par, Sn nu se divide cu a + b. VIII.29. Se considera piramida triunghiulara regulata V ABC cu latura bazei a, iar muchia laterala 2a. Fie M mijlocul lui (V A), iar N un punct pe (V B ) astfel 3a nct V N = . Afla ti distan ta de la V la planul (M N C ). 4 Adrian Corduneanu, Ia si a 6 . Folosind rela tia Solu tie. Cu teorema medianei n 4V AC , ob tinem CM = 2 a 31 . Din teorema cosinusului n 4V M N , lui Stewart n 4V BC , rezulta CN = 4 a ob tinem M N = . Cunoa stem prin urmare laturile 4CM N si cu formula lui Herron 2 10a2 15 . Calculnd acum volumul tetraedrului V M N C n afla m aria sa: SCMN = 128 a3 11 , iar cnd doua moduri, cnd folosim drept baza 4V M N ga sim ca VV MN C = 64 lua volumul fiind cunoscut, ob tinem ca ta de la V la m drept baza 4CM N , distan a 165 . planul (M N C ) este h = 25 VIII.30. Fie A, B, C, D patru puncte necoplanare astfel nct AB = 4 73, CD = 4 29. Nota m cu E, F mijloacele segmentelor (AB ), respectiv (CD). Sa se arate ca unui mijloacele segmentelor (AF ), (BF ), (CE ), (DE ) sunt vrfurile paralelogram s i sa tiind ca se calculeze aria acestuia s are o latura de lungime 194. Roman ta Ghi ta si Ioan Ghi ta , Blaj Solu tie. Fie M, N, P, Q mijloacele segmentelor (EC ), (AF ), (ED) si respectiv [BF ]. n 4ECD, (M P ) este linie mijlocie, iar (EF ) este mediana; atunci (M P ) si (EF ) se njuma tesc. Ra tionnd analog n 4F AB , deducem ca si (EF ) ta (N Q) se njuma tesc. Rezulta si N Q sunt concurente n mijlocul lui (EF ) si ta ca MP se njuma tesc, deci M, N, P, Q sunt coplanare si M N P Q este paralelogram; fie ta 1 1 O centrul acestuia. Avem ca OQ = AB = 73, OP = CD = 29. Daca 4 4 P Q = 194, aria 4OP Q se poate calcula cu formula lui Herron sau aflnd na timea; l 1 1 ob tinem SOP Q = si deci SMON = . Cum (P O) este mediana n 4P N Q, avem 2 2 1 1 ca si deci SOMQ = . n final, SM N P Q = 2. SP ON = SP OQ = 2 2 a a . Discu tie. = {x} + [x] {x} D. M. Ba tu-Giurgiu, Bucure sti tine Solu tie. Pentru existen t a numitorilor, x / [0 , 1) s i x / Z . Ecua t ia se scrie a echivalent ([x] {x}) 1 = 0, iar prima paranteza nu se poate anula. [x] {x} a si Ra mne ca [x] {x} = a, deci {x} = , unde n = [x] este din N deoarece a > 0 n IX.26. Daca tia [x] + a (0, ), sa se rezolve ecua 59

Clasa a IX-a

a , n N , n > [a]. n IX.27. Sa tiile f, g : [0, ) [0, ), unde g este surjectiva i se determine func s aditiva s i g ( y ) + g ( f ( x )) = f ( x + g ( y )) , x, y [0 , ) . Ioan Sa ca leanu, Hrla u tinem ca g ( f ( x )) = Solu tie. Fie y0 [0, ) pentru care g (y0 ) = 0; atunci ob f (x), x [0, ) (1). Prin urmare, g (y ) + f (x) = f (x + g (y )), x, y [0, ), rela tie care pentru x = 0 arata ca f (g (y )) = f (0) + g (y ), y [0, ) (2). Cum g este surjectiva , pentru orice z [0, ), ga sim y [0, ) astfel nct g (y ) = z . Din (2) deducem f (z ) = f (0) + z , z [0, ). nlocuind n (1) si folosind faptul ca g este aditiva tinem ca , ob g (z ) = z , z [0, ). {x} 0. Atunci x = n + = x + , R fixat, iar func tia g = f 1R este monotona . Mihail Bencze, Bra sov Solu tie. Aplicnd f n ambii membri ai egalita tii din enun t ob tinem ca f ( x + )= = f (x) + , x R, de unde g (x + ) = f (x + ) (x + ) = f (x) x = g (x), x R.Cum g monotona , rezulta ca g constanta : g (x) = k , x R. Atunci f (x) = x + k , x R, deci (f f f ) (x) = x + nk = x + . Urmeaza k= , | {z } n n ori adica f (x) = x + , x R. n IX.29. Sa se arate ca n orice triunghi ABC are loc inegalitatea 3 3R p l3 l3 la + b + c p (p3 3abc) ha hb hc 2r Viorel Cornea si Dan S tefan Marinescu, Hunedoara Solu tie. Din egalitatea Cauchy-Buniakowski-Schwarz, ob tinem ca X 3 2 X X 1 la 6 . (1) la ha h2 a 1 Se stie ca si atunci majora OG2 = R2 a2 + b2 + c2 0, deci a2 + b2 + c2 9R2 m 9 2 2 2 2 X 1 a +b +c 9R = . Acum al doilea factor: 2 h2 4 S 4S 2 a r 2bc A p (p a) p la = cos bc = p (p a), deci b+c 2 bc X X 3 6 (p a) = p3 3p3 3p2 (a + b + c) + 3p a2 + b2 + c2 a3 + b3 + c3 . la p3 Un calcul de rutina arata ca a3 + b3 + c3 = 3p a2 + b2 + c2 + 3abc 4p3 , de unde X 6 rezulta la p3 p3 3abc . Revenind n (1), deducem concluzia. ca IX.30. n patrulaterul ABCD considera i S pe diagonala BD, n m punctele R s interioarele triunghiurilor ABC , respectiv ACD. Notam {M } = CR AB , {N } = 60 IX.28. Sa tiile f : R R pentru care (f f f ) (x) = se determine func | {z }
n ori

= AR BC , {P } = AS CD s ca i {Q} = CS AD. Stiind +

AM n BN n CP n DQn DQ2 = 4 , s a se arate c a + + + = 4, n N. QA2 M Bn N Cn P Dn QAn Ca si ta lin Calistru, Ia Solu tie. Aplicnd teorema lui Ceva n 4ABC si 4ACD si combinnd rela tiile AM BN CP DQ ob tinute, rezulta = 1. Putem scrie: ca M B N C P D QA s BN 2 CP 2 DQ2 AM 2 BN 2 CP 2 DQ2 AM 2 + + + =44 , 2 2 2 2 MB NC PD QA M B 2 N C 2 P D2 QA2 AM BN CP DQ deci este atinsa = = = . egalitatea n inegalitatea mediilor; atunci MB NC PD QA Concluzia este acum imediata .

AM 2 BN 2 CP 2 + + + M B 2 N C 2 P D2

Clasa a X-a
X.26. Fie ecua tia x4 S1 x3 + Sx2 + mx m 1 = 0, unde S este aria unui triunghi neechilateral ABC , iar S1 este aria triunghiului A1 B1 C1 determinat de punctele de intersec tie a bisectoarelor interioare cu cercul circumscris triunghiului tia admite un numa ABC . Sa se determine mRs tiind ca ecua r impar de ra da cini n (0, 1). Dumitru Gherman, Pa scani Solu tie. Daca tia ca nota m f (x) = x4 S1 x3 + Sx2 + mx m 1, atunci condi ecua tia data sa admita un numa r impar de ra da cini n (0, 1) este echivalenta cu f (0) f (1) < 0, adica tei (m 1) (S S1 ) < 0 (). Sa determina m acum semnul diferen A+B A+C B+C 2 2 si S1 = 2R sin S S1 . Avem S = 2R sin A sin B sin C sin sin = 2 2 2 A A B C B C = 2R2 cos cos cos , de unde rezulta ca SS1 = 8 sin sin sin 1, cu egali2 2 2 2 2 2 tate daca si numai daca tiile problemei, triunghiul ABC este echilateral. Deci, n condi si atunci, avnd n vedere rela tia (), ob tinem m (, 1). are loc S < S1 X.27. Fie r [1, ), D = {z C; |z | r} s i P C [X ], P (X ) = aX 2 + +bX + c. Sa se arate ca daca P (z ) D, z D, atunci a, b, c D. D.M. Ba tu-Giurgiu, sti tine Bucure 2 Solu tie. Fie 1, si 2 ra tiei x3 = 1. Deoarece 1 , , D , rezult a da cinile ecua 2 2 = a + b D s i P + c D . ca P (1) = a + b + c D, P () = a2 + b +c 2 2 2 , 3 a = P (1) + P ( ) + De aici, ob tinem ca + P ( ) + P P 3c = P (1) egalita ti deducem ca () + P 2 . Folosind aceste si 3b = P (1) + 2 P : 3 |c| = 2 2 = P (1) + P () + P |P (1)| + |P ()| + P 3r si, analog, 3 |a| 3r si 3 |b| 3r. Deci, a, b, c D. 2 X.28. Rezolva ti ecua tia z 2 2|z| 1 + z 2|z1| 1 + 1 = 0, z C\R. Emil Vasile, Ploie sti |z |2 |z 1| 1 = R s i 2 1 = R. Solu tie. Fie z = a + bi (a, b R, b 6= 0 ), 2 2 2 + 2 abi + ( a + bi ) + 1 = 0 sau Cu aceste nota t ii ecua t ia noastr a devine: a b 2 tinem a2 b2 + 1 = 0, adica 2ab + b = a b2 + a + 1 = 0, de unde ob 2 a2 +b2 2 2 1 a + b = 1 (). 61

Deoarece func tia f (t) = t (2t 1) este strict cresca toare pe (0, ) rezulta ca c a = 1 ecua tia () are o solu tie unica si anume a2 + b2 = 1. De aici deducem si nlocuind n egalitatea 2a + = 0, ob tinem = 2a, adica 2 22a = 1 2a. Dac a tia c = 1 2a, cum |a| 1, rezult a facem nota c 3. Pe de alta parte din 2 1+c = c 0, ob tinem c 2, deci c = 2 1+c 2 3 > 25/3 > 3. Contradic tia la care am ajuns arata tia data tie. ca ecua nu are solu X.29. Un motan scoate cu ajutorul unui pahar un numa sti sori dintrr de pe un acvariu. C ti pe sti sori trebuie sa tina con acvariul astfel nct motanul sa aiba matematic speran ta ca va scoate 5 dintre ei? Gabriel Popa, Ia si Solu tie. Nota sti sori din acvariu. Fie X variabila aleatoare m cu n numa rul de pe care ia ca valori numa sti sori care se afla rul de pe n paharul motanului. Sa calcula m pk = P ({X = k}). Deoarece sunt n total 2n cazuri egal posibile (numa rul k submul timilor unei mul timi cu n elemente), dintre care sunt favorabile Cn , avem k Cn . A sadar, tabloul de reparti tie al variabilei aleatoare X este: pk = n 2 0 1 2 ... n 0 1 2 n. X : Cn Cn Cn Cn . . . 2n 2n 2n 2n Motanul poate spera ca sti sori egal cu speran ta matematica va extrage un numa r de pe (sau media) variabilei X , adica n n k X X kCn n2n1 n E (x) = m = pk xk = = = . n n 2 2 2 k=1 k=1 Prin urmare, avem n/2 = 5, deci n = 10. X.30. Fie M = {1, 2, . . . , n}. Sa k-uple se afle numa rul de k ! (A1 , A2 , . . . , Ak ) de k [ \ Ai = M s Ai = l, l n fixat. submul timi ale lui M astfel nct i Card
i=1 i=1

= (1, 1, . . . , 1)}) = l. Cum numa rul de moduri n care l elemente din M sunt duse k nl l l prin f n (1, 1, . . . , 1) este Cn 2 2 , rezulta . ca ra spunsul problemei este Cn 62

/ Ai (de si ji = 0 daca = (j1 , j2 , . . . , jk ) N, unde ji = 1 daca j Ai j exemplu, f (3) = (1, 1, 0, . . . , 0, 1) daca si numai dac a 3 A , 3 A , 3 Ak si 1 2 n tii este 2k 1 , dar nu toate satisfac 3 / A3 , A4 , . . . , Ak1 ). Numa rul acestor func ultima condi tie din ipoteza . Observa si numai daca Ak daca m ca j A1 f (j ) = (1, 1, . . . , 1) 2 A ! k \ N . Deci, condi tia Card Ai = l este echivalenta cu Card ({j M | f (j ) =
i=1

Lucian-Georges La si dunca , Ia Solu tie. Fie N = {(0, 0, . . . 0, 1), (0, 0, . . . , 1, 0), . . . , (1, 1, . . . , 1, 1)} Rk mul timea k -uplelor formate din 0 si 1, fa timea N are ra elementul (0, 0, . . . , 0, 0). Mul 2k 1 elemente. A parti tiona M = {1, 2, . . . , n} n k submul timi (A1 , A2 , . . . , Ak ) astfel nct k [ Ai = M este totuna cu a defini o func tie f : M N prin legea f (j ) =
i=1

Clasa a XI-a
XI.26. Fie A Mn (R). Daca tr (tA A + (tA) A ) = 2n det A, atunci tA = A . Iuliana Georgescu si Paul Georgescu, Ia si Solu tie. Se stie ca tr (X + Y ) = tr X + tr Y ( R), tr (XY ) = tr (Y X ), XX = (det X ) In si (tX ) = t (X ), X, Y Mn (R). Egalitatea data se rescrie astfel: tr tA A + tA A = tr A A + tA tA t tr A A + A t (A ) = tr A A + tA t (A ) tr A tA A + A tA t (A ) = 0 tr A t (A ) tA A = 0,

tinem tr (C t C ) = 0. Ultima rela tie, mpreuna de unde, notnd cu C = A t (A ), ob cu observa tia C Mn (R), ne conduce la condi tia C = On , adica tA = A . XI.27. Fie a [0, 1) s ir de numere reale astfel nct i (xn ) n0 un s 1 2 2 2 x , n N . , + x x2 a max x n n1 n1 2 n Ara ti ca ti limita sa. irul (xn ) n0 este convergent s i determina ta s Aurel Muntean, Sibiu 1 2 2 x2 + x2 Solu tie. Daca = x2 max x2 n1 , n1 n1 , atunci xn axn1 , sau 2 n 1 2 2 , + x x = |xn | k1 |xn1 |, unde k1 = a [0, 1). Daca max x2 n1 n1 2 n a 2 a 2 a 1 2 2 xn x2 xn + x2 xn + x2 , adica = x2 n1 , avem xn n1 sau 1 n 2 2 2 2 n1 r a a si deci |xn | k2 |xn1 |, unde k2 = x2 [0, 1). 2 a n1 2a Fie k = max {k1 , k2 }. Deoarece k [0, 1) si |xn | k |xn1 | k 2 |xn2 | n k |x0 |, rezulta sirul (xn )n0 converge la zero. ca XI.28. Sa irului (an )n1 definit prin se determine p R pentru care limita s n p X n p este finita termenul general an = i nenula s . k + k2 1 k=1 Constantin Chirila si , Ia Solu tie. Avem succesiv: n n X X np p 2 p n k + 1 k = = an = 2 k + k2 1 k=1 k=1 n np 2 np 2 X = k+1 k = n+11 = 2 2 k=1 ! r 1 2 p+ 1 1 = 1+ . n 2 2 n n 1 si numai daca Deci, sirul (an ) are limita finita nenula daca p= . 2 63

! 1 1 1 + + + 1 = 1. 2 n Marian Tetiva, Brlad n (un 1) ln an , unde Solu tie. Scriem termenul general xn sub forma: xn = ln an ln bn r 1 1 1 1 un = n 1 + + + , an = 1 + + + tia 1 < si bn = ln n. Din rela 2 n 2 n r 1 1 < n 1 + + + < n n, n 2 deducem ca lim un = 1. Atunci, avem: n 2 n n XI.29. Sa se arate ca lim n ln ln n
n

n (un 1) un 1 un 1 = lim = lim = 1. (1) n ln un n ln [1 + (un 1)] ln an an = 1. n consecin ta Folosind criteriul lui Stolz-Cesro, ga , sim ca lim n bn ln (an /bn ) ln an = lim + 1 = 1. De aici si din (1), ob tinem ca lim lim xn = 1. n ln bn n n ln bn XI.30. Fie f : R R o func tie discontinua s i care are proprietatea lui Darboux. Daca exist a o func t ie g : R R R astfel nct f ( x + y ) = g ( f ( x ) , y ) , pentru orice x, y R, atunci func tia f nu are limita la . S tefan Alexe, Pite sti Solu tie. Daca f ar fi injectiv a , cum f are proprietatea lui Darboux, ar nsemna ca si atunci exista f este continua , ceea ce contrazice ipoteza. Deci f nu este injectiva a, b R, a < b astfel nct f (a) = f (b). A sadar, avem f (a + x) = g (f (a) , x) = = g (f (b) , x) = f (b + x), x R, de unde rezulta ca f (x) = f (x + b a), x R, adica f este periodic a s i T = b a este o perioad a a ei. Cum f este discontinua rezulta c a f nu este constant a s i deci exist a , R , 6= astfel nct f () 6= f ( ). Considernd sirurile xn = + nT si yn = + nT , n N, care tind la +, avem f (xn ) = f ( + nT ) = f () f () si f (yn ) = f ( + nT ) = f ( ) f ( ), ceea ce demonstreaza ca f nu are limita la +.
n

lim

a2 a2 1 a2 2 2 ; a A , XII.26. Se considera 1 + a 2 a a multimea M = 1 a 2 a 2 unde ti ca (M, ) este grup; este acesta izomorf A= Z sau A = Q sau A = R. Arata cu A +, ? Gheorghe Costovici, Ia si Solu tie. Nota timii M . Deoarece m cu M (a), a A, un element oarecare al mul M (a) M (b) = M (a + b) (), a, b A, rezulta tirea este lege de compozi ca nmul tie interna tia () si avnd n vedere ca pe M . Folosindu-ne de rela adunarea este asociativa si comutativa tirea este asociativa si comutativa pe A rezulta ca nmul pe M . Mai mult, se observa c a M (0) este element neutru pentru nmul t irea pe M si orice M (a) M admite un simetric, si anume, M (a) M . Prin urmare, (M, ) a este grup comutativ. n fine, se verifica sor u func tia f : M A+ , f (M (a)) = e ca . este un izomorfism ntre grupurile (M, ) si A , + XII.27. Fie (G, ) un grup cu Z (G) 6= {e} s i H un subgrup netrivial al lui G. 64

Clasa a XII-a

si se calculeaza sor valoarea lui I2 . u Pentru n = 4, proceda m la fel: I4 J4 Z

Sa i yx H . se demonstreze ca exista x, y G\H , x 6= y 1 , astfel nct xy H s Da ti exemplu de grup care nu are aceasta proprietate. Ovidiu Munteanu, student, Bra sov Solu tie. Oricare ar fi x G\H si oricare ar fi u H \{e}, avem y = x1 u G\H (ntr-adeva si deci x = uy 1 H , ceea ce r, daca y = x1 u H , atunci y 1 H este fals). De aici, deducem ca xy = u H . Deoarece yx = x1 ux, trebuie sa mai demonstra si u H \{e} astfel nct x1 ux H . Cum m ca exista x G\H Z (G) = {a G | ab = ba, b G} 6= {e}, rezulta si u H \{e} ca exista x G\H astfel nct xu = ux, deci x1 ux = u H . Cu aceasta prima parte a demonstra tiei este ncheiata . Pentru a doua parte, considera si H = {e, }. Se m S3 = e, , , 2 , , 2 observa ca nu exista x, y G\H astfel nct xy = yx = . ntr-adeva r, daca , si prin absurd, ar exista x, y G\H astfel nct xy = yx = , atunci y = x1 x1 x = , deci x = x. Cum nu comuta cu nici un element din G\H , nseamna ca falsa . ultima egalitate este Z n , pentru n {2, 3, 4}. tg xdx, x 0, XII.28. Calcula ti 2 Daniel Jinga, Pite sti Z Z n n Solu tie. Notnd In = tg xdx, Jn = ctg xdx si efectund schimbarea Z Z n n 2 t t n tg x = t, ob tinem: In = n dt, Jn = n dt. 2 n 1+t 1 + t2n Pentru n = 3, Z Z Z 1 u 1 u+1 t3 t2 =u 3 du etc. dt = du = I3 = 3 + 1 + t6 2 1 + u3 2 u + 1 u2 u + 1 Pentru n = 2, considera m 0 1 1 Z Z 2 Z t2 1 2 t t 1 t t dt = 2 I2 J2 = 2 dt = 2 dt. 2 1 1 + t4 1 t2 t2 + 2 t 2 t t Prin urmare, tg x + ctg x 2 1 tg x ctg x +C + C, I2 J2 = ln I2 + J2 = 2 arctg 2 2 2 tg x + ctg x + 2 Z 1 1 t2 dt = = 4 1 t4 + 4 t 0 1 Z Z t du t = 4 dt = 4 etc. 4 2 4 u 4u2 + 2 1 1 t +2 4 t t t t4 t2 dt = 4 1 + t8 65

n t t X k t (k 1) b a+b a t . k f n (1 k ) nt k=1 Z 1 Cum f : [0, 1] R este continua f (x) dx. , rezulta ca lim n (f, ) = n 0 Z t h i 2 ln (1 + x) e XII.30. Sa ex dx ln 2 , ln 2 . se arate ca 8 16 (1 + x2 )2 0 Cristian Moan ta , Craiova 2 ex . Solu tie. Nota m cu I integrala din enun t s i fie f : [0 , 1] R , f ( x ) = 1 + x2 Deoarece ex 1 + x2 , x R, avem ca f (x) 1; deoarece f este cresca toare pe [0, 1] 3 x2 2 x e e (ca 0, x [0, 1]), rezulta ci f 0 (x) = ca f (x) f (1) = . Ca urmare 2 (1 + x2 )2 Z 1 Z 1 e ln (1 + x) ln (1 + x) dx I dx 2 1 + x 2 1 + x2 0 0 Z 1 ln (1 + x) dx = ln 2. si ra mne de ara tat ca J= 2 1+x 8 0 ntr-adeva si t = u, vom avea r, cu schimbara rile x = tg t 4 Z 0 Z /4 ln (1 + tg t) dt = ln 1 + tg u du = J = 4 0 /4 Z /4 Z /4 2 ln ln 2du J, du = = 1 + tg u o 0 de unde J = ln 2. 8

XII.29. Fie f : R R o func tie continua i t > 0. Pentru a, b > 0, sa s se calculeze n t X k t (k 1)t b a+b 1 a . k lim f n ( k 1) n nt k=1 Mihail Bencze, Bra sov t t 0 1 nt , , . . . , t o diviziune a intervalului [0, 1]. Atunci Solu tie. Fie n = nt nt n k t (k 1)t nt (n 1)t = max 0 pentru n . Lua kn k = m drept nt nt k{0,1,...,n} puncte intermediare media geometrica ponderata a punctelor de diviziune, adica 1 a+ ! b b t a t t k (k 1) b a+b a t = n ( k 1) , k = 1, n. k k = t n nt Atunci n (f, ) =

66

Solu tiile problemelor pentru prega tirea concursurilor din nr. 1/2002
A. Nivel gimnazial
G6. Daca un numa r natural se poate scrie ca suma a doua pa trate perfecte nenule distincte, atunci orice putere a sa se poate scrie, de asemenea, ca suma de doua pa trate perfecte nenule. *** Solu tie. Fie a = b2 + c2 , cu b, c N , iar n N . Daca n = 2k + 1, atunci 2 2 an = a2k a = a2k b2 + c2 = ak b + ak c , cu ak b, ak c N . Daca n = 2k , vom 2 4 demonstra mai nti afirma t ia pentru k = 1 . ntr-adev a r, a = b + c4 + 2b2 c2 = 2 2 2 2 2 2 = b c + (2bc) , cu B = b c , C = 2bc N . Pentru k 2, avem ca 2 k1 2 k1 2 n 2k2 2 2(k1) 2 k1 k1 B +C = a a =a a =a B + a C , cu a B, a CN . G7. Ara ti ca trat perfect, oricare ta numa rul aa . . . a (2001 cifre) nu poate fi pa ar fi cifra a n baza 10. *** Solu tie. Afirma tia este adeva rata n cazul general al unui numa r aa . . . a cu n 2 cifre. Numerele 22 . . . 2, 33 . . . 3, 77 . . . 7 si 88 . . . 8 nu pot fi pa trate perfecte din cauza ultimei cifre. Cum orice pa trat perfect este fie de forma 4k , fie de forma 4k + 1, k N, nu pot fi pa si 99 . . . 9. trate perfecte numerele 11 . . . 1, 55 . . . 5, 66 . . . 6 n sfr sit, daca 44 . . . 4 = 4 11 . . . 1 ar fi pa trat perfect, atunci 11 . . . 1 ar fi pa trat perfect, absurd. 3n (18n + 13) 28 este frac tie reductibila G8. Determina ti n Z pentru care . 3n + 1 Dumitru - Dominic Bucescu, Ia si Solu tie. Deoarece 3n (18n + 13) 28 = (3n + 1) (18n + 7) 35, frac tia data se simplifica tinem prin d N\ {0, 1} daca d este un divizor al lui 35. Pentru d = 5, ob ca tie diofantica tia particulara si cea 3n + 1 = 5l, l Z, ecua cu solu l = 1, n = 2 generala l = 1+3k , n = 2+5k , k Z. Pentru d = 7, ga sim l = 3p +1, n = 7p +2, p Z. n concluzie, valorile ca utate ale lui n sunt {5k 2 | k Z} {7p + 2 | p Z}. G9. Se dau trei fi sicuri de monede a sezate vertical, asupra ca rora putem efectua una dintre opera tiile O1 : lua sic s seza i le a m cele doua monede de deasupra unui fi m peste altul, sau O2 : lua sic s seza i le a m cele doua monede de deasupra unui fi m cte una peste fiecare dintre celelalte doua sicuri. fi a) Ga ti o condi tie necesara tii, toate fi sicurile si pentru ca, dupa un numa r de opera sa con t in a la fel de multe monede; b) Ara ti ca tie nu este suficienta ta aceasta condi daca este permisa o singura opera tie, nsa este suficient a n cazul n care sunt permise amndou a . Gabriel Popa, Ia si Solu tie. Deoarece numa rul total de monede r a mne constant pe parcursul efec tua tiilor, acest numa rii opera r trebuie sa fie n mod necesar un multiplu de 3 mai mare sau egal cu 6. Presupunnd ca tia ini tiala tiile n care distribu a monedelor este (3, 2, 1), n condi este permisa o singur a opera t ie, se arat a c a egalizarea celor trei fi s icuri nu este posibila considernd toate mi sca rile ce pot fi efectuate. n cazul n care ambele opera tii sunt 67

permise, a seznd n mod repetat cte doua sicul cel mai nalt peste monede din fi cel mai mic, ajungem fie ca fi sicurile sa se egalizeze, fie ca n vrfurile lor sa apara O2 O1 o situa tie de tipul (3, 2, 1). n aceasta tie, succesiunea (3, 2, 1) (4, 0, 2) situa (2, 2, 2) rezolva problema. G10. Pentru n N, n 2, rezolva ti ecua tia r r r n+1 n+1 n+1 x1 + x2 + . . . xn + x1 + x2 + + xn = n + 1. n n n Mihai Totolici, Gala ti r n+1 Solu tie. Cu nota tiile ui = xi , x = 1, n, un+1 = x1 + x2 + + xn n 2 ob tinem ca + + u2 u1 + u2 + + un+1 = n + 1, iar u2 1 + u2 n+1 = n + 1. De aici, 2 2 2 2 (u1 + u2 + + un+1 ) = (n + 1) u1 + u2 + + un , deci este atinsa egalitatea n inegalitatea Cauchy-Buniakowski-Schwarz aplicata numerelor u1 , u2 , . . . , un+1 ; 1, 1 1, . . . , 1. Urmeaza ca u1 = u2 = = un+1 = 1, de unde x1 = x2 = = xn = . n tia x2 + y 2 = 5445. G11. Rezolva ti n N N ecua Daniela Iosub, eleva si , Ia Solu tie. Vom folosi urma torul rezultat din teoria numerelor: daca p = 4k +3 este un numa i p | a2 + b2 , a, b N , atunci p | a s i p | b. Din ipoteza , 3 | x2 + y 2 r prim s 2 2 si 11 | x + y , x, y N , iar 3 sau 11 sunt numere prime de forma 4k + 3. Prin urmare, 33 | x si 33 | y , deci x = 33l, y = 33m, l, m N . nlocuind n ecua tie, 2 ob tinem ca l + m2 = 5, l, m N , adica (l, m) {(1, 2) , (2, 1)}. De aici, (x, y ) {(33, 66) , (66, 33)}. G12. Sa 1 + 2 + . . . [ n] = nm . se determine n, m N pentru care Adrian Zanoschi, Ia si Solu tie. Pentru n = 1, m N rela tia data se verific a . C a ut a m solu t ii cu n 2 . Nu putem avea m 2, deoarece h i h i 1 + 2 + . . . n 1 + 2 + + n < n n < n2 nm . Ra si n = 3 dau solu tii ale ecua tiei, mne de cercetat cazul m = 1; se observa ca n = 2 iar pentru n 4 ob tinem h i h i 1 + 2 + . . . n 1 + 1 + 1 + 4 + + n > 3 + 2 (n 3) = 2n 3 > n, si numa G13. Ara ti ca i 2n + 18n , n N, au acela r de cifre. ta numerele 18n s Gheorghe Iurea, Ia si Solu tie. Sa presupunem prin reducere la absurd ca 18n are k cifre, iar 2n + 18n are mai mult de k cifre, deci 2n + 18n 10k > 18n 10k1 . Evident, k > n si atunci mpa tind prin 2n aceasta tinem: r inegalitate, ob deoarece 2 5 N. Pe de alta parte, 1 + 9 = 1 + (8 + 1) = M 4 + 2, deci 2kn 5k = M 4 + 2, de unde k n = 1; rela tia (1) devine 2 5n+1 = 1 + 9n (2). 68
kn k

adica tii. n concluzie, (n, m) {(1, a) | a N } {(2, 1) , (3, 1)}. nu mai ga sim solu

1 + 9n 5k 2kn > 9n 2kn 5k = 1 + 9n ,


n

(1)
n

tia adica pentru n 4. Contradic 9n > 10 5n = 2 5n+1 , deci (2) nu este verificata ob tinuta tia. ncheie demonstra G14. Sa se arate ca nu exista nici un triunghi dreptunghic avnd catetele numere ra tionale, iar ipotenuza egala cu 2001. Constantin Cocea, Ia si x2 z2 Solu tie. Pentru a ara tia 2 + 2 = 2001 nu are solu tii n N , este ta ca ecua y t suficient sa tia m2 + n2 = 2001p2 (1) nu are solu tii n N . demonstra m ca ecua Folosind rezultatul amintit n solu tia problemei G11 si observnd ca 3 | 2001 , ob tinem ca n mod necesar m s i n sunt multipli de 3 ; m = 3 m , n = 3 n , m , n . Ecua tia N 1 1 1 1 2 2 = 667 p (1) devine 3 m2 + n , p . s i cum (3 , 667) = 1 , urmeaz a c a p = 3 p N 1 1 1 1 2 2 2 Dupa nlocuire, m + n = 2001 p (2) . 1 1 1 Daca tia (1) admite solu tii, fie o asemenea solu tie cu p minim. presupunem ca ecua Din (2) se ob tine nsa tie cu p1 < p, contradic tie! Urmeaza o noua solu ca (1) nu are solu tii n N , de unde concluzia. Nota ste metoda coborrii si a fost utilizata . Metoda folosita se nume n demonstrarea Marii Teoreme a lui Fermat n cazurile n = 3 si n = 4. G15. Sa se arate ca E (x, y, z ) 3, daca E (x, y, z ) = x2 2x sin z 4 cos z + 5+ p + y 2 2y sin z 6 cos z + 10, x, y, z R. Cristiana Artenie, eleva si , Ia Solu tie. Se observa ca q q E (x, y, z ) = (x sin z )2 + (2 cos z )2 + (y sin z )2 + (3 cos z )2 = M P +M Q, unde M (cos z, sin z ), P (2, x), Q (3, y ), x, y, z R. Punctul M parcurge cercul unitate C , iar punctele P si Q parcurg dreptele verticale d1 : x = 2, respectiv d2 : x = 3. Minimul lui E (x, y, z ) se atinge pentru {M } = C [Ox, {P } = d1 Ox, {Q} = d2 Ox; n acest caz E (x, y, z ) = 3, de unde concluzia. G16. Fie M un punct n interiorul triunghiului echilateral ABC astfel nct \ M A2 = M B 2+M C 2 2M B M C ; calcula ti ma C . Generalizare. sura unghiului BM Corneliu Bra teanu, Pa scani da Solu tie. n general, vom ara ta ca daca M A2 = \ = M B 2 + M C 2 2M B M C cos , atunci m(BM C) = tia problemei date, va rezulta ca = + 60 . n situa \ m(BM C ) = 105 . Fie D n semiplanul determinat de BC opus lui A ast\) = fel nct 4M BD este echilateral. Atunci m(ABM \ ) = 60 m(M \ = m(CBD BC ), de unde 4ABM 4CBD (LUL), deci AM = DC . Cum M D = M B , rela tia de mai sus se scrie CD2 = M D2 + M C 2 2M D \ \ M C cos , ceea ce arata C ) = , adica C ) = 60 + . ca m(DM m(BM 69

Numerele n = 0, 1, 2, 3 nu verifica (2), iar pentru n 4 avem ca n 9 = (1, 8)n (1, 8)4 = (3, 24)2 > (3, 2)2 = 10, 24 > 10, 5

G17. Fie ABCD un patrulater convex ce nu are diagonalele perpendiculare, B1 s i D1 proiec tiile punctelor B , respectiv D pe AC , iar A1 s tiile punctelor A, i C1 proiec 2 BD SBB1 DD1 2 respectiv C pe BD. Sa se arate c a s = cos2 (BD, AC ) = i SABCD SCC1 AA1 AC = SBB1 DD1 SCC1 AA1 . Claudiu- Stefan Popa, Ia si Solu tie. Avem ca SABCD = SACD + SACB = AC BB1 BB1 + DD1 AC DD1 + = AC . Pe = 2 2 2 DD este trapez sau paralelode alta parte, BB 1 1 gram (BB1 , DD1 B1 D1 ) cu na timea [B1 D1 ], l BB1 + DD1 deci SBB1 DD1 = Atunci B1 D1 . 2 B1 D1 SBB1 DD1 = . SABCD AC B1 O BO B1 D1 BD Observa si de aici = , adica = , m ca 4DOD1 4BOB1 D O DO D O DO 1 1 SBB1 DD1 D1 O BD \ BD). Rezulta deci B1 D1 = BD = = BD cos(AC, ca DO S AC ABCD SCC1 AA1 AC \ \ tine ca tind, BD). Analog se ob BD). mpa cos(AC, = cos(AC, r SABCD BD apoi nmul tind membru cu membru ultimele doua ti, ga tiile din con egalita sim rela cluzie. b) 90 . Pe latura (BC ) se considera G18. Fie ABC un triunghi cu m(A punctele M s ta i N astfel nct AM s i AN sa fie simetrice fa de bisectoarea unghiului A. Cercul circumscris triunghiului AM N intersecteaza i AC n E , laturile AB s BC respectiv F . Daca ti ca . i {P } = AI BC , demonstra {I } = BF CE s AP 2 Florin Nicolaescu, Bal s \ \ N AF , deci n Solu tie. Din ipoteza , EAM cercul C avem ca EM F N , de unde EF kM N . Fie {D} = AP EF ; atunci 4AED 4ABP si ED AD 4AF D 4ACP si va rezulta ca = = BP AP DF ED BP = , i.e. = (1). Din asema na rile PC FD CP 4EID 4CIP si 4DIF 4P IB ob tinem, ca CP ED = (2). Din (1) si (2) urmeaza mai sus, FD BP ca (BP ) (CP ). BC Presupunem prin reducere la absurd ca , adica si AP < P C . AP < AP < BP 2 b) b ), deci m(\ b )+ \) > m(B [ [ Atunci m(BAP si m(P AC ) > m(C BAP ) + m(P AC ) > m(B b), i.e. m(A b) > 90 , ceea ce contrazice ipoteza; b ), de unde m(A b) > 180 m(A +m(C problema este astfel rezolvata . G19. Fie A1 A2 A3 un triunghi echilateral nscris n cercul C (O, R) s i cercurile 70

= 6R2 6Rr = constant, unde (1) se justifica prin aplicarea teoremei lui Pitagora tia n triunghiurile dreptunghice P Pi Oi , iar (2) prin rela lui Leibniz. G20. Sa se arate ca pentru orice alegere a 12 numere naturale consecutive nu se pot numerota muchiile unui cub astfel ca suma numerelor aflate pe trei muchii care au un vrf comun sa si pentru toate vrfurile cubului (nu se numeroteaza fie aceea doua si numa muchii cu acela r). Sa se arate ca este posibila numerotarea descrisa daca se aleg convenabil 12 numere dintre oricare 13 numere naturale consecutive. Constantin Chirila si , Ia Solu tie. Fie n + 1, n + 2, . . . , n + 12, n N si sa presupunem prin absurd ca suma numerelor de pe oricare trei muchii adunate adiacente este s. Ob tinem ca 8s = = 2[(n + 1) + (n + 2) + + (n + 12)], de unde, dupa calcule, ga sim 2s = 3 (2n + 13). Am ajuns evident la o contradic tie, deoarece n stnga avem un numa r par, iar n dreapta unul impar. Pentru partea a doua, fa ra a restrnge generalitatea, putem considera numerele 1, 2, . . . , 13; cazul general se reduce imediat la acesta. Fie c numa rul pe care l vom elimina. Cu ra tionamentul de mai sus, ob tinem 4s = 91 c 91 c si cum s = N, n mod necesar c {3, 7, 11}, 4 deci s {22, 21, 20}. Pentru c = 7, vom da o a sezare a numerelor 1, 2, 3, 4, 5, 6, 8, 9, 10, 11, 12, 13 care sa respecte cerin tele problemei: fiecare pereche de numere simetrice fa ta ta de 7 de forma (p, 14 p) se scriu pe muchii simetrice fa de centrul cubului, astfel nct suma ntr-un vrf sa fie 21.

si raza Ci (i = 1, 2, 3) de aceea r, tangente interior cercului C n vrfurile Ai cores2 2 punza tia t2 toare. Sa se arate ca pentru orice P C (O, R) are loc rela 1 + t2 + t3 = constant, unde ti (i = 1, 2, 3) este lungimea tangentei dusa din P la cercul Ci . Temistocle Brsan, Ia si Solu tie. Fie Oi centrul cercului Ci , i = 1, 3. Evident ca 4O1 O2 O3 este echilateral, iar centrul sa u este punctul O. Avem: X X X (1) X 2 2 (2) P Oi t2 P Pi2 = P Oi = r2 = 3r2 + i = X 2 2 = 3r2 + 3P O2 + = 3r2 +3R2 +3 (R r) = OOi

B. Nivel liceal
L6. Fie x1 , x2 , . . . , xn , n N\{0, 1}, numere reale cu proprietatea x1 x2 xn + + + = 1, S x1 S x2 S xn P ti ca unde S = n ta i=1 xi . Ara 3 x3 x3 x1 S2 2 n + + + . S x1 S x2 S xn n Ra zvan Ba rbulescu, elev, Craiova 71

Solu tie. Din rela tia data n ipoteza , deducem succesiv: x2 xn Sx1 Sx2 Sxn x1 + + + =1 + + + =S S x1 S x2 S xn S x1 S x2 S xn x2 (S x2 ) + x2 xn (S xn ) + x2 x1 (S x1 ) + x2 1 2 n + + + =S S x1 S x2 S x2 n x2 x2 x2 1 2 n + x2 + + + xn + =S x1 + S x1 S x2 S xn x2 x2 x2 n 1 2 + + + =0 S x1 S x2 S xn x3 x3 x3 1 2 n + x2 + + x2 =0 x2 1+ 2+ n+ S x1 S x2 S xn x3 x3 x3 1 2 n 2 2 + + + = x2 1 + x2 + + xn . S x1 S x2 S xn r 2 2 x1 + x2 + + xn x2 1 + x2 + + xn , de unde rezulta nsa imediat concluzia. n n

b) > 60 , considera L7. n triunghiul ABC , m(A i bisecm medianele CN , BN 0 s 0 toarele BE , CE . Nota ti ca m {P } = CN BE , {Q} = CE 0 BN 0 . Ara ta punctele P s timea din A. i Q nu pot fi ambele pe na l Ioan Sa ca leanu, Hrla u Solu tie. Sa si Q apar tin na timii (AD). Aplicnd presupunem prin absurd ca P l teorema lui Menelaus n 4ABD cu transversala N P C , apoi teorema bisectoarei n acela si triunghi, ob tinem AN BC DP BC BD a c cos B a cos C =1 =1 =1 = . N B CD P A CD BA b cos C c b cos B a cos C a b Repetnd ra tionamentul n 4ACD, ob tinem ca = , deci = , adica c cos B c a 2 2 2 2 2 2 a = bc. nsa a = b + c 2bc cos A, de unde b + c bc (1 + 2 cos A) = 0, prin 2 b b urmare si cum = 4 cos2 A + 4 cos A 3 < 0 pentru (1 + 2 cos A) + 1 = 0 c c b 1 , deducem ca cos A 1, / R, absurd. 2 c Nota tie corecta tariu, elev, Ia si. . Solu s-a primit de la Marius Pachi L8. Fie triunghiul ABC s i M Int ABC , M A C (M BC ) = {M, A1 }, M B C (M CA) = {M, B1 }, M C C (M AB ) = {M, C1 }. Sa se arate ca M A1 M B1 M C1 + + 6. MA MB MC Neculai Roman, Mirce sti, Ia si \ \ \ B1 ), z = m(AM C1 ); evident ca x + Solu tie. Fie x = m(BM A1 ), y = m(CM +y + z = 180 . Daca R1 este raza cercului prin M, B, C , avem: A1 B = 2R1 sin x, A1 C = 2R1 sin z , BC = 2R1 sin (x + z ) = 2R1 sin y . Aplicnd teorema lui Ptolemeu 72

n patrulaterul inscriptibil M BA1 C , ob tinem succesiv: M A1 BC = M B A1 C + M C A1 B M A1 sin y = M B sin z + M C sin x M A1 M B sin z M C sin x = + . MA M A sin y M A sin y Scriind rela tiile analoage si adunndu-le, concluzia urmeaza imediat din faptul ca 1 a + 2, (0, ). a n cazul particular M = O, ob tinem inegalitatea remarcabila unde R este raza cercului circumscris 4ABC . Sa mai observa m ca egalitatea este atinsa n triunghiul echilateral. L9. Fie ABC un triunghi ascu titunghic cu a b c s i u, v, w (0, ), u v w. Daca uGA + vGB + wGC = (u + v + w) R, unde G este centrul de greutate al triunghiului, iar R este raza cercului circumscris, atunci triunghiul ABC este echilateral. Paul Georgescu si Gabriel Popa, Ia si Solu tie. Fie f : P R, f (M ) = u |zM zA | + v |zM zB | + w |zM zC |. 2z0 + zH 2 , din inegalitatea modulului ob tinem ca Deoarece zC = f (G) f (O) + 3 3 1 + f (H ). Din ipoteza , f (G) = f (O), deci f (H ) f (O). Pe de alta parte, aplicnd 3 h i [0, 1], ga inegalitatea lui Jensen func tiei concave cos : 0, sim ca 2 uA + vB + wC . f (H ) = 2R (u cos A + v cos B + w cos C ) 2R (u + v + w) cos u+v+w 1 Din inegalitatea lui Ceb sev, uA + vB + wC (u + v + w) (A + B + C ), deci 3 f (H ) 2R (u + v + w) cos = f (O). Am ob tinut ca si atunci este f (H ) = f (O ) 3 atinsa tile Jensen si Ceb sev, adica egalitatea n inegalita 4ABC este echilateral. L10. a) Fie n N , n 2. Sa se arate ca exista o progresie aritmetica de numere naturale care nu are nici un termen de forma xn , x N. b) Daca tine un termen de forma o progresie aritmetica de numere naturale con xn , x N, atunci sa tine o infinitate de termeni de aceasta se arate ca progresia con forma . Adrian Zanoschi, Ia si Solu tie. a) Sa tine demonstra m ca progresia aritmetica ak = 4k +2, k N, nu con nici un termen de forma xn , x N. ntr-adeva tiile: r, acest fapt rezulta din observa (4m)n = M4 , (4m + 1)n = M4 + 1, (4m + 2)n = M4 , (4m + 3)n = M4 1, m N. b) Fie o progresie de numere naturale cu ra tia r care con tine un termen xn , x N. n Numa rul natural ( x + r ) este termen al progresiei, deoarece (x + r)n = xn + nxn1 r + + rn = xn + nxn1 + + rn1 r. Analog se demonstreaza ca orice numa r de forma (x + kr) , k N, este termen al progresiei. 73
r

OA1 + OB1 + OC1 6R,

tia 2 3x = 3 2y + 174. L11. Sa se rezolve n N ecua Daniela Iosub, eleva si , Ia Solu tie. Cum x, y N , atunci a = x 1, b = y 1 sunt numere naturale. mpa tind ecua tia prin 6, ob tinem ecua tia echivalenta r 3a = 2b + 29, a, b N. Atunci b b a a 3 = (3 1) + 29, deci 3 = M3 + (1) + 29, prin urmare b trebuie sa fie par, b = 2k , k N (deoarece este evident ca tinem 3a = 4k + 29, b = 0 nu convine). Ob i.e. (4 1)a = 4k +29, de unde M4 +(1)a = 4k +29, adica sa fie par, a a trebuie = 2l, l N (a = 0 nu convine). n aceste condi tii, ecua tia devine 3l 2k 3l + 2k = 29 si cum 29 este prim, iar 3l 2k < 3l + 2k , ga sim ca 3l 2k = 1, 3l + 2k = 29. Sistemul astfel format nu are solu tii n N si atunci ecua tia ini tiala tii n N . nu are solu L12. Fie ABCD un patrulater convex; nota m {O} = AC BD, M mijlocul lui (AB ), N mijlocul lui (CD). Pentru propozi tiile P1 : ABCD inscriptibil; P2 : OM CD; P3 : ON AB , sa se arate c a : a ) P1 P2 P3 ; b) P2 P3 P1 ; c) P3 P1 P2 (n lega tura cu problema C:2265 din G. M. 3/2000). Viorel Cornea si Dan S tefan Marinescu, Hunedoara Solu tie. a) Daca ta ABCD inscriptibil, din puterea punctului O fa de cercul circumscris ob tinem ca OA OC = OB OD. Avem succesiv: 1 OA + OB OD OC = 0 P2 OM CD = 0 2 OA OD OA OC + OB OD OB OC = 0 OA OD OB OC + (OA OC OB OD) = 0 (1) OA OD OB OC (OA OC OB OD) = 0 OA OB OD + OC = 0 2 AB ON = 0 AB ON. b) Se procedeaza analog. c) Daca tine rela tia (1). Din ON AB deducem analog OM CD, se ob OD OB + OC OB OD OA + OA OC = 0

(2)

Adunnd (1) si (2), ga sim ca OA OC = OB OD, adica ABCD este inscriptibil.

L13. Fie P R [X ], P (X ) = a0 X n + a1 X n1 + + an1 X + an , n 2, cu a0 > 0 s i cu toate ra i subunitare. Sa da cinile pozitive s se arate ca (n 1) a0 + a1 + n + (1) an > 0. Gheorghe Molea, Curtea de Arge s a1 n n an Solu tie. Avem: (n 1) a0 + a1 + (1) an > 0 (1) < n1 a0 a0 P Qn ra n da cinile polinomului. i=1 xi i=1 xi < n 1, unde xi (0, 1), i = 1, n, suntQ Pn Prin induc tie completa ste u sor inegalitatea: 1 n , se dovede i=1 (1 bi ) < i=1 bi , bi (0, 1), i = 1, n, n 2. Lua si ob tinem: m n aceasta bi = 1 xi , i = 1, n, 1 q.e.d.
i=1 n Y

xi <

n X i=1

(1 xi ) 1

i=1

n Y

xi < n

n X i=1

xi

n X i=1

xi

i=1

n Y

xi < n 1,

74

X +1 1 1 ... 1 2 X 2 +2 2 . . . 2 . L14. Pentru n N considera m polinomul Pn (X )= ... ... ... ... ... n n n . . . X n +n n (n + 1) a) Ara ti ca a acestui polinom; ta zero este ra da cina multipla de ordin 2 b) Daca da cini reale nenule, iar daca n este impar, Pn are n este par, Pn nu are ra o singura i situata ra da cina reala nenula , care este simpla s n intervalul (2, 1]. Temistocle Brsan, Ia si Solu tie. Considernd ultima linie ca o suma de doua linii, avem: X + 1 1 ... 1 1 2 X2 + 2 . . . 2 2 ... ... ... ... Pn (X ) = X n Pn1 (X ) + n ... . n 1 n 1 . . . X n1 + (n 1) n 1 1 1 ... 1 1 Sca tita tinem rela tia znd ultima linie nmul respectiv cu 1, 2, . . . , n 1 din celelalte, ob de recuren ta Pn (X ) = X n Pn1 (X ) + nX n(n1)/2 , (1) din care se deduce, prin calcule de rutina , ca Pn (X ) = X n(n1)/2 Qn (X ) (2) (3) cu Qn (X ) = X n + X n1 + 2X n2 + + (n 1) X + n. Afirma tia a) rezulta si (3). Afirma tia b) n cazul n par rezulta direct din (2) scriind polinomul Qn sub forma 1 3 3 Qn (X ) = X n + X n1 + X n2 + X n2 + X n3 + X n4 + 2 2 2 5 n4 5 n3 2 n 3 + X + 5X n5 + X n6 + + X 4 + (n 3) X 3 + X + 2 2 2 2 n1 2 + X + (n 1) X + n 2 si observnd ca parantezele pa trate au discriminantul nul, iar cele rotunde strict negativ. Daca n este impar, verifica m mai nti ca Q0 m are valori pozitive pentru x < 0; ntr-adeva r, proceda m ca mai sus, observnd ca n1 Q0 + (n 1) X n2 + 2 (n 2) X n3 + + (n 2) 2X + (n 1) = n (X ) = nX 1 (n 1) n3 + X n3 + = nX n1 + 1 (n 1) X n2 + X 2 3 (n 3) n5 3 (n 3) n3 + X n5 + + + + 3 (n 3) X n4 + X X 2 2 (n 4) 4 2 (n 4) 4 4 + X + (n 4) 4X 3 + X + X 2+ 2 2 (n 2) 2 2 + X + (n 2) 2X + (n 1) . 2 75

Partea a doua a arma tiei b) rezulta din faptul ca Qn (x) > 0 pentru x > 0, Qn (2) Qn (1) < 0 si func tia x Qn (x) este strict cresca toare pentru x < 0. L15. Fie R\Z s i (cn )n1 un s ir convergent de numere reale. Sa se arate ca s irul (xn )n1 definit prin xn = {n + cn } nu este monoton. Iuliana Georgescu si Paul Georgescu, Ia si Solu tie. Sa observa m ca {x} {y } {x y } = {x} {y }. Presupunem ca sir este ma (xn )n1 este cresca tor. Atunci, deoarece acest rginit, el va fi convergent si xn+1 xn 0. Pe de alta parte, xn+1 xn = {(n + 1) + cn+1 } {n + cn } = = { + cn+1 cn } .Dar + cn+1 cn si R\Z, deci { + cn+1 cn } {} > 0. Prin urmare, xn+1 xn {} > 0. Absurd. L16. a) Fie a < b s ti ca i M = {f : [a, b] [a, b] ; f monotona }. Ara ta exista f M cu f (x) 6= x, x [a, b] s tie nu are proprietatea lui i ca orice asemenea func Darboux. b) Demonstra ti ca f M , c [a, b] astfel nct f (c) [a + b f (c)] = c (a + b c). S tefan Alexe, Pite sti Solu tie. a) Func tia f : [a, b] [a, b] definita prin f (x) = a + b x, daca x [a, (a + b) /2) si f (x) = a, daca tiile enun tului. x [(a + b) /2, b], satisface condi Presupunem ca tie f M fa si cu proprietatea lui Dar exista o func ra puncte fixe boux pe [a, b]. Fiind monotona ti doar de prima spe ta , f poate avea discontinuita ; avnd proprietatea lui Darboux, f nu are nici discontinuita ti de acest fel. Rezulta ca si tot a sa este si g : [a, b] R, g (x) = f (x) x. Cum f este continua pe [a, b] g (a) g (b) = [f (a) a] [f (b) b] 0, deducem ca c [a, b] astfel nct g (c) = 0, adica tiei f , ceea ce contrazice pre f (c) = c. Atunci c este un punct fix al func supunerea fa cuta . b) Fie f M . Presupunem ca si nota f este cresca toare m E = {x [a, b] ; f (x) x}. Observa si ma si E [a, b]). Ca urmare, m ca E este nevida rginita (a f (a) c = sup E si, evident, c [a, b]. Din x c, x E , deducem ca x f (x) f (c), x E . Deci f (c) este un majorant al mul timii E si avem c f (c). Cum f este cresca si, deci, toare, rezulta ca f (c) f (f (c)), de unde deducem ca f (c) E f (c) c. A sadar, f (c) = c (1). Daca f este descresca toare, atunci h : [a, b] [a, b], h (x) = a + b f (x) este cresca si, procednd ca toare mai sus, d [a, b] astfel nct h (d) = d, adica f (d) = = a + b d (2). Din (1) si (2) rezulta tia [f (x) x] [a + b x f (x)] = 0 sau ca f M ecua f (x) [a + b f (x)] = x (a + b x) are solu tii n [a, b], deci c [a, b] astfel nct f (c) [a + b f (c)] = c (a + b c). L17. Fie A un numa tie derivabila i f : [0, ) [0, ) o func r real pozitiv s pentru care f (0) = 0 s i |f 0 (x)| Af n (x), x [0, ), unde n este un numa r natural dat, n 1, iar f n = f f f . Atunci f este identic nula . Sorin Pu spana , Craiova Solu tie. Este suficient sa ar a t a m c a f este identic nul a pe orice interval de forma [0, ], > 0. Presupunem ca > 0 astfel nct f nu-i identic nul a pe [0, ], adica sterilor finite ob tinem: avem M > 0, unde M = sup f (x). Cu teorema cre x [0, ] are loc rela tia f (x) = xf 0 (c), unde c (0, x); deci f (x) = x |f 0 (c)| 76
x[0,]

xA |f n (c)| AM , x [0, ]. Ca urmare, f (x) AM , x [0, ], de unde M AM sau A 1. Fie = (0, 1 , 2 , . . . , m1 , ) o diviziune a intervalului [0, ] cu A kk < 1 (). Daca si ca mai sus f nu-i identic nula pe [0, 1 ], atunci M1 = sup f (x) > 0 ob tinem f (x) 1 AM1 , x [0, 1 ]; deducem ca M1 1 AM1 , adica 1 A 1, ceea ce contrazice (). n concluzie f este identic nula pe [0, 1 ]. n particular, avem f (1 ) = 0 si cu intervalul [1 , 2 ] proceda si cu [0, 1 ] etc. m la fel ca Dupa si, deducem ca un numa r finit de pa f este identic nula pe ntregul interval [0, ]. Presupunerea ini tiala fa cuta este falsa . n concluzie, f este identic nula pe [0, ).
x[0,1 ]

L18. Fie A Mn (Z), n N, n 2 astfel nct In + sA este inversabila i s 1 (In + sA) Mn (Z) pentru orice s {1, 2, . . . , n} . 1 a) Sa i (In + kA) pentru orice k Z s se arate ca In + kA este inversabila Mn (Z); b) Daca se arate ca G = {In + kA; k Z} este grup n raport cu A2 = On , sa nmul tirea matricelor s i sa se determine toate subgrupurile lui G. Marian Ionescu, Pite sti Solu tie. a) Se arata sor afirma tia: C Mn (Z) este inversabila u n Mn (Z) det C = 1. Fie P (x) = det (In + xA), grad P n, P Z (X ). Pentru orice s {0, 1, 2, . . . , 2n}, avem: Cs = In + sA este inversabila n Mn (Z) det Cs = = 1 P (s) = 1. Prin urmare, exista tin n + 1 numere s n care P ia cel pu valoare 1 sau cel pu tin n + 1 numere s n care P este 1. Considera m ca exista u1 , u2 , . . . , un+1 {0, 1, 2, . . . , 2n} astfel nct P (u1 ) = = P (u2 ) = = P (un+1 ) = 1; analog se procedeaza n cela lalt caz. Polinomul Q (X ) = P (X ) 1, de grad cel mult n, se anuleaza pentru n + 1 valori distincte, deci Q=0 si P = 1. Rezulta si, n consecin ta ca det (In + kA) = 1, k Z , concluzia dorita . b) Daca Cs = In + sA, Ct = In + tA, s, t Z, atunci, n ipotezele problemei, Cs Ct = Cs+t . Se verifica sor ca tirea matricelor u G este grup n raport cu nmul si ca (G, ) ' (Z, +) prin f : Z G, f (k ) = In + kA. Deoarce subgrupurile lui (Z, +) sunt de forma H = mZ, m 0, rezulta ca subgrupurile lui (G, ) au forma {In + mkA; k Z} cu m N. L19. Fie H un subgrup al grupului altern (A2002 , ). Daca 1 2 . . . 1999 2000 2001 2002 = H 1 2 . . . 1999 2001 2002 2000

s i 1 H , A2002 , sa se arate ca H = A2002 . Lucian-Georges La si dunca , Ia Solu tie. Se stie ca grupul altern (An , ), n 3, este generat de cicluri de lungime 3. Pentru a demonstra ca H = A2002 este sucient sa ara ta m ca orice astfel de ciclu se afla . Pentru aceasta, fie ( , , ) un 3 -ciclu oarecare ( , , {1, 2, . . . , 2002}). n H . . . a . . . b . . . 2000 2001 2002 , unde a, b, a0 , b0 sunt alese astFie = . . . a0 . . . b0 . . . fel nct A2002 . Se constata ca (, , ) = 1 H . 77

L20. Fie a R, a > 1. Se considera tia f : [1, a] R de doua func ori derivabila . Sa tia g : [1, a] R, g (x) = xf 0 (x) este monoton cresca toare, se arate ca daca func atunci Z a f (t) a ln a f dt. t 1 Marcel Chiri ta sti , Bucure Solu tie. Cum g este func tie cresca toare, rezulta ca g 0 0, adica f 0 (x)+xf 00 (x) 0, x [1, a]. Fie func tia h : [0, 1] R, h (x) = f (ax ). Avem: h0 (x) = ax ln a f 0 (ax ) si 00 x 2 h (x) = a ln a [f 0 (ax ) + ax f 00 (ax )] 0, x [1, a], de unde rezulta ca h este convexa . Conform inegalita tii lui Jensen, avem: x1 + x2 + + xn h (x1 ) + h (x2 ) + + h (xn ) h , x1 , x2 , . . . , xn [0, 1] . n n k si trecnd la limita tinem Pentru xk = , k = 1, n pentru n ob n Z 1 Z 1 1 h h (t) dt, adica a f (ax ) dx. f 2 0 0 n ultima rela tie efectua si ob tinem inegalitatea ceruta m schimbarea ax = t .

LISTA MEMBRILOR FILIALEI IA SI a S. S. M.1


- continuare din nr. 1/2000, 1/2001 si 1/2002 90. GALL Eduard 91. URSACHE Felicia-Camelia ATIC 92. LAM Lidia-Carmen 93. MACSIMIUC Delia 94. FARCA SANU Ana-Corina 95. BAICAN Tatiana 96. BUCATARU Mihaela 97. BUCATARU Ion Ines 98. CRETU 99. ASIMINOAIEI Ana 100. NAZARIE Elena 101. PSLARU Margareta Adriana Mihaela 102. BOTRCA Lucian-Georges 103. LADUNC A 104. GO SMAN Neculai 105. ONICIUC Carmen-Elena 106. LUPULEASA Iuliana Ecaterina 107. S TIURCA Alice 108. ANITA 109. PREDA Ani soara
1

Inginer, S.C. Easten, Ia si S coala gen. nr.36, Ia si Grupul S colar Agricol Holboca, Ia si S coala "Otilia Cazimir", Ia si S coala gen. nr.36, Ia si Colegiul "C.Negruzzi", Ia si Colegiul "E.Racovi ta si ", Ia Fac. de matematica si , Univ. "Al.I.Cuza", Ia S coala gen. nr.42, Ia si Liceul de chimie, Ia si Liceul de chimie, Ia si S coala prof. speciala si) , Tg. Frumos (Ia S coala gen. nr.10, Ia si Liceul de informatica si "Gr.Moisil", Ia S coala "G.Ibra si) ileanu", Tg.Frumos (Ia S coala nr.6 "M.Busuioc", Pa scani Grupul S colar "M.Sturza", Ia si Colegiul Na tional, Ia si S coala "D.D.Pa scanu", Tome sti (Ia si) tra

Lista va continuata n numerele urm a toare.

78

Probleme propuse
Clasele primare
P.44. Un vecinul al unui vecin al numa rului 81 este egal cu un vecin al unui vecin al numa rului 77 . Despre ce num a r este vorba? ( Clasa I ) Mihaela Rusu, eleva si , Ia P.45. Adunnd trei numere naturale a, b, c ob tinem suma 62. Primul numa r este mai mare dect al treilea si mpreuna au suma 12. Care sunt cele trei numere? ( Clasa a II-a) nv. Maria Racu, Ia si P.46. Mihai, Dan si Petru practica si anume: tenis, fiecare un alt fel de sport fotbal sau volei. Mihai si voleibalistul locuiesc n acela si bloc. Cel care joaca si volei cel care joaca fotbal l-au urma rit pe Petru la un meci. Ce sport practica fiecare? ( Clasa a II-a) Adina Dohotaru, eleva si , Ia P.47. Diferen ta a doua ta numere este 48. Aceasta diferen este cu 22 mai mare dect juma ti numerele. tatea unuia dintre ele. Determina ( Clasa a III-a) nv. Rodica Rotaru, Brlad P.48. Un agricultor mparte un teren n trei parcele. n fiecare an, fiecare parcela este cultivata numai cu una din culturile: gru, porumb sau legume. ncepnd cu anul 2003, agricultorul se hota ste ca pe fiecare parcela ra sa fie alta cultura n trei ani consecutivi. a) Care este primul an dupa 2003 n care se repeta culturile pe cele trei parcele? b) Se poate preciza care este ordinea culturilor pe cele trei parcele n anul 2019? ( Clasa a III-a) Andreea Surugiu, eleva si , Ia P.49. La un moment dat, cernd unei persoane anul na sterii, aceasta ra spunde: "anul acesta mplinesc 25 ani, iar daca s scrie toate numerele ncepnd cu 1 si a terminnd cu anul na sterii si apoi toate numerele ncepnd cu 1 si terminnd cu anul n care ne a am mi-ar trebui 13710 cifre. n ce an ne aam cnd am pus ntrebarea? ( Clasa a III-a) Prof. Ca si ta lin - Cristian Budeanu, Ia P.50. a) Cte numere trebuie ada sirului 1, 2, 4, 5, 7, 8, . . . , 97, 98 pentru a ugate ob tine toate numerele de la 1 la 98? b) Efectua ti 1 + 2 + 4 + 5 + 7 + 8 + + 97 + 98 2 (3 + 4 + 5 + + 34). ( Clasa a IV-a) Georgiana Ciobanu, eleva si , Ia P.51. Produsul a doua numere naturale este 913 368. Unul din numere are cifra unita tilor si cifra zecilor mai mare ca 2 si mai mica dect 8. Daca la acest numa r ma si mic sora tilor cu 1, ob tinem un produs egal cu rim cifra zecilor cu 2 m cifra unita 951 425. Aa ti cele doua numere. ( Clasa a IV-a) nv. Elena Za si rnescu, Ia P.52. n trei cutii sunt 212 bile. Din prima cutie se scoate un numa r de bile, din a doua de 2 ori mai mult si nca dou a bile, din a treia se scoate ct triplul numa rului de bile scos din a doua cutie. n fiecare cutie ra mne un num a r de bile egal cu numa rul total al bilelor scos din cele trei cutii la un loc. Cte bile au fost n fiecare cutie? ( Clasa a IV-a) nv. Maria Racu, Ia si P.53. Efectund o singura cnta rire, sa se ia 475g dintr-un kilogram de zaha r 79

utiliznd doua ti, una de 200g si cealalta greuta de 150g. ( Clasa a IV-a) Prof. Petru Asaftei, Ia si

Clasa a V-a
tite V.36. Fie n un numa r r impar, iar a1 , a2 , . . . , an , n N numere care mpa la n dau cturi distincte si resturi distincte. Ara ti ca ta valoarea minima a sumei S = a1 + a2 + + an este multiplu de 12. Drago s Ungureanu, elev, Ia si 333331 222221 V.37. Compara ti frac tiile a = si b = . 333334 222223 Maria Cojocaru, Ia si V.38. Sa se arate ca 2a + 2b + 2c + 2d + 2e 6= 2003, a, b, c, d, e N. Irina Ispas, studenta si , Ia V.39. Sa se determine numerele prime p1 < p2 < p3 < p4 astfel nct numerele p1 + p2 + p3 + p4 , p3 p2 , p4 p3 sa e, de asemenea, prime. Petru Minu t, Ia si V.40. Este posibila tionare a mul timii {1, 2, . . . , 12n + 9} n 2n + 3 sub o parti mul timi disjuncte, fiecare cu cte trei elemente, astfel nct n fiecare submul time un element sa fie suma celorlaltor dou a ? Titu Zvonaru, Bucure sti

Clasa a VI-a
VI.36. Fie k N, k 3. Ara ti ca ta printre valorile naturale ale lui n care fac . . n + k , exista tin trei pa adeva tia n2 + k . cel pu trate perfecte. rata propozi Claudiu S tefan Popa, Ia si VI.37. Numerele 1160, 1604 si 2270 dau acela si rest la mpa tirea prin n. Afla ti r mpa titorul n. r Cristian Laza si r, Ia VI.38. Demonstra ti ca tionale cu nu exista numere naturale x, y, z direct propor trei numere naturale consecutive, astfel nct x + y + z sa fie numa r prim. Alexandru Negrescu, elev, Boto sani VI.39. Radu si Mihai joaca stiga de mai multe ori un joc n urma ca ruia c torul prime ste a puncte, iar cel care pierde prime ste b puncte (a, b N , a > b). Daca scorul final este 6149 n favoarea lui Radu, iar Mihai a c stigat 4 partide, afla ti a si b. Adrian Zanoschi, Ia si b VI.40. Fie 4ABC cu m(A) = 120 . Perpendiculara n C pe AC intersecteaza mediatoarea lui [AB ] n D; nota m {E } = CD AB . Sa se arate ca AB = 2AC daca \ ) = 90 si numai daca si BE = 2AB . m(BDE Ioan Sa ca leanu, Hrla u

Clasa a VII-a
VII.36. Sa se arate ca

1 + n

2 + + n

2n 1 < 2n 1, n N, n 2. n Ca si ta lin Calistru, Ia

VII.37. Ara ti ca tie 7 printre numerele ce se scriu cu cifrele ta n baza de numera 0, 1, 2 exista o infinitate care sunt p a trate perfecte si o infinitate ce nu sunt pa trate 80

perfecte. Aceste afirma tii ra mn valabile daca se folosesc cifrele 3, 5, 6? Ruxandra Ioana Vlcu, eleva si , Ia abb . . . bc ac = ca cbb . . . ba (termenii primei frac tii con tinnd cte 2003 cifre b), atunci b = a + c. Mihaela Buca si taru, Ia VII.38. Fie a, b, c cifre nenule, a 6= c. Sa se arate ca daca VII.39. Daca x < y < z sunt lungimile laturilor unui triunghi dreptunghic, atunci xn + y n 6= z n , n N, n 3. Dumitru Neagu, Ia si b) = 60 , iar M Int ABC VII.40. Fie ABC un triunghi ascu titunghic cu m(A \ astfel nct m(BM C ) = 150 . Nota tiile lui M pe BC , CA si m cu P, Q, R proiec respectiv AB . Sa se arate ca 4P QR este dreptunghic. Constantin Cocea, Ia si

Clasa a VIII-a
VIII.36. Determina ti cardinalul minim al unei mul timi B pentru care putem defini func tii f : R B astfel nct f (1) < 0 si f (xy ) = f (x) f (y ), x, y R. Iulia Zanoschi, eleva si , Ia VIII.37. If a, b, c the following inequalities: (0, ) prove a) (a + b + c)3 a3 + b3 + c3 24 where abc = 1; 3 8 3 3 3 3 where ab + bc + ac = 1. b) (a + b + c) a + b + c 3 Zdravko Starc, Vr sac, Jugoslavia VIII.38. Fie n N fixat. Ara ti ca ta exista o infinitate de numere x, y, z Z astfel nct x2n + y 2n + z 2n = x2n+1 + y 2n+1 + z 2n+1 . Lucian Tu tescu, Craiova VIII.39. Fie ABCD un patrulater strmb cu [AD] [BC ]. Sa se construiasca dreptele paralele d1 , d2 , d3 , d4 astfel nct A d1 , B d2 , C d3 , D d4 si dist (d1 , d4 ) = dist (d2 , d3 ). Horia Mihail Teodorescu, elev, Ia si si C 0 O VIII.40. Fie ABCDA0 B 0 C 0 D0 un cub, iar O (BB 0 ). Dreptele A0 O 0 0 0 intersecteaza si CO intersecteaza (ABC ) n E , respectiv F , iar AO (A B C ) n E 0 , 0 respectiv F . a) Ara ti ca tia lui O; ta EF E 0 F 0 nu depinde de pozi b) Ara ti ca si determina ti O pentru care se atinge egalitatea. ta SBB 0 E0 E SABCD Monica Nedelcu, Ia si

Clasa a IX-a

y IX.36. Determina ti x < 0 < y astfel nct xy + = y 3 5y + 2. x Cezar Lupu, elev, Constan ta

ti inegalitatea IX.37. Pentru x [1, ), n N , demonstra n n+1 + 1 (x 1) 2nxn (x 1) . x 81

Marius Pachi tariu, elev, Ia si

xn+1 y n+1 z n+1 + n + n x + y + z , x, y, z > 0, n N. yn z x Gigel Buth, Satu Mare 1 2 1 . + q = IX.39. Sa tia q se rezolve ecua [x] [x + 2] 2 [x]3 3 3 [x] [x + 1]3 Daniel Jinga, Pite sti IX.40. Fie M 6= G n planul 4ABC si D, E, F mijloacele laturilor [BC ], [CA] si respectiv [AB ]. Considera m punctele X, Y, Z astfel nct XD = mXM , Y E = mY M , ZF = mZM , m 6= 1. 2m 3 a) Daca SM . m 6= , atunci AX, BY, CZ sunt concurente n S , cu SG = 2 3 3 b) Daca m = , atunci AX, BY, CZ sunt paralele cu GM . 2 Virgil Nicula, Bucure sti IX.38. Sa se arate ca

Clasa a X-a

X.36. Sa tia alogb x + xlogb x a + b, unde a, b (1, ). se rezolve inecua Daniela Dodan, eleva si , Ia X.37. Fie a, b (0, 1) (1, ) si func tia injectiva f : (0, ) R astfel nct si ca func tia g : R R, g (x) = f (ax ) + f (bx ) este constanta . Sa se arate ca ab = 1 exista tii f care satisfac ipotezele problemei. func Dan Popescu, Suceava X.38. Fie a, b, c, d R cu a > b > c > d. Sa ca a, b, c, d sunt n progresie se arate 3 ad aritmetica si numai daca . daca (a b) (b c) (c d) = 3 A. V. Mihai, Bucure sti 0 0 0 0 X.39. Fie ABCDA B C D un paralelipiped dreptunghic cu dimensiunile AB = a, AD = b, AA0 = c. Daca M Int A0 B 0 C 0 D0 , nota m cu , , ma surile unghiurilor pe care AM le face cu AB , AD si respectiv AA0 . Sa se arate ca AM < a cos + b cos + c cos < AC 0 . Ca si ta lin Calistru, Ia X.40. a) Pentru x, y, z 0, demonstra ti inegalitatea p x + y + x + z + y + z xy + xz + yz 3 6xyz. b) Cu nota tiile uzuale, n orice triunghi are loc inegalitatea 2 2 2 a b + ( a c) + b c 9 R . 2 2 r 4 a+ b+ c Marian Tetiva, Brlad

Clasa a XI-a
XI.36. Fie D, M doua matrice nesingulare de ordin n, D diagonala , iar M t M DM , s a se arate c a M este tot o matrice diagonal a triunghiulara . Dac a D = , avnd 1 pe diagonala principala . Adrian Corduneanu, Ia si 82

(xn )n1 R ti ca sirul xn xn+1 (1 + xn yn+1 ), n 1, ara ta + are proprietatea ca 1 este convergent. xn n1 Gheorghe Molea, Curtea de Arge s 3 = x XI.40. Fie x0 [1, 1]; ara ta t i c a pentru orice n N , ecua t ia 3 x 4 x n 1 1 n ti ca sirurile (xn )n0 are o singura tie xn+1 , . Demonstra si (3 xn )n0 solu 2 2 sunt convergente si calcula ti limitele lor. Marian Tetiva, Brlad

XI.37. Fie A M3 (C) astfel nct det (A + tA) = 0, unde R\{1, 0, 1}. 2 ( 1)2 det A. Sa se arate ca det (A + tA) = Marian Ionescu, Pite sti si Lucian Tu tescu, Craiova XI.38. Sa tiile continue f : [0, ) [0, ) pentru care se determine func f (f (x)) + 2f (x) = 3x, x 0. Mihail Bencze, Bra sov n P XI.39. Fie sirul (yn )n1 astfel nct sirul yi este convergent. Daca
i=1 n1

1 are solu tie XII.36. Sa tia x2 = x + b se determine n N, n 2 pentru care ecua unica ti ecua tia n acest caz. n Zn ; rezolva Andrei Nedelcu, Ia si XII.37. Fie (G, +) un subgrup al grupului (R, +). Sa se determine morfismele cresca toare de la (G, +) la (R, +). Dan S tefan Marinescu si Viorel Cornea, Hunedoara XII.38. Determina ti func tiile derivabile f, g : R R astfel nct f 0 (x) = g (x)+x 0 si g (x) = f (x) x, x R. Gheorghe Iurea, Ia si XII.39. Fie f, g : (0, ) R astfel nct lim f (x) = lim g (x) = , iar x x Z 1 f (x) xg(n) lim = R. Sa dx, unde [1, ). se calculeze lim f (n) x g (x) n 0 x+ Adrian Sandovici, Piatra Neam t XII.40. Fie f : [0, 1] R o func tie derivabila cu derivata continua astfel nct f (x) 0 exista si este finita xf (x) f (x), x [0, 1], iar lim . Sa se arate ca x0 x x>0 Z 1 Z 1 f (x) f (x) dx, f (1) min 2 dx . x 0 0 Marcel Chiri ta sti , Bucure

Clasa a XII-a

83

Probleme pentru prega tirea concursurilor


A. Nivel gimnazial
G36. Fie x, n N astfel nct x divide 10n 1, nsa x nu divide 10k 1 pentru . k < n. Sa si numai daca . n. se arate ca x divide 10m 1 daca m. N. N. Hr tan, Ia si G37. 2n muzicieni (n > 2) participa la un festival. La fiecare concert, o parte dintre ei cnta ti asculta iar ceilal . Sa se determine numa rul minim de concerte astfel nct fiecare muzician sa ti ceilal ti. -i asculte pe to Titu Zvonaru, Bucure sti G38. Mul timea A Z are cinci elemente. Adunnd n toate modurile posibile cte trei elemente din mul time, ob tinem urma toarele 10 sume: 3, 6, 8, 10, 11, 13, 15, 16, 18, 20. Determina ti mul timea A. (n lega tura cu o problema de concurs din Iugoslavia.) Gabriel Popa, Ia si G39. Fie xi R, i = 1, n, unde n 2003, astfel nct x1 (n + 1) x2 + nx3 n 1 .................................... xn2 (n + 1) xn1 + nxn n 1 xn1 (n + 1) xn + nx1 n 1 n2 xn (n + 1) x1 + nx2 2n 1. Daca x1 = 1, sa se calculeze x2003 . Romeo Cernat, Ia si 2 2 G40. Compara ti numerele reale a si b, stiind ca a 14a + b + 6b + 33 = 0. Bogdan Ra si ducanu, elev, Ia G41. Daca 0 < x y z , sa se arate ca x z x y x x2 y 2 z2 y z z 3 + + + + + 1 + 2 + 2 + 2. z y x y z x z x y z x Ovidiu Pop, Satu Mare G42. Determina ti a, b R daca [x] + [x + a] = [bx], x R. Gheorghe Iurea, Ia si d G43. Fie xOy un unghi oarecare si P un punct n interiorul sa u. Se considera punctele A, B [Ox cu A (OB ) si C, D [Oy cu C (OD) astfal nct triunghiurile P AB si P CD sa ti ca si BC sunt fie echilaterale. Ara ta dreptele OP , AD concurente daca si numai daca P se afla pe bisectoarea unghiului dat. Temistocle Brsan, Ia si G44. Fie V ABC o piramida , iar G centrul de greutate al 4ABC . Un plan ce trece prin G taie dreptele V A, V B, V C n A0 , B 0 si respectiv C 0 . Sa se arate ca VA VB VC + + = 3. V A0 V B0 V C0 Constantin Cocea, Ia si G45. Fie SABC un tetraedru n care 4ABC nu este echilateral, iar muchiile [SA] , [SB ] , [SC ] nu sunt toate congruente. Demonstra ti ca sase puncte exista si respectiv AB astfel ca paA1 , B1 , C1 , A2 , B2 , C2 pe dreptele SA, SB , SC , BC , AC trulaterele A1 B1 A2 B2 , B1 C1 B2 C2 si A1 C1 A2 C2 sa fie trapeze izoscele (A1 B1 kA2 B2 , 84

A1 C1 kA2 C2 , B1 C1 kB2 C2 ) daca si numai daca SA2 AB 2 AC 2 + SB 2 BC 2 BA2 + SC 2 CA2 CB 2 = 0. Daly Marciuc, Satu Mare

B. Nivel liceal

L36. Fie 4ABC si M triunghiul sa u median. Daca P este un punct aflat n tiile dreptelor AP , BP , interiorul sau pe laturile lui M, iar A0 , B 0 , C 0 sunt intersec 1 AP BP CP 8 CP cu laturile BC , CA si respectiv AB , atunci < . 4 AA0 BB 0 CC 0 27 Marian Ionescu, Pite sti si C astfel nct C1 si C2 sunt tangente exterior n D, iar L37. Fie cercurile C1 , C2 cercurile C1 si C2 sunt tangente interior lui C n B , respectiv C . Tangenta comuna interioara si A1 , dreapta AB taie C1 n K , iar si C2 taie cercul C n A cercurilor C1 1 2 1 = AC taie C2 n L. Sa + . se arate ca DA DA1 KL Neculai Roman, Mirce sti (Ia si) L38. Fie 4ABC si punctele D, D0 BC conjugate armonic n raport cu vrfurile si AC n N . Ara ti B si C . Cercul circumscris 4ADD0 intersecteaza AB n M ta 0 b ca , dac a M N BC , atunci [ AD s i [ AD sunt bisectoarele unghiului A (interioar a s i b exterioara ) sau m(A) = 90 . Temistocle Brsan, Ia si an (an + 2) L39. Determina ti toate numerele naturale nenule n pentru care este p (p + 1) pa trat perfect, unde a, p N . Mihai Haivas, Ia si 2 2 L40. Fie A, B Mn (Z) astfel nct det A B + AB este impar. Sa se arate ca A + B este inversabila pentru orice Q. Marian Ursa rescu, Roman L41. Demonstra ti ca grupul simetric S32 nu are elemente de ordin 2002. Paul Georgescu si Gabriel Popa, Ia si L42. Fie ( A, + , ) un inel finit cu cel pu t in 5 elemente s i cu 1 + 1 A inversabil. Fie M = x A | x2 = 1 , I = x A | x2 = x . Sa se arate ca card M = card I < < card A /2. Ovidiu Munteanu, Bra sov L43. Determina ti polinoamele P R [X ] pentru care P (z ) C\R, z C\R. Gheorghe Iurea, Ia si sir de polinoame denit L44. Fie n 2 numa r natural, iar f0 , f1 , f2 , . . . un p1 0 prin: f0 = (X + 1)n , fp+1 = X fp , p 0. Definim nca hp = fp 1 fp1 + X 1 n + + (1)p1 p i1 i2 . . . ik , k {1, 2, . . . , n}, p1 f1 , p 1, unde k = sunt sumele simetrice fundamentale ale numerelor 1, 2, . . . , n. Sa se arate ca hp = n (n 1) . . . (n p + 1) X p (X + 1)np , p = 1, 2, . . . Marian Tetiva, Brlad L45. Fie f : [0, ) [0, ) continua . Daca func tia F : [0, ) R, F (x) = Z 1 Z x f (t) dt este ma = rginita , sa se arate ca lim n xf (nx) dx = 0. n 0 0 Adrian Zanoschi, Ia si 85
1i1 <<ik n

Pagina rezolvitorilor
BOTO SANI Scoala nr. 7 "Octav Ba ncila ". Clasa a VIII-a. NEGRESCU Alexandru: VI(2628,30,32,34), VII(26,32), VIII(26,27,32,34), G(7,11,32). BRA SOV Raluca: V(26,31,33), VI(34, Scoala generala nr. 5. Clasa a VI-a. POSTEUCA Bogdan: V(26,31,33), VI(34,35), VII(34). 35), VII(34). Clasa a VII-a. POSTEUCA Scoala generala nr. 20. Clasa a VII-a. BOERIU Adela: VI(31-35), VII(32,35). Liceul "N. Titulescu". Clasa a IX-a. ANDRA S Cristian: VII(26,29,32,34,35), VIII(32); BORICEAN Mihai: VII(26,29,32,34,35), VIII(32,33); BURLACU Alen: Andreea: VII(26,29,32,34,35), VIII VII(26,29,32,34,35), VIII(27,32,33); CIOBOTA Cristian: VII(26,29,32,34,35), VIII(32,33); CMPEAN Simona: (32,33); CIOBOTA VII(26,29,32,34,35), VIII(27,32,33); COSTEA Rodica: VII(26,29,34), VIII(32,33); FERAR Achim: VII(26,29,32,34,35), VIII(27,32,33); FUNDUREANU Alexandra: VII(26,29,32,34,35), VIII(32,33); GHILIFTOIU Mirela: VII(26,29,32,34,35); GOICEA Ovidiu: VII(26,29,32,34,35), VIII(27,32, 33); MANEA George: VII(26,29,32,34, 35), VIII(32); MIHALCEA Ca ta lin: VII(26, 29,32,34,35), VIII(32); MNZAT Mihai: VII(26,29,32,34,35), VIII(27,32,33); MUNTEAN Alexandru: VII(26,29,32,35), VIII(32); MUNTEANU Lumini ta: VII(26,32,24), VIII(32,33); PUCHEANU Bogdan: VII(26,29,32, 34,35), VIII(27,32,33); R SCU Laura: VII(26,27,29,34,35), VIII(32,33); VLAD Daniel: VII(26,29,32,34,35), VIII(27). CRAIOVA Colegiul Na tional "Fra tii Buze sti". Clasa a VI-a. TUTESCU Anca- Stefania: V(31,35), VI(31,32,34), VII(31,32), G(21); Clasa a VIII-a. DINU Lavinia: VII(31), VIII(31,32), G(21,23,26). FOC SANI Andreea: VI(31-35), Colegiul Na tional "Unirea". Clasa a VIII-a. SECARA VII(31,32), VIII(32,34,35), G(21,23,25). (IA HRLAU SI) Liceul Teoretic "Stefan cel Mare". Clasa a VII-a. ANTOCI Bogdan: V(27,28, 30), VI(29,33,34,35), VII(29,32); BURICAN Bogdan Alexandru: V(28,30), VI(26,29), Lucian: V(27,28,30,32), VI(29,34,35), VII(29,31,32); PASCARIU VII(29); MIHULCA Marian-Drago s: V(27,28,30,32,33), VI(29,33-35), VII(29); ROTARU Lucian: V(2628,32), VI(29,32-35), VII(29). IA SI Colegiul Na tional "C.Negruzzi". Clasa a VI-a. RO SU Eugenia: V(31-35), VI(31-35), VII(32), VIII(32), G(21,23,27). Clasa a X-a. IACOB Alin: IX(32,34,35), L(21,28,29). Liceul "Garabet Ibra tefana: P(41,42), ileanu". Clasa a VI-a. BUDEANU S V(31-33), VI(31); FUIOREA Bogdan: P(42), V(31,32), VI(31,32); UNGUREANU Drago s: P(42), V(31-33), VI(31,32). Clasa a VIII-a. ANDRIESCU Alina: VI(33 STEANU S 35), VII(32), VIII(34); BRANI tefana:VI(35), VII(31,32), G(22,23); JUVERDEANU George: VI(33,34), VII(32,35), VIII(34); MORO SANU Mircea: VI(33 35), VII(31,32); TANASE Ioana: VI(33-35), VII(31,32); TUDORACHI Lucia: VI(34, 86

35), VII(31,32), VIII(32). Clasa a X-a. TONU Constantin: VIII(34), IX(28), X(33), G(32,33). Liceul Teoretic "M.Eminescu". Clasa a VIII-a. AVRAM Mircea: VI(25,27,3335), VII(26,28,29,32),VIII(27); CIUCANU Radu: VI(27,34,35), VII(26,29,32), VIII AIL A Mihai: VI(27,33-35), VII(26,29,32), VIII(27); DU (27); DAN SA Cristian: VI (27,34), VII(31,32), VIII(33); TOFAN Andrei: VI(27,34,35), VII(26,29,32), VIII(27); TUDOSE S tefan: VI(27,34,35), VII(26,32), VIII(27); TURLIUC Rare s: VI(31,3335), VII(29); GRAMSCHI Raluca: VI(26,27,34,35) (4 probleme). Clasa a IX-a. DUMITRESCU Roxana: VII(26,29,31,32), VIII(31,32,34,35), IX(31). OanaScoala "G.Co sbuc". Clasa a II-a (nv. GALIA Paraschiva). CIOABA Ca t a lina: P(24,27,34-36); MIH AILESCU Laura-Ioana: P(24,27,34-36); SCUTARU Ioana: Constantin: P(24,27,34-36). Clasa a II-a (nv. RACU Maria). BARABULA P(24,27,34-36); BURLACU Claudiu: P(24,27,34-36); CALOIAN Andrei: P(24,27,34 36); CALIN Georgiana: P(24,27,34-36); CRACIUN Ma da lina: P(24,27,34-36); LEA Crina-Alexandra: P(24,27,34-36); MOISA Bogdan: P(24,27,34-36); PINTILIE GAN Ra zvan: P(24,27,34-36); RUSU Flavia: P(24,27,34-36). Scoala "Al. Vlahu ta ". Clasa a IV-a (nv. MAXIM Gabriela). CIOCOIU AdrianFlorin: P(34-42); MUNTEANU Ioana-Alexandra: P(34-42); SOFICU Crina-Maria: P(35-37,39,40); STURZU Tudor-Nicolae: P(33-42). Scoala "Alexandru cel Bun". Clasa a II-a (nv. SPNU Doini ta). BURLACU Ionu t-Mihai: P(24,25,27,33-35); COJOCARIU Oana-Alexandra: P(24, 25,27,33-37); COJOCARU Veronica: P(24,25,27,33-36); DAMIAN Daniel: P(24,25,27,33-36); FLO Marta: P(24, 25,27,33,35); IFTEREA Roxana-Maria: P(24,25,27,33-37); FURTUNA A NIE Ioana: P(24,25,27,34-36); IVANOV Alla: P(24,25,27,33-36); MARINCU T A Tofana-Maria: P(24,25,27,33-37); MOCANU Mihai: P(24,25,27,33-36); MIHAIL Ciprian: P(24,25,27,33-37); PATRA SC Ilinca: P(24,25,27,33,36,37); RUSU Alexandru: P(24,25,27,33-37); SRBU Silviu Alexandru: P(24,25,27,33,35,37); SITARU Andreea: P(24,25,27,33-37); URSU Gina-Ioana: P(24,25,27,33-36). Scoala "B.P.Ha sdeu". Clasa a IV-a (nv. S TEFAN Liviu). PINTILIE Mina Liviu: P(24-31); PINTILIE Nicoleta: P(24-30); S TERBULEAC Daniel: P(24,2631). Clasa a IV-a (nv. TRZIORU Iuliana). CHIHAIA Mihai-Sebastian: P(34 Bogdan: P(34-43); SILION Ca A 43); RAIT s-Andrei: ta lina: P(34-42); SPNU Drago P(33-43). Scoala "N.Tonitza". Clasa a III-a (nv. MARCU Monica). BUTNARU Valentin: Alin: P(35-37,39,40). Clasa a IV-a (nv. ZARNESCU A P(35-37,39,40); ONUT Loredana: P(34,38-42); BONCU Andrei: P(34,38-41). Elena). ANDRU SCA Beatrice). TUDORACHE Scoala "T.Maiorescu". Clasa a III-a (nv. CHIRILA Alexandru-Gabriel: P(34-41). Dumitru). TIBA Marius: Scoala "O.Cazimir". Clasa a IV-a (nv. PRIALA P(38-43). PLOIE STI Colegiul Na tional "I.L.Cargiale". Clasa a VI-a. JELEA Anca: P(23,31), V(21,23,28) (solu tiile au fost primite nainte de apari tia nr. 1/2002).

87

IMPORTANT
n scopul unei lega tia revistei, pot fi utilizate urma turi rapide cu redac toarele adrese e-mail: tbi@math.tuiasi.ro, popagabriel@go.com . Pe aceasta tia un dialog privitor la cale colaboratorii pot purta cu redac materialele trimise acesteia, procurarea numerelor revistei etc. La problemele de tip L se primesc solu tii de la orice iubitor de matematici elementare (indiferent de preocupare profesionala sau vrsta ). Fiecare dintre solu tiile acestor probleme - ce sunt publicate n revista dupa un an - va fi urmata de numele tuturor celor care au rezolvat-o. Adresa ta m cu insisten ruga mintea ca materialele trimise revistei sa si altor publica tii. nu fie (sa nu fi fost) trimise

Redac tia revistei "Recrea tii matematice" acorda si un cte o diploma premiu n ca ti urma r torilor elevi: ANDRIESCU Alina (Lic. "G. Ibra ileanu", cl. a VIII-a): 2/2001 (5pb), 2/2002 (5pb), 1/2003 (5pb); STEANU S BRANI tefana (Lic. "G.Ibra ileanu", cl. a VIII-a): 2/2001 (5pb), 2/2002 (11pb), 1/2003 (5pb); BUDEANU S tefana (Lic. "G. Ibra ileanu", cl. a VI-a): 1/2002 (6pb), 2/2002 (5pb), 1/2003 (6pb); CHIHAIA Mihai - Sebastian ( Sc. "B. P. Ha sdeu", cl. a IV-a): 1/2002 (9pb), 2/2002 (9pb), 1/2003 (10pb); JUVERDEANU George (Lic. "G. Ibra ileanu", cl. a VIII-a): 1/2002 (6pb), 2/2002 (5pb), 1/2003 (5pb); Bogdan ( A RAIT Sc. "B. P. Ha sdeu", cl. a IV-a): 1/2002 (10pb), 2/2002 (9pb), 1/2003 (10pb); SILION Ca Sc. "B. P. Ha sdeu", cl. a IV-a): 1/2002 (10pb), 2/2002 (9pb), ta lin ( 1/2003 (9pb); SPNU Drago s - Andrei ( Sc. "B. P. Ha sdeu", cl. a IV-a): 1/2002 (10pb), 2/2002 (9pb), 1/2003 (11pb); TUDORACHE Alexandru - Gabriel ( Sc. "T. Maiorescu", cl. a IV-a): 1/2002 (12pb), 2/2002 (7pb), 1/2003 (8pb); TUDOSE S tefan (Lic. "M. Eminescu", cl. a VIII-a): 1/2001 (5pb), 1/2002 (5pb), 1/2003 (5pb); TUTESCU Anca S tefania (Colegiul Na tional "Fra tii Buze sti", Craiova, cl. a VI-a): 1/2002 (6pb), 2/2002 (8pb), 1/2003 (8pb); UNGUREANU Bogdan (Lic. "G. Ibra ileanu", cl. a VI-a): 1/2002 (7pb), 2/2002 (7pb), 1/2003 (6pb), autor al problemei V.36. Ca tile au fost oferite de revista "Recrea tii matematice" si r

88

S-ar putea să vă placă și